Vous êtes sur la page 1sur 259

GV: Le c Thanh

Chng 1

CAC KHAI NIEM C BAN


1.1 KHAI NIEM VE MON HOC SC BEN VAT LIEU ( SBVL )OI TNG, NHIEM VU, AC IEM CUA MON SBVL
1.1.1 OI TNG NGHIEN CU CUA SBVL- HNH DANG VAT THE
SBVL nghien cu vat the thc ( cong trnh, chi tiet may )
Vat the thc co bien dang di tac dung cua nguyen nhan ngoai
( tai trong, nhiet o, lap rap cac chi tiet che tao khong chnh xac)
Vat the thc s dung trong ky thuat c chia ra ba loai c ban:
Khoi: co kch thc theo ba phng tng ng: e ap, mong may...

H. 1.1 Vat the dang khoi

H. 1.2 Vat the dang tam vo

Tam va vo: vat the mong co kch thc theo mot phng rat nho so vi hai
phng con lai; tam co dang phang, vo co dang cong: san nha, mai vo
Thanh: vat the dai co kch thc theo mot phng rat ln so vi hai
phng con lai: thanh dan cau, cot ien, truc may SBVL nghien cu
thanh, he thanh.
Thanh c bieu dien bang truc thanh va
mat cat ngang F vuong goc vi truc thanh
(H.1.3).
Truc thanh la quy tch cua trong tam mat cat
ngang.
Cac loai thanh (H.1.4):
+Thanh thang, cong: truc thanh thang,
a)
cong,
+He thanh : thanh gay khuc
b)
(phang hay khong gian)
H. 1.4

Chng 1: Khai niem c ban

H. 1.3 Truc thanh va mat


cat ngang

c)

Ca c dang truc thanh

http://www.ebook.edu.vn
1

d)

GV: Le c Thanh

1.1.2 Nhiem vu: SBVL la mon hoc ky thuat c s, nghien cu tnh chat
chu lc cua vat lieu e e ra cac phng phap tnh cac vat the chu cac
tac dung cua cac nguyen nhan ngoai, nham thoa man yeu cau an toan va
tiet kiem vat lieu.
Vat the lam viec c an toan khi:
- Thoa ieu kien ben : khong b pha hoai (nt gay, sup o).
- Thoa ieu kien cng: bien dang va chuyen v nam trong mot gii
han cho phep.
- Thoa ieu kien on nh : bao toan hnh thc bien dang ban au.
Thng, kch thc cua vat the ln th kha nang chu lc cung tang va
do o o an toan cung c nang cao; tuy nhien, vat lieu phai dung nhieu
hn nen nang ne va ton kem hn. Kien thc cua SBVL giup giai quyet hp
ly mau thuan gia yeu cau an toan va tiet kiem vat lieu.
Ba bai toan c ban cua SBVL:
+ Kiem tra cac ieu kien ben, cng, on nh.(Tham ke)
+ nh kch thc, hnh dang hp ly cua cong trnh hay chi tiet may.
+ nh gia tr cua cac nguyen nhan ngoai ( tai trong, nhiet o) cho
phep tac dung ( Sa cha)
1.1.3 ac iem:
SBVL la mon khoa hoc thc nghiem: e am bao s tin cay cua cac
phng phap tnh, mon hoc ket hp chat che gia nghien cu thc nghiem
va suy luan ly thuyet.
Nghien cu thc nghiem nham phat hien ra tnh chat ng x cua cac
vat lieu vi cac dang chu lc khac nhau, lam c s e xuat cac gia thiet
n gian hn e xay dng ly thuyet. V vay, ly thuyet SBVL mang tnh gan
ung.
Th nghiem kiem tra cac ly thuyet tnh toan a xay dng
Trong nhieu trng hp, phai lam th nghiem tren mo hnh cong trnh
thu nho trc khi xay dng hoac th tai cong trnh trc khi s dung.
SBVL khao sat noi lc ( lc ben trong vat the ) va bien dang cua vat
the ( C Ly Thuyet khao sat can bang va chuyen ong cua vat the).
SBVL cung s dung cac ket qua cua C Ly Thuyet

Chng 1: Khai niem c ban

http://www.ebook.edu.vn
2

GV: Le c Thanh

1.2 NGOAI LC- CAC LOAI LIEN KET- PHAN LC LIEN KET
1.2.1 Ngoai lc

Tai trong

a) nh ngha: Ngoai lc la lc
tac ong t moi trng hoac vat the
ben ngoai len vat the ang xet.
b) Phan loai :
Tai trong : a biet trc (v tr,

Phan lc
H. 1.5 Tai trong va phan lc

phng va o ln), thng c quy nh bi cac quy pham thiet ke hoac


tnh toan theo trang thai chu lc cua vat the. Tai trong gom:
+Lc phan bo: tac dung tren mot the
tch, mot dien tch cua vat the ( trong lng
ban than, ap lc nc len thanh be...)
Lc phan bo the tch co th nguyen la
lc/the tch,hay [F/L3].
Lc phan bo dien tch co th nguyen la
lc/dien tch, hay [F/L2].
Neu lc phan bo tren mot dai hep th thay
lc phan bo dien tch bang lc phan bo ng
vi cng o lc co th nguyen la lc/chieu
dai, hay [F/L] (H.1.6). Lc phan bo
ng la loai lc thng gap trong SBVL.

H. 1.6 Cac loai lc phan


bo

+Lc tap trung: tac dung tai mot iem


cua vat the, th nguyen [F]. Thc te, khi dien tch truyen lc be co the coi
nh lc truyen qua mot iem
+ Momen (ngau lc) co th nguyen la lc x chieu dai hay [FxL]
Phan lc : la nhng lc thu ong (phu thuoc vao tai trong), phat sinh tai
v tr lien ket vat the ang xet vi cac vat the khac.
c) Tnh chat tai trong
Tai trong tnh: bien oi cham hay khong oi theo thi gian, bo qua gia
toc chuyen ong (bo qua lc quan tnh khi xet can bang). Ap lc at len
tng chan, trong lng cua cong trnh la cac lc tnh
Tai trong ong: lc thay oi nhanh theo thi gian, gay ra chuyen ong
co gia toc ln ( rung ong do mot ong c gay ra, va cham cua bua xuong
au coc). Vi lc ong th can xet en s tham gia cua lc quan tnh .
Chng 1: Khai niem c ban

http://www.ebook.edu.vn
3

GV: Le c Thanh

1.2.2 Lien ket phang, phan lc lien ket, cach xac nh


1.2.2.1 Cac loai lien ket phang va phan lc lien ket:
Mot thanh muon duy tr hnh dang, v tr ban au khi chu tac ong cua
ngoai lc th no phai c lien ket vi vat the khac hoac vi at.
Goi di ong (lien ket
thanh): ngan can mot chuyen v
thang va phat sinh mot phan lc
R theo phng cua lien ket

(H.1.7a)

Goi co nh ( Lien ket

khp, khp, ban le) : ngan can

a)

b)

chuyen v

thang theo phng

V
c)

H. 1.7 Lien ket va phan lc lie n ke t

bat ky va phat sinh phan lc R cung theo phng o. Phan lc R thng


c phan tch ra hai thanh phan V va H (H.1.7b)
Ngam: ngan can tat ca chuyen v thang va chuyen v xoay. Phan lc
phat sinh trong ngam gom ba thanh phan V, H va M (H.1.7c)
1.2.2.2 Cach xac nh phan lc:
Giai phong cac lien ket, thay bang cac phan lc tng ng, cac phan
lc c xac nh t ieu kien can bang tnh hoc gia tai trong va phan lc.
Bai toan phang co ba phng trnh can bang oc lap, c thiet lap
cac dang khac nhau nh sau:
= 0 (2 phng X, Y khong song song)

1.

X = 0; Y = 0; M

2.

M A = 0; M B = 0; MC = 0 ( 3 iemA, B, C khong thang hang)

3.

X = 0; M A = 0; M B = 0 (phng AB khong vuong goc vi X)

Bai toan khong gian co sau phng trnh can bang oc lap, thng
co dang:
X = 0; Y = 0; Z = 0; M / Ox = 0; M / Oy = 0; M / Oz = 0
Chu y:e co nh mot thanh trong mp can toi thieu 3 lien ket n e chong
lai 3 chuyen ong t do. Neu u lien ket va bo tr hp ly 3 phan lc se tm
c t 3 ptcb tnh hoc.Thanh c goi la tnh nh. Neu so lien ket tng
ng ln hn 3 goi la bai toan sieu tnh.

Chng 1: Khai niem c ban

http://www.ebook.edu.vn
4

GV: Le c Thanh

1.3 CAC DANG CHU LC VA BIEN DANG C BAN CHUYEN V


1.3.1Bien dang cua vat the:
Trong thc te, s chu lc cua mot thanh co the phan tch ra cac
dang chu lc c ban:
Truc thanh khi chu keo (nen) se dan dai (co ngan) (H.1.8a,b)
Truc thanh chu uon se b cong (H.1.8e)
Thanh chu xoan th truc thanh van thang nhng ng sinh tren
be mat tr thanh ng xoan tru (H1.8.d).
Khi chu cat, hai phan cua thanh co xu hng trt oi vi nhau
(H1.8.c).
P

2P

a)

dx

c)

dx

a)

T1

b)

T2

T1

T2

d)
b)

H. 1.9 Cac bien


dang c ban
e)

Hnh 1.8 Cac dang chu lc c ban

1.3.2 Bien dang cua phan to: Neu tng tng tach mot phan to hnh
hop t mot thanh chu lc th s bien dang cua no trong trng hp tong
quat co the phan tch ra hai thanh phan c ban:
Phan to tren H.1.9a dai dx ch thay oi chieu dai, khong thay oi goc.
Bien dang dai tuyet oi theo phng x :
Bien dang dai tng oi theo phng x :

dx.
x =

dx
dx

Phan to tren H.1.9b ch co thay oi goc, khong thay oi chieu dai


Bien dang goc hay goc trt, ky hieu la : o thay oi cua goc
vuong ban au

Chng 1: Khai niem c ban

http://www.ebook.edu.vn
5

GV: Le c Thanh

1.3.3 Chuyen v:
Khi vat the b bien dang, cac iem

P1

P3

trong vat the noi chung b thay oi v tr.


o chuyen di t v tr cu cua iem A

A+

sang v tr mi A c goi la chuyen v

A +

dai. Goc hp bi v tr cua mot oan


thang AC trc va trong khi bien dang

+C
+ C

P4

P2

AC cua vat the c goi la chuyen v

H. 1.10

goc ( H.1.10).
1.4 Cac gia thiet

Khi giai bai toan SBVL, ngi ta chap nhan mot so gia thiet nham n
gian hoa bai toan nhng co gang am bao s chnh xac can thiet phu hp
vi yeu cau thc te.
1.4.1 Gia thiet ve vat lieu
Vat lieu c coi la lien tuc, ong nhat, ang hng va an hoi
tuyen tnh.
Ta tng tng lay mot phan to bao quanh mot iem trong vat the.
Neu cho phan to be tuy y ma van cha vat lieu th ta noi vat lieu lien tuc
tai iem o.
Gia thiet ve s lien tuc cua vat lieu cho phep s dung cac phep tnh
cua toan giai tch nh gii han, vi phan, tch phan.... Trong thc te, ngay ca
vi vat lieu c coi la hoan hao nhat nh kim loai th cung co cau truc
khong lien tuc.
Lc

Vat lieu ong nhat : Tnh chat c hoc


tai moi iem trong vat the la nh nhau.
Vat lieu ang hng : Tnh chat c hoc
tai mot iem theo cac phng eu nh nhau.

Bien dang

H. 1.11 an hoi tuyen

Tnh chat an hoi cua vat the la kha


tnh
nang khoi phuc lai hnh dang ban au cua no
khi ngoai lc thoi tac dung. Neu quan he gia ngoai lc va bien dang la
bac nhat, th vat lieu c goi la an hoi tuyen tnh (H.1.11).
Gia thiet vat lieu an hoi tuyen tnh lam giam bt s phc tap cua bai
toan SBVL.
Chng 1: Khai niem c ban

http://www.ebook.edu.vn
6

GV: Le c Thanh

1.4.2 Gia thiet ve s o tnh


Khi tnh toan, ngi ta thay vat the thc bang s o tnh (H1.12).
q

a)

b)
H. 1.12 S o tnh

1.4.3 Gia thiet ve bien dang va chuyen v


Vat the co bien dang va chuyen v be so vi kch thc ban au cua
vat Co the khao sat vat the hoac cac bo phan cua no tren hnh dang
ban au ( tnh tren s o khong bien dang cua vat the).
Gia thiet nay xuat phat ieu kien bien dang va chuyen v ln nhat trong
vat the phai nam trong mot gii han tng oi nho.
He qua:
Khi vat the co chuyen v be va vat lieu an hoi tuyen tnh th co the ap
dung nguyen ly cong tac dung nh sau:
Mot ai lng do nhieu nguyen nhan ong thi gay ra se bang
tong ai lng o do tng nguyen nhan gay ra rieng le. (H.1.13)
P2
P1

P2

P1

H.1.13 Nguyen ly cong tac dung

Chuyen v tai au thanh do lc P1 va P2 gay ra co the phan tch nh


(P1 , P2 ) = 1 (P1 ) + 2 (P2 )
sau:
Nguyen ly cong tac dung bien bai toan phc tap thanh cac bai toan n
gian de giai quyet hn. V vay, thng c s dung trong SBVL.

Chng 1: Khai niem c ban

http://www.ebook.edu.vn
7

GV: Le c Thanh

Chng 2

LY THUYET NOI LC
2.1 KHAI NIEM VE NOI LC - PHNG PHAP KHAO SAT - NG SUAT
1- Khai niem ve noi lc:
Xet mot vat the chu tac dung cua ngoai lc va trang thai can bang
(H.2.1). Trc khi tac dung lc, gia cac phan t cua vat the luon co cac
lc tng tac gi cho vat the co hnh dang nhat nh. Di tac dung cua
ngoai lc, cac phan t cua vat the co the dch lai gan nhau hoac tach xa
nhau. Khi o, lc tng tac gia cac phan t cua vat the phai thay oi e
chong lai cac dch chuyen nay. S thay oi cua lc tng tac gia cac
phan t trong vat the c goi la noi lc.
Mot vat the khong chu tac ong nao t ben ngoai th c goi la vat
the trang thai t nhien va noi lc cua no c coi la bang khong.
2-Phng phap khao sat noi lc: Phng phap mat cat
Xet lai vat the can bang va 1 iem C trong vat the (H.2.1),.
Tng tng mot mat phang cat qua C va chia vat the thanh hai
phan A va B; hai phan nay se tac ong lan nhau bang he lc phan bo tren
dien tch mat tiep xuc theo nh luat lc va phan lc.
Neu tach rieng phan A th he lc tac ong t phan B vao no phai can
bang vi ngoai lc ban au (H.2.2).

P2

P1

P6

P1
A

P3

P5
P4

P2

A
P3

H.2.1 Vat the chu l c can ba ng

p
F

H.2.2 Noi l c tre n ma t ca t

Xet mot phan to dien tch F bao quanh iem khao sat C tren mat cat
co phng phap tuyen v. Goi

la vector noi lc tac dung tren F . Ta

nh ngha ng suat toan phan tai iem khao sat la:

p d p
=
F 0 F
dF
Th nguyen cua ng suat la [lc]/[chieu dai]2 (N/m2, N/cm2).
p = lim

Chng 2: Ly Thuyet Noi Lc

1
http://www.ebook.edu.vn

GV: Le c Thanh

ng suat toan phan p co the phan ra hai thanh


phan:
+ Thanh phan ng suat phap v co phng
Hnh 2.3 Cac thanh
phap tuyen cua mat phang
phan
+ Thanh phan ng suat tiep v nam trong mat
ng suat
phang ( H.2.3 ).
Cac ai lng nay lien he vi nhau theo bieu thc:
pv2 = v2 + v2
(2.1)

ng suat la mot ai lng c hoc ac trng cho mc o chu ng cua


vat lieu tai mot iem; ng suat vt qua mot gii han nao o th vat lieu b
pha hoai. Do o, viec xac nh ng suat la c s e anh gia o ben cua
vat lieu, va chnh la mot noi dung quan trong cua mon SBVL.
Tha nhan: ng suat phap v ch gay ra bien dang dai.
ng suat tiep v ch gay bien dang goc.

Chng 2: Ly Thuyet Noi Lc

2
http://www.ebook.edu.vn

GV: Le c Thanh

2.2 CAC THANH PHAN NOI LC - CACH XAC NH


1- Cac thanh phan noi lc:
Nh a biet, oi tng khao sat cua SBVL la nhng chi tiet dang thanh,
ac trng bi mat cat ngang (hay con goi la tiet dien) va truc thanh.

P2
P3

P1

P6

P1
A

P5
P4

P2

A
P3

Qy

P1

Qx

Nz

P2

Mz

Mx

A
P3

My

x
z

y
H.2.4 Cac tha nh phan noi l c

Goi hp lc cua cac noi lc phan bo tren mat cat ngang cua thanh la R.
R co iem at va phng chieu cha biet .
Lc R
co phng bat ky
Momen M

Di R ve trong tam O cua mat cat ngang

at mot he truc toa o Descartes vuong goc ngay tai trong tam mat cat
ngang, Oxyz, vi truc z trung phap tuyen cua mat cat, con hai truc x, y
nam trong mat cat ngang.
Khi o, co the phan tch R ra ba thanh phan theo ba truc:
+ Nz, theo phng truc z ( mat cat ngang) goi la lc doc
+ Qx theo phng truc x (nam trong mat cat ngang) goi la lc cat.
+ Qy theo phng truc y (nam trong mat cat ngang) goi la lc cat.
Momen M cung c phan ra ba thanh phan :
+ Momen Mx quay quanh truc x goi la momen uon .
+ Momen My quay quanh truc y goi la momen uon .
+ Momen Mz quay quanh truc z goi la momen xoan.
Sau thanh phan nay c goi la cac thanh phan noi lc tren mat cat
ngang (H.2.4)
.

Chng 2: Ly Thuyet Noi Lc

3
http://www.ebook.edu.vn

GV: Le c Thanh

2 Cach xac nh:


Sau thanh phan noi lc tren mot mat cat ngang c xac nh t
sau phng trnh can bang oc lap cua phan vat the c tach ra, tren
o co tac dung cua ngoai lc ban au PI va cac noi lc.
Cac phng trnh can bang hnh chieu cac lc tren cac truc toa o:
n

Z = 0 N z + Piz = 0 N z
i =1
n

(2.2)

Y = 0 Qy + Piy = 0 Qy
i =1
n

Z = 0 Qx + Pix = 0 Qx
i =1

trong o: Pix, Piy, Piz - la hnh chieu cua lc Pi xuong cac truc x, y, z.
Cac phng trnh can bang momen oi vi cac truc toa o ta co:
n

M / Ox M x + mx ( Pi ) = 0 M x
i =1
n

M / Oy M y + m y ( Pi ) = 0 M y

(2.3)

i =1
n

M / Oz M z + mz ( Pi ) = 0 M z
i =1

vi:mx(Pi), my(Pi), mz(Pi) - cac momen cua cac lc Pi oi vi cac truc x,y, z.
3-Lien he gia noi lc va ng suat:
Cac thanh phan noi lc lien he vi cac thanh phan ng suat nh sau:
- Lc doc la tong cac ng suat phap
- Lc cat la tong cac ng suat tiep cung phng vi no
- Momen uon la tong cac momen gay ra bi cac ng suat oi vi truc x
hoac y
- Momen xoan la tong cac momen cua cac ng suat tiep oi vi truc z

Chng 2: Ly Thuyet Noi Lc

4
http://www.ebook.edu.vn

GV: Le c Thanh

2-3 BAI TOAN PHANG:


Trng hp bai toan phang ( ngoai lc nam trong mot mat phang ( th
du mat phang yz)), ch co ba thanh phan noi lc nam trong mat phang yz :
Nz, Qy, Mx.
Qui c dau (H.2.5)
- Lc doc Nz > 0 khi gay keo
oan thanh ang xet (co chieu
hng ra ngoai mat cat)

P1
P2

A
P3

- Lc cat Qy > 0 khi lam quay


oan thanh ang xet theo chieu kim
ong ho.
- Momen uon Mx > 0
th di ( th y dng ).
Mx > 0

MX> 0

Nz > 0

MX> 0

Qy > 0

Nz > 0

Qy > 0

P4
P5

B
P6

Hnh 2.5: Chieu dng

cac thanh phan noi

khi cang

Mx > 0

Mx < 0

Mx < 0

Momen M x > 0 , Momen M x < 0

Cach xac nh:


Dung 3 phng trnh can bang tnh hoc khi xet can bang phan A) hay
phan B)
T phng trnh Z = 0 Nz
T phng trnh Y = 0 Qy

(2.4)

T phng trnh M/O = 0 Mx

Chng 2: Ly Thuyet Noi Lc

5
http://www.ebook.edu.vn

GV: Le c Thanh

Th du 2.1 Xac nh cac tr so noi lc tai mat cat 1-1 cua thanh AB, vi :
q = 10 kN/m; a = 1m; Mo = 2qa2. ( H.2.6)
P=

q
HA

1,5a
VA

2qa2

2qa
1,5a

VB

P=

A
VA

M=

2qa

H.
2.6

Giai.
Tnh phan lc: Giai phong cac lien ket va thay vao o bang cac phan lc
lien ket VA, HA, VB.
Viet cac phng trnh can bang tnh hoc khi xet can bang thanh AB
Z = 0 HA = 0
Y = 0 VA +VB - qa P = 0

M A = 0

a
qa + P x a - M 0 VB x 2a = 0
2

HA = 0;

VA =

11
1
qa = 27,5 kN ; VB = qa = 2,5 kN
4
4

Tnh noi lc: Mat cat 1-1 chia thanh lam hai phan.
Xet s can bang cua phan ben trai (H.2.6) :

= 0 N =0

1
= 0 VA qa P Q = 0 Q = qa = 2,5 kN
4
a 17
M O1 = 0 M = VA 1,5a qa a 2qa 2 = 8 qa 2 = 21,25 kNm

Neu xet can bang cua phan phai ta cung tm c cac ket qua nh tren.
Chng 2: Ly Thuyet Noi Lc

6
http://www.ebook.edu.vn

GV: Le c Thanh

2.4 BIEU O NOI LC ( BAI TOAN PHANG )


1. nh ngha: Thng cac noi lc tren cac mat cat ngang cua mot
thanh khong giong nhau.
Bieu o noi lc (BNL) la o th bieu dien s bien thien cua cac noi lc
theo v tr cua cac mat cat ngang. Hay goi la mat cat bien thien.
Nh vao BNL co the xac nh v tr mat cat co noi lc ln nhat va tr
so noi lc ay.
2. Cach ve BNL- Phng phap giai tch:
e ve bieu o noi lc ta tnh noi lc tren mat cat cat ngang mot v
tr bat ky co hoanh o z so vi mot goc hoanh o nao o ma ta chon trc.
Mat cat ngang chia thanh ra thanh 2 phan. Xet s can bang cua mot phan
(trai, hay phai) , viet bieu thc giai tch cua noi lc theo z..
Ve ng bieu dien tren he truc toa o co truc hoanh song song vi
truc thanh (con goi la ng chuan), tung o cua bieu o noi lc se c
dien ta bi cac oan thang vuong goc cac ng chuan.
Th du 2.2- Ve BNL cua dam mut tha (H.2.7)
Giai
Xet mat cat ngang 1-1 co hoanh o
z so vi goc A, ta co ( 0 z l )

va momen uon tai mat cat 1-1


phan phai cua thanh:

c xac nh t viec xet can bang

B
p

Z = 0 N = 0
Y = 0 Q P = 0 Q = P
M O = 0 M + P(l z ) = 0 M
y

P
K

Bieu thc giai tch cua lc cat

M Pl

= P (l z )

Cho z bien thien t 0 en l, ta se c


bieu o noi lc nh tren H.2.7.

Hnh 2.7

Qui c:+Bieu o lc cat Qy tung o dng ve pha tren truc hoanh.


+Bieu o momen uon Mx tung o dng ve pha di truc hoanh.

Chng 2: Ly Thuyet Noi Lc

7
http://www.ebook.edu.vn

GV: Le c Thanh

(Tung o cua bieu o momen luon ve pha th cang cua thanh).


Th du 2.3 Ve BNL cua dam n gian chu tai phan bo eu q (H.2.8a).
Giai

1
H= A
K
a
z
thay bang cac phan lc ( H.2.8a).
0
ql
1
l
) V 2
1 M
Z = 0 HA =0.
A
b
V
Do oi xng V A = VB = ql
z 1Q N
2
)
Qy y
Noi lc: Chon truc hoanh nh tren
ql
+ 2
2
H.2.8b. Xet mat cat ngang 1-1 tai K co c
ql
)
8
hoanh o la z, ( 0 z l ). Mat cat chia d
Mx
thanh lam hai phan.
H.2.8
)
Xet can bang cua phan ben trai AK
(H.2.8b)
T cac phng trnh can bang ta suy ra:
Phan lc: Bo cac lien ket tai A va B,

q
B

Z = 0 N z = 0

ql
l

qz = q ( z )
Y = 0 Q y =
2
2

ql
qz 2
qz
M
O
M
z

/
=
0

=
(l z )

x
1
2
2
2

Qy la ham bac nhat theo z, Mx la ham bac 2 theo z.


Cho z bien thien t 0 en l ta ve c cac bieu o noi lc (H2.8).
Cu the: +Khi z=0 Qy = ql/2 , Mx = 0
+Khi z=l Qy = -ql/2 , Mx = 0
+Tm Mx, cc tr bang cach cho ao ham dMx / dz =0,
l
ql
2 qz =0 z = 2
dMx / dz =0
ql 2
M
=
x,max

Qua cac BNL, ta nhan thay:


Lc cat Qy co gia tr ln nhat mat cat sat goi ta,
Momen uon Mx co gia tr cc ai gia dam.

Chng 2: Ly Thuyet Noi Lc

8
http://www.ebook.edu.vn

V ql2
=
B

ql
2

GV: Le c Thanh

Th du 2.4 Ve BNL cua dam n gian chu lc tap trung P ( H.2.9a) .


Giai
Phan lc: Cac thanh phan phan lc tai cac goi ta la:
H A = 0 ; VA =

Pb
l

VB =

Pa
l

Noi lc : V tai trong co phng vuong goc vi truc thanh nen lc doc
Nz tren moi mat cat ngang co tr so bang khong.
Phan oan thanh: V tnh lien tuc cua cac ham so giai tch bieu dien
cac noi lc nen phai tnh noi lc trong tng oan cua thanh; trong moi oan
phai khong co s thay oi ot ngot cua ngoai lc .
oan AC- Xet mat cat 1-1 tai iem K1 trong oan AC va cach goc A
mot oan z, ( 0 z a ).
Khao sat can bang cua phan ben trai ta c cac bieu thc giai tch cua
noi lc:
Pb P (l a)

Q y = VA = l =
l

P (l a)
Pb
M = V .z =
z
z=
A
x
l
l

(a)
P

oan CB- Xet mat cat 2-2 tai iem K2


Trong oan CB cach goc A mot oan z , ( a

z l ). Tnh noi lc tren mat cat 2-2 bang


cach xet phan ben phai (oan K2B). Ta
c:
Pa
Q y = VB =
l
M x = VB (l z) =

a)

(b)

T (a) va (b) de dang ve c cac bieu

b
)

)
e)

VA

l
Mx

1
z1

Qy
P
bl

(b)

VB

Mx

Qy

Pa
bl

VA

Pa
(l z)
l

b
2 K2

1 K1

l-z

c
VB )

Qy
Pa
l

Mx

H. 2.9

o noi lc nh H.2.9d,e.
Trng hp ac biet : Neu a=b= L/2, khi o momen cc ai xay ra tai gia
dam va co gia tr: Mmax = PL/4

Chng 2: Ly Thuyet Noi Lc

9
http://www.ebook.edu.vn

GV: Le c Thanh

Th du 2.5 Ve BNL cua dam n gian chu tac dung cua momen tap trung
Mo (H.2.10a.)
Giai
Phan lc: Xet can bang cua toan dam ABC cac phan lc lien ket tai
H A = 0 ; V A = VB =

A va B la:

Mo
l

, chieu phan lc nh H.2.10a.

Noi lc:
a

oan AC: Dung mat cat 1-1 cach goc A


mot oan z1 ;(0 z1 a ).Xet can bang cua
oan AK1 ben trai mat cat K1 cac noi lc
nh sau

Mo

Q y = V A = l

M = V z = M o z
1
A 1
x
l
1

(c)

K1

z1

oan CB: Dung mat cat 2-2 trong oan


CB cach goc A mot oan z2 vi (a z2 l ) .
Xet can bang cua phan ben phai K2B cac
bieu thc noi lc tren mat cat 2-2 la:

z2

VA
b)

2 K2

z1

VA

Mo

Qy 2 = VB = l

M = V (l z ) = M o (l z )
B
2
2
x 2
l

a)

Mo

1 K1

M x2

M x1

l z2

VB

d) Qy

2
Q y1

Q y2

l z2

VB)
z

Mo / l
Mo
a
l

e) M
x

Mo
(l - a)
l

H. 2.10

(d)

Mo

Trng hp ac biet: Momen tap trung Mo


at tai mat cat sat goi ta A (H.2.11).
Qy va Mx se c xac nh bi (d) ng vi

a)

BNL c ve t cac bieu thc (c), (d) cua noi


lc trong hai oan (H.2.10d-e).

l
Mo
VA =
l

b Q

Mo/ l

c M
)

Mo

H. 2.11

a = 0. BNL ve nh H.2.11

Chng 2: Ly Thuyet Noi Lc

10
http://www.ebook.edu.vn

VB =

Mo
l

GV: Le c Thanh

Cac nhan xet :


- Ni nao co lc tap trung, bieu o lc cat ni o co bc nhay. Tr so
cua bc nhay bang tr so lc tap trung. Chieu bc nhay theo chieu lc
tap trung neu ta ve t trai sang phai
- Ni nao co momen tap trung, bieu o momen uon ni o co bc
nhay. Tr so cua bc nhay bang tr so momen tap trung. Chieu bc nhay
theo chieu momen tap trung neu ta ve t trai sang phai
Kiem chng cac nhan xet :

P0
1
P0
Q1

M0

a)

M2

M1

M0

Q2
1

H. 2.12

z
b)

Khao sat oan z bao quanh mot iem K co tac dung lc tap trung P0 ,
momen tap trung M0 ( H.2.12b).
Viet cac phng trnh can bang
Y = 0 Q1 + P0 Q2 = 0 Q2 Q1 = P0

(i)

M/K = 0 M1 +M0 - M2 + Q1 z - Q2 z =0
2

Bo qua vo cung be bac mot Q1 z , Q2 z , M2 - M1 = M0


2

(ii)

Bieu thc (i) a kiem chng nhan xet ve bc nhay cua bieu o lc cat.
Bieu thc (ii) a kiem chng nhan xet ve bc nhay cua bieu o momen.

Chng 2: Ly Thuyet Noi Lc

11
http://www.ebook.edu.vn

GV: Le c Thanh

2.4. LIEN HE VI PHAN GIA NOI LC VA TAI TRONG PHAN BO TRONG


THANH THANG
Xet mot thanh chu tai trong bat ky (H.2.13a). Tai trong tac dung tren
thanh nay la lc phan bo theo chieu dai co cng o q(z) co chieu dng
hng len (H.2.13b).
q(z)
q(z)

Mo

Qy

M+
x xdM

Mx

dz

Qyy+ dQ
1

H. 2.13

a)

dz
b)

Khao sat oan thanh vi phan dz, gii han bi hai mat cat 1-1 va 2-2
(H.2.13b). Noi lc tren mat cat 1-1 la Qy va Mx. Noi lc tren mat cat 2-2 so
vi 1-1 a thay oi mot lng vi phan va tr thanh Qy + dQy; Mx + dMx . V
dz la rat be nen co the xem tai trong la phan bo eu tren oan dz.
Viet cac phng trnh can bang:
1-Tong hnh chieu cac lc theo phng ng
Y = 0 Qy + q(z)dz (Qy + dQy) = 0

q( z) =

dQ y

(2.4)

dz

ao ham cua lc cat bang cng o cua lc phan bo vuong goc vi truc
thanh.
2- Tong momen cua cac lc oi vi trong tam mat cat 2-2 ta c:
M/o2 = 0

Q y dz + q( z) dz

Bo qua lng vo cung be bac hai

q( z)

dz
+ M x (M x + dM x ) = 0
2

dz 2
2

dM x
= Qy
dz

(2.5)

ao ham cua momen uon tai mot mat cat bang lc cat tai mat cat o
T (2.4) va (2.5)

d2 M x
dz 2

= q( z)

(2.6)

ngha la: ao ham bac hai cua momen uon tai mot iem chnh la bang
cng o cua tai trong phan bo tai iem o.
Chng 2: Ly Thuyet Noi Lc

12
http://www.ebook.edu.vn

GV: Le c Thanh

Th du 2.6 Ve BNL cho dam


n gian AB chu tac dung cua tai
phan bo bac nhat nh H.2.14.

q(z)

A
a)

Phan lc: Giai phong lien


ket, at cac phan lc tng ng
cac goi ta, xet can bang cua toan
thanh,

b)

X =0 HA = 0,

Qy

qol
3

+
l 3

Mmaz

1
1
l
B = 0 VAl = qol VA = qo l
2
3
6
1
Y = 0 VB = 3 qol

VB = 1 qo l
3

Mx

VAo= 1 q 0 l
6

qol

VB

VA

Giai

qo

H.2.14

Noi lc: Cng o cua lc

phan bo mat cat 1-1 cach goc A mot oan z cho bi: q(z)= q0 z
l

Dung mat cat 1-1 va xet s can bang cua phan ben trai (H.2.14b).
Y = 0

Qy = VA q( z)

M/o1 = 0

Mx =

q l q z2
z
= o o
2
6
2l

(e)

ql
q z3
qo l
z z
= o z o
z q( z)
6
2 3
6
6l

(g)

T (e) va (g) ta ve c bieu o lc cat va momen cho dam a cho.


Cac bieu o nay co tnh chat nh sau:
Bieu o lc cat Qy co dang bac 2. Tai v tr z = 0, q(z) = 0 nen ay
bieu o Qy at cc tr: (Qy)z = 0 = Qmax =

qo l 6

Bieu o momen uon Mx co dang bac 3. Tai v tr

z =l

3;

Qy = 0. Vay tai

ay Mx at cc tr:
(M x )

z=

= M max =

Chng 2: Ly Thuyet Noi Lc

qo l 2

9 3

13
http://www.ebook.edu.vn

GV: Le c Thanh

Th du 2.7 Ve BNL cho dam chu lc tong quat (H.2.15)


Giai
Phan lc: Giai phong lien ket, xet can bang
toan thanh, suy ra phan lc lien ket tai A va
C la:

q
A

HA = 0 , VA = 2qa; VC = 2qa

P = 2qa

1
VA = 2qa
a

Q1 = 2qa qz

qz2
M1 = 2qaz
2

Q1

VA =

Mo
VA

q
a

3 q
2
a

2qa

* oan BC: Mat cat 2-2, goc A (a z 2a)


va xet can bang phan trai:
Q2 = qa

3 2
M 2 = qaz + 2 qa

Mx

M1

Qy

q
a2

a+
q
a2

H. 2.15

M2
Q2

* oan CD: Mat cat 3-3, goc A, (2a z 3a) xet can bang phan phai:
Q3 = q(3a z)

(3a z) 2
M3 = q
2

Q3
M3

q
3a z

(2a z 3a)

Bieu o momen va lc cat ve nh H.2.15.

Chng 2: Ly Thuyet Noi Lc

VC= 2qa
a

Noi lc:
* oan AB: Mat cat 1-1, goc A (0 z a),
xet can bang phan trai

Mo= qa
2
B
2 C

14
http://www.ebook.edu.vn

GV: Le c Thanh

Th du 2.8 Ve bieu o noi lc trong khung chu tai trong nh tren H.2.16.
q
2

qa
B

2
K2

z2

K1

K3

3 a

5
qa
2

VD

Hnh 2.15

+
2qa

D
a

5
q
a) 2
a

3
q
2
a

HA
VA

5
q
2
a

z3

z1

qa

P = qa

5
q
2
a

b
q

B
q

a
5
q
2
a

M
q

parabol

e
c
)

H..16

5)
q
2
a

5
q
2
a
d

3
q
2
a

q
5 a2
qa
2

qa

5
q
2
a

Giai
Tnh phan lc lien ket
Xet s can bang cua toan khung di tac dung cua tai trong ngoai va
cac phan lc lien ket ta suy ra:
Ngang = 0 HA = 0

MD = 0

Va a + qa

5
a
+ qa 2 + qa a = 0 VA = qa
2
2

ng = 0 VA + VD= 0 VD = +

5
qa ( ung chieu a chon )
2

Vay chieu that cua VA ngc vi chieu a chon

Chng 2: Ly Thuyet Noi Lc

15
http://www.ebook.edu.vn

GV: Le c Thanh

Ve bieu o noi lc
oan AB: dung mat cat 1-1 va xet can bang oan AK1 ta c:

5
qa
N1 =
2

Q1 = 2 qa qz1

2
M = 2 qaz qz1
1
1
2

M1

N1
Q1
K1
z1

(0 z1 a)
2q
a

q
a
oan BC: dung mat cat 2-2 va xet can bang oan ABK2 ta c:
q

M2

a
B

K2
z2

N2

N 2 = qa

Q2 = qa
2

5 2 5

M2 = 2 qa 2 qaz2

Q2

2q
a

A
5 q
2
a

(0 z2 a)

oan CD: dung mat cat 3-3 va xet can bang DK3
N3

N 3 = 2 qa

Q3 = 0
M = 0
3

M3
K3 Q3

(0 z3 a)

Kiem tra s can bang nut

Z3
D
VD = 5 q
2
a

oi vi khung, co the kiem tra ket qua bang viec xet can bang cac nut.
Neu tach nut ra khoi he th ta phai at vao nut cac ngoai lc tap trung
(neu co) va cac noi lc tai cac mat cat, gia tr cua chung c lay t bieu
o va ve.
Sau khi at cac lc tren, neu tnh ung cac noi lc cac nut th nut se
can bang, ngha la cac phng trnh can bang c thoa man. Ngc lai,
neu cac phng trnh khong thoa man th cac noi lc tnh sai.

Chng 2: Ly Thuyet Noi Lc

16
http://www.ebook.edu.vn

GV: Le c Thanh

Cu the oi vi khung ang xet, ta tach nut B va at vao o momen tap


trung qa2 va cac thanh phan noi lc tren cac oan thanh ngang va ng
nh H.2.16d:
- Tai mat cat tren thanh ngang co lc doc +qa hng ra ngoai mat cat,
lc cat

5 qa 2 2

co chieu hng len va momen

5 qa 2 2

- Tai mat cat tren thanh ng co lc doc

gay cang th di.

+ 5qa 2

hng ra ngoai mat cat

(hng xuong) lc cat +qa hng t phai sang trai va momen

3qa 2 2

gay ra

cang th trong khung nen chieu quay co mui ten hng ra ngoai.
Ta de dang thay cac phng trnh can bang thoa man:
X = 0 ; Y = 0 ; M/B = 0
Tng t, tach nut C va at vao o lc tap trung qa hng t trai sang
phai va cac thanh phan noi lc tren cac oan thanh ngang va ng nh
H.2.16d.
- Tai mat cat tren thanh ngang co lc doc +qa hng ra ngoai mat cat,
lc cat

5qa 2

co khuynh hng lam quay phan oan thanh ang xet ngc

chieu kim ong ho nen co chieu hng xuong, con momen th bang khong.
- Tai mat cat tren thanh thang ng ton tai lc doc

5qa 2

co chieu

hung vao mat cat (hng len) va khong co lc cat cung nh momen.
Ta de dang thay rang cac phng trnh can bang c thoa man:
5

X = qa + qa = 0 ; Y = 2 qa + 2 qa = 0 ; M B = 0
Vay cac nut B va C eu can bang ngha la cac he noi lc tai cac
nut ung.

Th du 2.9 Ve BNL trong thanh cong (H.2.17)


Chng 2: Ly Thuyet Noi Lc

17
http://www.ebook.edu.vn

GV: Le c Thanh

Giai
Cat thanh tai tiet dien
1-1, xac nh bi goc (0

2P

90o), xet can bang

2P

a)

b
)

PR
1,7PR

45

2P

Phng trnh can


bang hnh chieu cac lc

2.12P

c nh H.2.17b.

Q max =2,236P

2P

0,7P

M
N

dung cua cac ngoai lc


at theo chieu dng quy

1Q

cua phan tren di tac


va cac thanh phan noi lc

theo phng phap tuyen vi mat cat cho:

3PR

e)

H. 2.17

2Psin Pcos

P(2sin cos)
(a)
Phng trnh can bang hnh chieu cac lc theo phng ng knh
Q = 2Pcos + Psin = P(2cos + sin)
(b)
Phng trnh can bang cua cac momen cac lc oi vi trong tam mat
cat dan en:

cho:

(c)
M = 2PRsin PR(1 cos) = PR(2sin + 1 cos)
Cho mot vai tr so ac biet va tnh cac tr so noi lc tng ng, ta ve
c bieu o.
dQy

Lc cat at cc tr khi

= 0 , ngha la khi:

-2sin + cos = 0 tg = 0,5 = o = 26o56


sino = 0,4472 ; coso = 0,8944
Ta co bang noi lc sau:

45o

900

0,7 P

2P

2P
0

2,236 P

2,12 P

+P

- PR

-1,7 PR

-3PR

Khi ve can chu y at cac tung o theo phng vuong goc vi truc thanh,
tc la theo phng ban knh nh tren H.2.17c,d,e.
Chng 2: Ly Thuyet Noi Lc

18
http://www.ebook.edu.vn

GV: Le c Thanh

2.5 CACH VE BIEU O NHANH


2.5.1 Phng phap ve tng iem
Da tren cac lien he vi phan, ta nh dang cac BNL tuy theo dang tai
trong a cho va t o ta xac nh so iem can thiet e ve bieu o.
Tren 1 oan thanh
+ q =0 Q = hang so, M = bac nhat.
+ q = hang Q = bac nhat, M = bac hai.
.
+ Neu bieu o co dang hang so , ch can xac nh mot iem bat ky.
+ Neu bieu o co dang bac nhat , can tnh noi lc tai hai iem au va cuoi
oan thanh.
+ Neu bieu o co dang bac hai tr len th can ba gia tr tai iem au, iem
cuoi va tai ni co cc tr, neu khong co cc tr th can biet chieu loi lom cua
bieu o theo dau cua ao ham bac hai. oan thanh co lc phan bo q
hng xuong se am, nen be lom cua bieu o momen hng len. Ngc lai,
neu q hng len se dng nen be lom cua bieu o momen hng xuong.
Tom lai, ng cong momen hng lay lc phan bo q.

Th du 2.10: Ve BNL trong dam cho tren H.2.18 (phng phap ve iem)

Chng 2: Ly Thuyet Noi Lc

19
http://www.ebook.edu.vn

GV: Le c Thanh

Giai.
Phan lc lien ket

B = 0 qa 2 + 2qa 2 + 2qa 2 VC 2a = 0 VC =

= 0 VB =

5
qa
2

Noi lc
oan AB: q=0 Qy = hang so,
Mx = bac nhat.
Trong trng hp nay Qy la hang
so bang khong v QA(AB) = 0.
Mx trong oan nay se la hang so
(AB)

(BA)

= MB
= Mo = -qa
MA
oan BD: q= hang Qy = bac 1,
Mx = bac 2.
Tai B:
Tai D:

3
qa
2

5
( BD )
= + qa
QB
2

M ( BD ) = M = qa 2
o
B
5
3
( BD)
=
qa qa =
qa
QD
2
2

2
M ( BD) = 3 qa 2 qa = qa 2
D
2
2

Mo = qa2

P=

2qa
a)

A
a

b
)

5 q
2
a

Qy

q
c
)

Mx

D
a

a
VB = 5 q
2
a
+
1 q
2
a

VC =
3 q
2
a

3 q
2
a

3 q
2
a

a
H. 2.18

Bieu o Qy trong oan nay khong co v tr nao =0 bieu o Mx khong


co cc tr.
Ch can noi hai gia tr momen tai B va D bang ng cong bac hai co
be lom sao cho hng lay lc q.
oan DC: q= hang Qy = bac 1, Mx = bac 2.
Tai D:
Tai C:

( DC )
QD
=

1
qa
2

M D( DC ) =

QC = VC =

3
qa ;
2

3 2 qa 2
= qa 2
qa
2
2
MC = 0

Bieu o Qy trong oan nay khong co v tr nao =0 bieu o Mx khong


co cc tr.
Ch can noi hai gia tr momen tai D va C bang ng cong bac hai co
be lom sao cho hng lay lc q.
Cac bieu o lc cat Qy va momen Mx lan lt c ve tren H.2.18b,c.

Chng 2: Ly Thuyet Noi Lc

20
http://www.ebook.edu.vn

GV: Le c Thanh

2.5.2 Cach ap dung nguyen ly cong tac dung


Khi thanh chu tac dung nhieu loai tai trong, ta co the ve bieu o noi
lc trong thanh do tng tai trong rieng le gay ra roi cong ai so lai e c
ket qua cuoi cung.
Th du 10. Ve bieu o mo men trong dam nh H.2.18a bang cach cong
bieu o.
q

a)

P = 2qa

b) Pa
c

2
) qa /
2
d

) Pa +
qa2/
2
H.2.18

Giai. Tai trong tren thanh c chia thanh hai trng hp c ban:
+ Hnh 2.18b bieu dien mo men do lc tap trung P gay ra
+ Hnh 2.18c bieu dien mo men do lc phan bo eu q gay ra
Hnh 2.18dbieu dien mo men tong hp can tm, cac tung o bang tong
ai so cac tung o tai cac tiet dien tng ng tren H.2.18b,c

Bang tom tat dam console , dam n gian, dam au tha


P
Chng 2: Ly Thuyet Noi Lc B

21
http://www.ebook.edu.vn

GV: Le c Thanh

BAI TAP CHNG 2


2.1. Ve bieu o noi lc cua cac dam cho tren H.2.1.
M = 10 kNm

P = 5 kN

Chng 2: Ly Thuyet Noi Lc

q = 5 kN/m

P = 2qa

22
http://www.ebook.edu.vn

GV: Le c Thanh

M = qa
1

a)
P = 6qa

P = 4 kN

2a
c

P = qa

2a

M = 15 kNm

P = qa

3a
e)

b
q = 2 kN/m
)

)
M = qa

M = 16 kNm

1
m

)
q = 10 kN/m
P = 20 kN

1m

H.2.1

f
)

2.2. Khong can tnh ra phan lc, ve BNL cua cac dam cho tren H.2.2.
P = 2qa

2a

P = qa

3a

2
M = 1 qa
2

4a

a)

b
)

H.2.2

2.3. Ve bieu o noi lc nh tren H.2.3.


P = 8 kN

qo = 2 kN/m

D
B

a)

a
b

H.2.3

2.4. Ve bieu o noi lc cua dam tnh nh nh tren H.2.4.


M = qa

3a

M = qa

3a

H. 2.4

Chng 2: Ly Thuyet Noi Lc

23
http://www.ebook.edu.vn

GV: Le c Thanh

2.5. Ve bieu o noi lc cho he khung sau (H.2.5).


q

q
P = ql

2q

0,75a

0,75a

l
a)

H.2.5

2.6. Ve bieu o lc doc, momen uon, momen xoan cho thanh khong
gian (H.2.6).

P = qa

P = qa
q

P = qa

a)

Chng 2: Ly Thuyet Noi Lc

H. 2.6

2P

b
)

24
http://www.ebook.edu.vn

Chng 3. KEO - NEN UNG TAM


3.1 KHAI NIEM
nh ngha: Thanh c goi la chu keo hay
nen ung tam khi tren moi mat cat ngang cua

Nz
Y

thanh ch co mot thanh phan noi lc la lc doc Nz.


Nz > 0 khi hng ra ngoai mat cat- Keo

y
H. 3.1

Nz < 0 khi hng vao trong mat cat- Nen

ay la trng hp chu lc n gian nhat. Ta gap trng hp nay khi


thanh chu 2 lc bang nhau va trai chieu hai au doc truc thanh .
Thanh chu keo ung tam (H.3.2a) hay chu nen ung tam (H.3.2b).

a)

b)

H. 3.2 nh ngha thanh chu keo nen ung


t

Thc te : co the gap cac cau kien chu keo hay nen ung tam nh:
day cap trong can cau (H.3.3a), ong khoi (H.3.3b), cac thanh trong dan
(H.3.3c).

Q
a)

b)

c)

H. 3.3 Mot so cau kien chu keo nen ung tam

http://www.ebook.edu.vn

3.2. NG SUAT TREN MAT CAT NGANG

Xet thanh thang chu keo (nen) ung tam (H.3.3a) cac mat cat ngang CC
va DD trc khi thanh chu lc cach nhau oan dz va vuong goc truc thanh.
Cac th doc trong oan CD (nh la GH) bang nhau (H.3.3b).
Khi thanh chu keo (nen), noi lc tren mat cat ngang DD hay bat ky mat
cat ngang khac la Nz = P (H.3.3c) thanh se dabn ra, mat cat DD di chuyen doc
truc thanh z so vi mat cat CC mot oan be dz (H.3.3b).
P

Nz

Nz

c)

a)
D

D
H

x
D
C

dF

Nz

z
dz
b)

dz
d)

Ta thay bien dang cac th doc nh GH eu bang HH va khong oi, mat


cat ngang trong suot qua trnh bien dang van phang va vuong goc vi truc
thanh, ieu nay cho thay cac iem tren mat cat ngang ch co ng suat phap z
khong oi (H.3.3d).

dz
Ta co: z dF = N z v ( z =
z = z )
dz
E
F
Nen z = const ta c: z F = N z
N
z = z
(3.1)
hay:
F
vi: F- dien tch mat cat ngang cua thanh.

3.3. BIEN DANG CUA THANH CHU KEO (NEN) UNG TAM
1- Bien dang doc
http://www.ebook.edu.vn

Bien dang doc truc z cua oan dai dz chnh la dz (H.3.3b).


dz
Nh vay bien dang dai tng oi cua oan dz la: z =
dz
Theo nh luat Hooke ta co:

(a)

(b)
E
trong o: E - la hang so ty le, c goi la mo un an hoi khi keo (nen), no

lc
, n v N/m2 , xac
phu thuoc vao vat lieu va co th nguyen
2
(chieu dai)
nh t th nghiem .
Bang 3.1 cho tr so E cua mot so vat lieu.

z =

E (kN/cm2)

Vat lieu

Thep (0,15 0,20)%C

2 x 104

0,25 0,33

Thep lo xo

2,2 x 104

0,25 0,33

Thep niken

1,9 x 104

0,25 0,33

Gang xam

1,15 x 104

0,23 0,27

ong

1,2 x 104

0,31 0,34

ong thau

(1,0 1,2)104

0,31 0,34

Nhom

(0,7 0,8)104

0,32 0,36

Go doc th

(0,08 0,12)104

Cao su

0,8

0,47

T
T (a) tnh dz, the (b) vao, ta c bien dang dai doc truc cua oan dz la:

dz = z dz =

z
E

dz =

Nz
dz
EF

(c)

Suy ra bien dang dai (dan khi thanh keo, co khi thanh nen) cua oan thanh
dai L:
L = dz =
L

Nz
dz
EF

(3.2)

Neu E, Fla hang so va Nz cung khong oi tren chieu dai L cua thanh, ta se
c:
L =

Nz
N L
dz = z

EF L
EF

(3.3)

http://www.ebook.edu.vn

Neu thanh gom nhieu oan chieu dai Li va tren moi oan Nz, E, A khong oi
th:
L = L i =

N zi L i
E i Fi

(3.3)

Tch so EF goi la o cng khi chu keo hay nen ung tam cua thanh.
2- Bien dang ngang
Theo phng ngang thanh cung co bien dang, ta a chon z la truc thanh,
x, y la cac phng vuong goc vi z (H.3.3d). Neu ta goi x va y la bien dang dai
tng oi theo hai phng x va y, th ta co quan he sau:
x = y = z
(3.4)
trong o: - he so Poisson, la hang so vat lieu
Dau () trong bieu thc ch rang bien dang theo phng doc va ngang ngc
nhau.
Th du 3.1. Ve bieu o doc Nz tnh ng suat va bien dang dai toan phan cua

thanh tren H.3.4a cho biet E = 2.104 kN/cm2; F1 = 10 cm2; F2 = 20 cm2.

10 kN
F2

30 cm

IV

30 cm

III

50 cm

II

10 kN
20 kN

P2=40k
N

50 cm

F1

30 kN
P1=30kN
a)

H.3.4

Nz
b)

Giai. Dung phng phap mat cat ta de dang ve c bieu o Nz (H.3.4b)


T o ta tm c ng suat tren mat cat ngang moi oan la:

http://www.ebook.edu.vn

I =

III

N zI 30
=
= 3 kN/cm 2 ,
F1 10

II =

N zIII 10
=
=
= 0,5 kN/cm 2 ,
F2
20

IV

N zII 10
=
= 1 kN/cm 2
F1
10

N zIV 10
=
=
= 0,5 kN/cm 2
F2
20

e xac nh bien dang doc toan phan chnh la bien dang dai tuyet oi cua
thanh ta s dung cong thc (3.3) ap dung cho bon oan cua thanh.
10 30
30 50
10 50
10 30
L =
+
+
= 0,005 cm
+
4
4
4
2 10 10 2 10 10 2 10 20 2 10 4 20
Bien dang doc mang dau + ngha la thanh b dai ra.
Ta

co

the

tnh

bien

dang

bang

phng

phap

cong

tac

dung.

20x30
30 100
+ 30x60
- 40x60
40x50
L=
= 0,005cm
+
+
+
+
4
4
4
4
2 10 10
2 10 20
2 10 10
2 10 20
2x10 4 x20

3.4. AC TRNG C HOC CUA VAT LIEU


1. Khai niem
Van e cua chung ta la can phai so sanh o ben, o cng cua vat lieu khi
chu lc vi ng suat bien dang cua vat lieu cung loai a biet. Ta can th
nghiem keo, nen e tm hieu tnh chat chu lc va qua trnh bien dang t luc
bat au chu lc en luc pha hong cua cac loai vat lieu khac nhau.
Ngi ta phan vat lieu thanh hai loai c ban: Vat lieu deo, vat lieu don.
Nh vay co bon th nghiem c ban sau:
d0

2. Th nghiem keo vat lieu deo (thep)


1- Mau th nghiem
Theo tieu chuan TCVN 197 - 85
L
(H.3.5)
H.3.5
Chieu dai Lo th nghiem la oan thanh
ng knh do, dien tch Fo
2- Th nghiem
Tang lc keo t 0 en khi mau t, vi bo phan ve bieu o cua may keo,
ta nhan c o th quan he gia lc keo P va bien dang dai L cua mau nh
H.3.6. Ngoai ra sau khi mau b t ta chap mau lai, mau se co hnh dang nh
H.3.7.
3- Phan tch ket qua
Qua trnh chu lc cua vat lieu co the chia lam ba giai oan.
0

OA: an hoi, P va L bac nhat, Lc ln nhat la lc t le Ptl. tl =

Ptl
Fo

(3.5)

http://www.ebook.edu.vn

AD: giai oan chay, lc keo khong tang nhng bien dang tang lien tuc. Lc
keo tng ng la lc chay Pch va ta co gii han chay. ch =

Pch
Fo

(3.6)

DBC: giai oan cung co (tai ben), tng quan gia lc P va bien dang L
la ng cong. Lc ln nhat la lc ben PB va ta co gii han ben. b =

Pb
Fo

(3.7)
P

PB

d 1, A 1

C
D

Pch
Ptl

A
L1

H.3.7
L

H.3.6

Neu chieu dai mau sau khi t (H.3.7) la L1 va dien tch mat cat ngang ni
t la A1 th ta co cac nh ngha ac trng cho tnh deo cua vat lieu nh sau:
L L1
100% (3.8)
Bien dang dai tng oi (tnh bang phan tram): = 0
Lo
o that ty oi (tnh bang phan tram): =

Fo F1
100 %
Fo

4- Bieu o - (bieu o qui c)


T bieu o P-L ta de dang suy ra bieu o tng
quan gia ng suat z = P Fo va bien dang dai tng
oi z = L Lo .
Bieu o nay co hnh dang giong nh bieu o P - L
(H.3.8). Tren bieu o ch ro tl , ch , b va ca mo un
an hoi:

E=

= tan

b
ch
tl

D
A

Neu ke en s bien oi dien tch mat cat ngang ta


se co bieu o tng quan gia z va ng suat
thc (ng net t).
3. Th nghiem keo vat lieu don
Bieu o keo vat lieu don co dang ng
cong (H.3.9). Vat lieu khong co gii han ty le
va gii han chay ma ch co gii han ben.

(3.9)

H.3.8

P
Pb
Ptl

ng cong thc

ng qui c

http://www.ebook.edu.vn
H.3.9

b =

Pb
Fo

(3-10)

Tuy vay ngi ta cung qui c mot gii han an hoi nao o va xem o th
quan he lc keo va bien dang la ng thang (ng qui c).
P

4. Nen vat lieu deo


Bieu o nen vat lieu
deo nh H.3.10a. Ta ch
xac nh c gii han ty
P
le va gii han chay, ma P
khong xac nh c gii
han ben do s phnh ngang
cua mau lam cho dien tch O
a)
mat cat ngang mau lien
tuc tang len. Sau th
nghiem mau co dang hnh trong (H.3.10c).

b)

ch
tl

L
c)

H.3.10

d)

5. Nen vat lieu don. ng cong tng t bieu o keo vat lieu don. Pb.
Nghien cu cac th nghiem keo va nen cac vat lieu deo va don, ngi ta
thay rang: gii han chay cua vat lieu deo khi keo va nen nh nhau, con oi vi
vat lieu don gii han ben khi keo be hn nhieu so vi gii han ben khi nen.
3.6. THE NANG BIEN DANG AN HOI (TNBDH)
1- Khai niem
Xet thanh chu keo lam viec trong giai oan an hoi (H.3.13a). Lc tang
dan t 0 en gia tr P, thanh dan ra t t en gia tr L. Bo lc, thanh ve v tr
ban au.
Ngi ta noi cong cua W cua ngoai lc phat sinh trong qua trnh di
chuyen a chuyen hoa thanh the nang bien dang an hoi U tch luy trong
thanh va chnh the nang nay lam cho thanh an hoi sau khi khong tac dung
lc.
P
2- Tnh the nang bien
dang an hoi
A
P + dP
P va L bieu dien
P
L
nh H.3.13b. Cong cua
lc P tren chuyen di
L.
L

O
P

a)

L
dL

b)
http://www.ebook.edu.vn

H.3.13

dW = (P + dP)dL = PdL + dPdL= PdL


Suy ra cong cua lc keo P tang t 0 en P c bieu th bang dien tch tam
giac OAC.
PL
W=
2
PL P 2 L
Cong nay bien thanh TNBD H U:
U=W=
=
(3.11)
2 EF
2
Goi u la TNBDH rieng (the nang tch luy trong mot n v the tch), ta
co:
U z2 z z
=
u= =
(3.12)
V 2E
2
Xet oan thanh co chieu dai dz co noi lc Nz (H.3.14): dU =

N z2 dz
2 EF

Suy ra the nang bien dang an hoi cua oan thanh dai L, co noi lc Nz la:

N z2 dz
U = L dU = L
2 EF
N z2 L
Nz
Khi trong oan thanh
khong oi ta co: U =
(3.13)
EF
2 EF
N zi2 L i
Vi nhieu oan dai Li ta se co: U = Ui =
(3.13)
2 E i Fi

Nz

dz

The nang bien dang an hoi thng dung e tnh


chuyen v cua he thanh.
Nz

V du 3.2. Xac nh chuyen v ng cua iem at lc. Cho


E = 20000 kN/cm ; (H.3.15a). Cho L = 200 cm; P = 300 (KN); = 30o ; F= 10
L
L
cm2
H.3.14

Giai
B
- Xac nh noi lc
Tach mat A (H.3.15b).
Dung hai phng trnh hnh chieu:
X = 0: NAB = NAC = N
Y = 0: 2Ncos = P
P
suy ra:
N=
2 cos

a)

C
NAB

NAC

AC

AB

b)

H. 3.15

http://www.ebook.edu.vn

- Chuyen v ng cua iem A


a) Phng phap dung cach tnh theo bien dang hnh hoc.
Goi AB, AC cac bien dang cua oan AB, AC (H.3.15a).
T I, K ke hai ng vuong goc vi AB va AC, chung cat nhau A, AA
chnh la o di chuyen cua iem A.
Trng hp he thanh tren v NAB = NAC nen AB = AC va A nam tren
ng thang ng ke t A, hay AA chnh la chuyen v can tm.
Xet tam giac AIA ta co:

AI
AAcos = AI
hay:
AA =
= AB
cos
cos
AA =

N AB L AB
PL
=
(EF )AB cos 2EF cos 2

Vi P = 300 kN, E = 20000 kN/cm2, A = 10 cm2, = 300 ta c: AA = 0,4 cm


b) Phng phap dung the nang bien dang an hoi
Ta co: W = U
(*)
Cong ngoai lc:
W = 1 P.AA
2

The nang bien dang an hoi cua he:U =

2
N2 L
N AB
L AB
N 2L
+ AC AC = 2
2( EF ) AB
2( EF ) AC
2 EF

The vao (*) ta c:

N 2L
2 EF

suy ra:

AA =

1
2
2
P

P.AA = 2

PL
N 2L
=
= 0,4 cm
EF
2 EF cos 2

3.7. NG SUAT CHO PHEP - HE SO AN TOAN - BA BAI TOAN C BAN


Ta goi ng suat nguy hiem, ky hieu o , la tr so ng suat ma ng vi no
vat lieu c xem la b pha hoai. oi vi vat lieu deo o = ch , oi vi vat lieu
don o = b .
Nhng khi che tao, vat lieu thng khong ong chat hoan toan, va trong qua
trnh s dung tai trong tac dung co the vt qua tai trong thiet ke, ieu kien
lam viec cua ket cau hay chi tiet cha c xem xet ay u, cac gia thiet khi
tnh toan cha ung vi s lam viec cua ket cau. V the ta khong tnh toan
theo o . Chung ta phai chon mot he so an toan n ln hn 1 e xac nh ng
suat cho phep.

[ ] =

(3.15)
n
Va dung tr so [ ] e tnh toan.
He so an toan do nha nc hay hoi ong ky thuat cua nha may qui nh.
http://www.ebook.edu.vn

e chon he so an toan c chnh xac, nhieu khi ngi ta phai chon nhieu he
so theo rieng tng nguyen nhan dan en s khong an toan cua cong trnh hay
chi tiet may, co the ke en:
- He so ke en o ong chat cua vat lieu
- He so ke en s vt qua tai trong thiet ke
- He so ke en s lam viec tam thi hay lau dai
Nh vay muon am bao s lam viec an toan ve o ben khi thanh chu keo
(nen) ung tam, ng suat trong thanh phai thoa man ieu kien ben la:
z =

Nz
[ ]
F

(3.16)

T ieu kien ben, ta co ba bai toan c ban:


Kiem tra ben: z =

Nz
[ ] 5%
F

Chon kch thc mat cat ngang: F

Nz

[ ]

5%

nh tai trong cho phep: N z [ ]F 5%

hay:

[N z ] = [ ]F

Th du 3.4. Cho he nh H.3.17a. nh tai trong cho phep [P] theo ieu kien
ben cua cac thanh 1, 2, 3. Cho biet [ ] = 16 kN/cm2, F1= 2 cm2, F2= 1 cm2, F3=
2 cm2.
Giai. Trc tien ta can tnh noi lc trong cac thanh. Co lap he nh H.3.17b.
Xet can bang vi cac phng trnh:
X = 0 => N2 cos45o + N3 = 0
Y = 0 => P + N1 + N2 sin45o = 0
M/A = 0 => P2a + N1a = 0
Ta c N1 = 2P, N2 = P 2 (nen), N3 = P
Viet ieu kien ben cua cac thanh 1, 2, 3:
1 =

N1
2P
=

F1
F1

2 =

| N2 |
P 2
=
[ ] =>
F2
F2

3 =

N3
P
=

F3
F3

[ ] =>

[ ] =>

[ ]F1 =
2

16.2
2

= 16 kN

[ ]F2 = 16.1 = 11,3 kN


2

P [ ] F3 = 16.2 = 32 Kn

So sanh ta c [P] = 11,3 KN.


http://www.ebook.edu.vn

10

1
2

a)

45o
3
a

N1

N2

b)

N3

H. 3.17

3.8. BAI TOAN SIEU TNH


nh ngha: Bai toan sieu tnh la bai toan ma ch vi cac phng trnh can bang
tnh hoc se khong u e giai c tat ca cac phan lc hay noi lc trong he.
Cach giai. Can tm them cac phng trnh dien ta ieu kien bien dang cua he
sao cho cong so phng trnh nay vi cac phng trnh can bang tnh hoc va
u bang so an so phan lc, noi lc can tm.
Th du 3.5. Xet thanh chu lc nh H.3.18a. hai ngam co hai phan lc VA va
(a)
VB. Ta co phng trnh can bang: VA + VB P = 0
Phng trnh nay co hai an, muon giai c ta phai tm them phng
trnh ieu kien bien dang cua thanh.
Tng tng bo ngam B va thay bang phan lc VB (H.3.18b). ieu kien bien
dang cua he la:
L = BA = BC + CA = 0
(b)
Goi NBC va NCA la noi lc tren cac mat cat cua cac oan BC va CA ta se c:

L =

N BC L BC
N L
+ CA CA = 0
EF
EF

vi NBC = VB ; NCA = VB + P, (c) tr thanh:

(c)
VB b (VB + P )a
=0
+
EF
EF

Pa
a+b
Ta a tnh c phan lc VB, bai toan tr thanh bai toan tnh nh bnh
thng
suy ra:

VB =

http://www.ebook.edu.vn

11

VA

a
C

VB

VB

a)

H.3.18

b)

Th du 3.6. Xet he gom ba thanh treo lc P (H.3.19a) hay tnh noi lc trong cac
thanh treo.
Giai. Ta co hai phng trnh can bang ( tach nut A):
X = NAB sin + NAD sin = 0
(a)
Y = P + NAB cos + NAC + NAD cos = 0 (b)
e giai ba an so noi lc ta can them mot phng trnh ieu kien bien dang.
Xet he thanh sau khi chu lc. V oi xng nen iem A di chuyen theo phng
AC en A. T A ke ng AI va AK lan lt vuong goc vi AB va AD. Bien
dang nho nen goc ABA va ADA vo cung be va goc BAC va DAC van .
Suy ra IA la o dan dai cua AB va tng t KA la o dan dai cua AD.
Ngoai ra AA cung chnh la o dan dai cua AC
Xet tam giac AIA va AKA ta co lien he:
IA = KA = AAcos ( c )
Thay IA =

N L
N AB L
N L
; KA = AD
; AA = AC vao (c) roi vao (a) va (b) ta
EF cos
EF cos
EF

P
P cos 2
; NAC =
se cNAB = NAD =
3
1 + 2 cos
1 + 2 cos 3
D

NAB

NAC

NAD

EA
L

EA

EA

A
I

a)

K
A

b)

H.3.19

http://www.ebook.edu.vn

12

Th du 3.7. Cho thanh ABC tuyet oi cng lien ket khp tai A c treo bi
day CD co tiet dien F va co chieu dai L nh hnh ve.
1/ Tnh noi lc cua CD.
2/ Tnh [q] theo ieu kien ben cua thanh CD .
Cho biet [ ] = 16 kN/cm2, L=2m F1= 2 cm2 .
3/ Tnh chuyen v ng cua iem C . Cho E = 20000 kN/cm2
4/ Bay gi them thanh chong BH hay thanh treo CH (net cham) . Tnh lai noi
lc cua cac thanh chong CD vaBH
.
H
D
D
F
L

EF

M = 2qL 2

A
2qL

30

2L

qL2

P=2ql
B

1.5EF

A
L

L
A

H
L

L/2

L/2

Cho q =10kN/m, L = 1m , F = 1.5cm 2 , E=20000kN/cm 2 , [ ] = 16 kN/cm2


-Kiem tra ben thanh CD.
-Tnh chuyen v ng cua iem C

http://www.ebook.edu.vn

13

GV: Le c Thanh
____________________________________________________________________

Chng 4

TRANG THAI NG SUAT


4.1 NHNG KHAI NIEM VE TRANG THAI NG SUAT.
4.1.1 TRANG THAI NG SUAT (TTS)TAI MOT IEM.
Xet mot iem K trong mot vat the can
bang va cac mat cat qua K, tren cac mat
cat ay co cac ng suat phap va ng suat
tiep

y
P1

Cac ng suat nay thay oi tuy v tr

mat cat (H.4.1).


nh ngha TTS: TTS tai mot iem
la tap hp tat ca nhng ng suat tren cac
mat i qua iem ay.

P2

P3

P4

H.4.1. ng suat tai mot iem

TTS tai mot iem ac trng cho mc o chu lc cua vat the tai iem
o. Nghien cu TTS la tm ac iem va lien he gia cac ng suat , ,
xac nh ng suat ln nhat, nho nhat e tnh toan o ben hay giai thch,
oan biet dang pha hong cua vat the chu lc.
4.1.2 Bieu dien TTS tai mot iem

Tng tng tach mot phan to hnh

hop vo cung be bao quanh iem K. Cac

mat phan to song song vi cac truc toa

o (H 4.2).

Tren cac mat cua phan to se co chn


z
thanh phan ng suat:
H.4.2
+Ba ng suat phap: x , y , z
Cac thanh phan ng suat
+Sau ng suat tiep. xy , yx , xz , zx ,
yz , zy ,
ng suat phap co 1 ch so ch phng phap tuyen mat co .
y

yx

yz

xy

zy

zx

xz

ng suat tiep co hai ch so: Ch so th nhat ch phng phap tuyen cua


mat cat co , ch so th hai ch phng cua .
______________________________________________________________
Chng 4: Trang thai ng suat

http://www.ebook.edu.vn
1

GV: Le c Thanh
____________________________________________________________________

4.1.3 nh luat oi ng cua ng suat tiep


Tren hai mat vuong goc, neu mat nay co ng suat tiep hng vao
canh ( hng ra khoi canh ) th mat kia cung co ng suat tiep hng vao
canh ( hng ra khoi canh ), tr so hai ng suat bang nhau ( H.4.3)
(4.1)
xy = yx ; xz=zx ; yz =zy
TTS tai mot iem con 6 thanh phan ng suat

4.1.4 Mat chnh, phng chnh va ng suat chnh. Phan loai TTS
Ly thuyet an hoi a chng minh rang tai mot iem bat ky cua vat the
chu lc luon tm c mot phan to hnh hop vuong goc ma tren cac mat
cua phan to o ch co ng suat phap, ma khong co ng suat tiep (H4.4a).
Nhng mat o goi la mat chnh.
Phap tuyen cua mat chnh goi la phng chnh.
ng suat phap tren mat chnh goi la ng suat chnh va ky hieu la:

1 , 2 va 3. Quy c: 1 > 2 > 3.


Th du :

1 = 200 N/cm2;
2 = 400 N/cm2;
3 = 500 N/cm2
Phan loai TTS :
- TTS khoi : Ba ng
suat
chnh
khac
khong (H.4.4a).

a)

b)

c)

H. 4.4 Cac loai trang thai ng suat

- TTS phang: Hai ng suat chnh khac khong (H.4.4b).


- TTS n: Mot ng suat chnh khac khong (H.4.4c).
______________________________________________________________
Chng 4: Trang thai ng suat

http://www.ebook.edu.vn
2

GV: Le c Thanh
____________________________________________________________________

TTS khoi va TTS phang goi la TTS phc tap.


4.2 TTS TRONG BAI TOAN PHANG- PHNG PHAP GIAI TCH.
4.2.1 Cach bieu dien Quy oc dau
Cach bieu dien:

yy
yx

xy

x
K

xy
z

a)

yx

yx

yx

xy

a)y

uv
b)

b)

xy
v

H. 4.5 TTS trong bai toan phang

Xet mot phan to (H.4.5a). ng suat tren mat vuong goc vi truc z
bang khong va mat nay la mot mat chnh v co ng suat tiep bang khong.
e de hnh dung, ta bieu dien phan to ang xet bang hnh chieu cua
toan phan to len mat phang Kxy (H.4.5b).
Quy c dau: + > 0 khi gay keo ( hng ra ngoai mat cat)
+ > 0 khi lam cho phan to quay thuan kim ong ho
Hnh 4.5b bieu dien cac ng suat > 0
(qui c nay phu hp vi bai toan thanh)
4.2.2 ng suat tren mat cat nghieng bat ky
Van e: Xac nh ng suat tren mat cat nghieng song song vi truc z va co
phap tuyen u tao vi truc x mot goc ( > 0 khi quay ngc chieu kim
ong ho ke t truc x ) (H.4.6a). Gia thiet a biet ng suat x, y va xy.
Tnh u va uv : Tng tng cat phan to bang mat cat nghieng a
neu, mat cat chia phan to ra lam hai phan, xet can bang cua mot phan
phan to (H.4.6b)

______________________________________________________________
Chng 4: Trang thai ng suat

http://www.ebook.edu.vn
3

GV: Le c Thanh
____________________________________________________________________

y
u
u

ds

xy

dy

dz

x
xy

uv
v
yx

dx

uv
yx

y
b)

a)

H.4.6 ng suat tren mat nghieng

Tren mat nghieng co ng suat u va uv , chung c xac nh t


phng trnh can bang tnh hoc.
* U=0

u dsdz x dzdy cos + xy dzdy sin y dzdx sin + xy dzdx cos = 0

* V=0

uv dsdz x dzdy sin xy dzdy cos + y dzdx cos + xy dzdx sin = 0

Ke en: xy = yx ; dx = ds sin ; dy = ds cos,


1
1
(1 + cos 2 ); sin 2 = (1 cos 2 )
2
2
1
sin cos = sin 2
2
cos 2 =

u =

x +y
2

uv = +

x y
2

x y
2

cos 2 xy sin 2 (4.2a)

sin 2 + xy cos 2

(4.2b)

Tnh v : Xet mat nghieng co phap


tuyen v, vuong goc mat co phap tuyen u
(H.4.7). Thay the bang ( + 90) vao (4.2a)
,

v
o
+ 90

vu

uv u

ng suat phap tac dung tren mat co phap

tuyen v:
v =

x +y
2

x y
2

cos 2 + xy sin 2

(4.3)

ng suat tren
H. 4.7
2 mat vuong goc nhau

Tong (4.2a) va (4.3),


______________________________________________________________
Chng 4: Trang thai ng suat

http://www.ebook.edu.vn
4

GV: Le c Thanh
____________________________________________________________________

u +v = x + y

(4.4)

Bieu thc tren cho thay, tong ng suat phap tac dung tren hai mat
vuong goc cua phan to ng suat phang tai mot iem la hang so va khong
phu thuoc vao goc .
o la Bat Bien Th Nhat cua ng suat phap
Th du 4.1 Thanh co dien tch 5 cm2, chu keo vi lc P = 40 kN. Xac nh
ng suat tren mat cat nghieng mot goc 30o vi mat cat ngang (H.4.8).
Giai
ng suat phap tren mat cat ngang (Chng 3)
x =

P 40
=
= 8 kN/cm 2
5
F

30o

Tach phan to hnh hop bao iem K


nam tren mat cat ngang.
Ta co: x = + 8 kN/cm 2 , y = 0

uv = +

x
2

cos 2 =

P = 40 kN

v
u
u

hp vi truc vi truc x (truc thanh) mot


goc( +30o ).
T (4.2)
x

uv

Mat cat nghieng co phap tuyen

n =

30
x

uv

H.4.8

8
1 + cos 2.30o = 6 kN/cm 2
2

8
sin 2 = + sin 2.30o = + 3,46 kN/cm 2
2
2

4.2.3 ng suat chnh - Phng chnh - ng suat phap cc tr


1- ng suat chnh - phng chnh

( 2)

Ngoai mat chnh la mat a biet vuong goc

(1)

vi truc z, hai mat chnh con lai la nhng mat


song song vi truc z (v phai vuong goc vi
mat chnh a co).
Mat chnh la mat co ng suat tiep = 0 Tm
hai mat chnh con lai bang cach cho uv =0

(1)

o =o +90
x

1
2

H. 4.9ng suat chnh

______________________________________________________________
Chng 4: Trang thai ng suat

http://www.ebook.edu.vn
5

GV: Le c Thanh
____________________________________________________________________

Neu goi o la goc cua truc x hp vi phng chnh th ieu kien e tm


phng chnh la: uv =0 +

sin 2 + xy cos 2 = 0

tan 2 o =

Phng trnh xac nh 0 :

o =

x y

01 =

2 xy

x y
02 =

va

= tan

(4.5)

(4.5) cho thay co hai gia tr 0 sai biet nhau 90. V vay, co hai mat chnh
vuong goc vi nhau va song song vi truc z. Tren moi mat chnh co mot
ng suat chnh tac dung.
Hai ng suat chnh nay cung la ng suat phap cc tr (ky hieu la
max hay min ) bi v
2 xy
d u
= 0 tan 2 =
dz
x y

giong vi (4.5)

Gia tr ng suat chnh hay ng suat phap cc tr co the tnh c


bang cach the ngc tr so cua trong (4.5) vao (4.2a).
e y rang:

sin 2 o =

max = 1,3 =
min

x +y
2

tan 2 o

; cos2 o =

1 + tan 2 o
2

1
1 + tan 2 2 o

x y
+ xy2

2

(4.6)

Ta lai thay max + min = 1 + 3 = x + y


Th du 4.2 Tm ng suat
chnh va phng chnh cua
TTS (H.4.10a). n v cua
ng suat la kN/cm2.

1
4

Giai
Theo quy c dau, ta co:

67o30

1
x

a)

H. 4.10

22o30

b)

x = 4 kN/cm 2 ; y = 2 kN/cm 2 xy = +1 kN/cm 2

Phng chnh xac nh t (4.5):


tan 2 o =

2 xy

x y

2
= 1 2 o = 45o + k180 o
42

o(1) = 22 o 30' ; o( 2 ) = 67 o 30'

(i)

______________________________________________________________
Chng 4: Trang thai ng suat

http://www.ebook.edu.vn
6

GV: Le c Thanh
____________________________________________________________________

Co 2 phng chnh ( 2 mat chnh) vuong goc nhau


Cac ng suat chnh c xac nh t (4.6):
max
min

2
4,41 kN/cm 2
4+2
4 2
=

+1 = 3 2 =
2
1,58 kN/cm 2
2

(ii)

e xac nh mat chnh nao t (i) co ng suat chnh (ii) tac dung, ta
dung (4.2b), chang han vi o(1) = 22 o 30' , ta co:
u =

4+2 42
+
cos 2 22 o 30' 1sin 2 22o 30' = 4,41 kN/cm2
2
2

Vay : 1 = 4,41 kN/cm2 ng vi goc nghieng

o(1) = 22 o 30' ,

2 = 1,58 kN/cm2 tac dung tren mat co

o( 2) = 67 o 30' .

Cac mat va ng suat chnh bieu dien tren phan to H.4.10b.


2- ng suat tiep cc tr
Tm ng suat tiep cc tr va mat nghieng tren o co ng suat tiep cc
tr bang cach cho

d uv
= 0
d

(2) (2) o

1 =o +45

d uv
= ( x y ) cos 2 2 xy sin 2 = 0
d
x y
(4.7)
=
tan 2 =
2 xy

So sanh (4.7) vi (4.5)

tan 2 =

(4.7)

max

ng suat tiep cc tr
H.
4.11

1
tan 2 o

(4.8)

= o k45o

2 = 2 o k90o hay

Mat co ng suat tiep cc tr hp vi nhng mat chnh mot goc 45.


The (4.8) vao (4.2b), ta c :
x y
+ xy2
=
2

max
min

4.2.4 Cac trng hp ac biet

(4.9)

TTUSphang ac biet

1- TTS phang ac biet


Phan to tren H.4.12 co: x = ; y = 0; xy =
T

H.4.12

(4.6)

______________________________________________________________
H. 4.13 TTUS Trt thuan tuy
http://www.ebook.edu.vn
7
Chng 4: Trang thai ng suat

GV: Le c Thanh
____________________________________________________________________

max = 1, 3 =

min

1
2 + 4 2
2

(4.10)

Phan to co 2 ng suat chnh ( se gap trng hp thanh chu uon ).


2- TTS trt thuan tuy (H.4.13)
ay,

x = y = 0 ; xy =

max = 1, 3 = hay

;Thay vao (4.6)

min

(4.11)

1 = 3 =

Hai phng chnh c xac nh theo (4.5):

o = + k
tan 2 o =
(4.12)
4

Nhng phng chnh xien goc 45o vi truc x va y.

H. 4.14

3- Trng hp phan to chnh (H.4.14)


Phan to chnh ch co 1 , 3 , = 0;
Thay vao (4.9), ta c: max,min =

1 3
2

(4.13)

4.3 TTS TRONG BAI TOAN PHANG- PHNG PHAP O TH.


1- Vong tron Mohr ng suat.
Cong thc xac nh ng suat tren mat cat nghieng (4.2) co the bieu
dien di dang hnh hoc bang vong tron Mohr. e ve vong tron Mohr, ta
sap xep lai (4.2) nh sau:
u
uv =

x + y
2
x y
2

x y
2

(4.14)

cos 2 xy sin 2

(4.14)

sin 2 + xy cos 2

Bnh phng ca hai ve cua hai ang thc tren roi cong lai, ta c:
+y

u x
2

at:

c=

x +y
2

y
+ uv2 = x
2

x y
; R 2 =
2

(4.15) thanh:

+ xy2

( u c )2 + uv2 = R 2

+ xy2

(4.15)

(4.16)

(4.17)

Trong he truc toa o, vi truc hoanh va


truc tung , (4.17) la phng trnh cua mot
ng tron co tam nam tren truc hoanh vi

H. 4.15 Vong
tron ng suat

hoanh o la c va co ban knh R . Nh vay, cac


gia tr ng suat phap va ng suat tiep tren tat ca cac mat song song vi
______________________________________________________________
Chng 4: Trang thai ng suat

http://www.ebook.edu.vn
8

GV: Le c Thanh
____________________________________________________________________

truc z cua phan to eu bieu th bang toa o nhng iem tren vong tron. Ta
goi vong tron bieu th TTS cua phan to la vong tron ng suat hay vong
tron Mohr ng suat cua phan to.
Cach ve vong tron: (H.4.16)

- nh he truc toa o O : truc hoanh // truc x, truc tung // truc y cua


phan to va hng len
P
tren.

-Tren truc nh iem


O
E
F C
E(x, 0) va iem F(y, 0)
y
Tam C la trung iem
x
cua EF
xy

Cach ve vong tron ng suat


H.4.16

- nh iem cc P (y,
xy ) .

- Vong tron tam C, qua


P la vong tron Mohr can ve
Chng minh: + C la trung iem cua EF OC =
Trong tam giac vuong CPF: FC =

OE + OF x + y
=
=c
2
2

OE OF x y
=
; FP = xy
2
2

y
+ xy2 = R 2
Do o CP = FC + FP = x
2
2

2- ng suat tren mat cat nghieng

______________________________________________________________
Chng 4: Trang thai ng suat

http://www.ebook.edu.vn
9

GV: Le c Thanh
____________________________________________________________________
u

max
max

uv

xy

yx

B
xy

uv

uv

max

min
y
minx

x
u
max

H. 4.17 nh n g suat tren mat nghien g

Dung vong tron Mohr e tm ng suat tren mat cat nghieng cua phan
to co phap tuyen u hp vi truc x mot goc .
Cach tm u ; uv
Ve vong tron Mohr nh H.4.17.
T cc P ve tia Pu // vi phng u cat vong tron tai iem M.
Hoanh o cua M = u ; Tung o cua M = uv
Chng minh:
Ky hieu 21 la goc (CA,CD), 2 la goc (CD,CM).
Hnh 4.17 cho:
OG = OC + CG =
=

nhng:
nen:

x +y
2

x +y

+ R cos(21 + 2 )

+ R cos 21 cos 2 R sin 21 sin 2

R cos 21 = CE =

x y

OG =

Rsin 2 1 = ED = xy

x +y
2

x y
2

cos 2 xy sin 2 = u

Tng t, ta co:

GM = R sin (21 + 2 ) = R cos 21 sin 2 + R sin 21 cos 2


y
sin 2 + xy cos 2 = uv
= x
2

Ta nhan lai c phng trnh (4.2)


3- nh ng suat chnh- phng chnh- ng suat phap cc tr
______________________________________________________________
Chng 4: Trang thai ng suat

http://www.ebook.edu.vn
10

GV: Le c Thanh
____________________________________________________________________

Tren vong tron ng suat ( H.4.17)


iem A co hoanh o ln nhat, tung o = 0 max = OA ; =0
Tia PA bieu dien mot phng chnh.
iem B co hoanh o nho nhat, tung o = 0 min = OB ; =0
Tia PB bieu dien phng chnh th hai.
4- nh ng suat tiep cc tr
Tren vong tron (H.4.17): hai iem I va J la nhng iem co tung o
ln va nho nhat. Do o, tia PI va PJ xac nh phap tuyen cua nhng mat
tren o co ng suat tiep cc ai va cc tieu. Nhng mat nay tao vi nhng
mat chnh mot goc 45o.
ng suat tiep cc tr co tr so bang ban knh ng tron.
ng suat phap tren mat co ng suat tiep cc tr co gia tr bang hoanh
o iem C, tc la gia tr trung bnh cua ng suat phap:
tb =

x +y

5- Cac trng hp ac biet


- TTS phang ac biet
Phan to co hai ng suat
chnh 1 va 3 (H.4.18).
- TTS trt thuan tuy
Phan to co 2 ng suat chnh:
1 = 3 = | |

Cac phng chnh xien goc


45 vi truc x va y (H.4.19)
o

a)

Th du 4.3 Phan to TTS phang


(H.4.21),cac ng suat tnh theo

E
max

b)

min

H. 4.18 TTS phang ac biet va vong Morh

min = -

max =

H. 4.19TTS trt thuan tuy va vong Morh

- TTS chnh ( H.4.20)


1 2

max, min =

max
1

max

C
2
1

min

H. 4.20 TTS CHNH- Vong Morh

______________________________________________________________
Chng 4: Trang thai ng suat

http://www.ebook.edu.vn
11

GV: Le c Thanh
____________________________________________________________________

kN/cm2. Dung vong tron Mohr, xac nh:


a) ng suat tren mat cat nghieng = 45o
b) ng suat chnh va phng chnh
c) ng suat tiep cc tr.

45

o(3)= 26 36

uv
1

71 36
o

45

o
161 36'

max

B
-7

-5

-2

C
3

M
u

H. 4.21

A
F
(1)
O1 3
o
o = - 67 24
4
5

uv

min

Giai.
Theo quy c ta co:
x = 5 kN/cm 2 ; y = 1 kN/cm 2 ; xy = + 4 kN/cm 2

Tam vong tron C 5 + 1 ,0 .

Cc P(1, + 4). T P ve tia song song vi truc u cat vong tron Mohr
tai M. Toa o iem M bieu th ng suat tren mat cat nghieng vi = 45o :
u = 6 kN/cm 2 ; uv = 3 kN/cm 2

Hoanh o A va B bieu th ng suat chnh co gia tr bang:


1 = A = 3 kN/cm 2 ; 3 = B = 7 kN/cm 2

Hai phng chnh xac nh bi goc o:


o(1) = 67 o 42' ; o(3) = 26 o 36'

Tung o I va J co gia tr bang ng suat tiep cc tr:


max = 5 kN/cm 2 ; min = 5 kN/cm 2

Cac ng suat nay tac dung len cac mat, tng ng vi cac goc
nghieng: 1(1) = 71o36' ; 1( 2) = 161o36'

______________________________________________________________
Chng 4: Trang thai ng suat

http://www.ebook.edu.vn
12

GV: Le c Thanh
____________________________________________________________________

4.3 BIEU DIEN HNH HOC TTS KHOI


II

2
1

x
z

2
1 1

III

H. 4.23TTS khoi va cac mat // truc chnh

H.4.22. TTS khoi vi mat


cat nghieng bat ky

Tong quat, TTS tai mot iem la TTS khoi (H.4.22).


Xet nhng mat // mot phng chnh ( th du phng III) , ng suat
chnh 3 khong anh hng en , tren cac mat nay (H.4.23). co the
nghien cu ng suat tren nhng mat nay tng t TTS phang.
Ve vong tron ng suat bieu

max,
dien cac ng suat tren mat nghieng
max,
nay (vong tron so 3 tren H.4.24) .
max,3
T vong tron nay, ta thay tren
3

nhng mat song song vi phng


2

1
3O
1
2
1
chnh III co mat co ng suat tiep cc
ai (ky hieu max,3) ,

max,3 =

1 2
2

H.4.24
Ba vong tron Mohr ng suat

Tng t, oi vi nhng mat


song song vi phng chnh th I va th II, ta cung ve c cac vong tron
ng suat (Vong tron so 1 va vong tron so 2) (H.4.24).
Ly thuyet an hoi a chng minh rang gia tr cua va tren mot mat
bat ky cua mot phan to trong TTS khoi co the bieu th bang toa o cua
mot iem nam trong mien gach cheo ( H.4.24 ).
Qua hnh ve, ng suat tiep ln nhat cua phan to bieu th bang ban
knh cua vong tron ln nhat, (H.4.24).

max, 2 =

1 3
2

(18)

______________________________________________________________
Chng 4: Trang thai ng suat

http://www.ebook.edu.vn
13

GV: Le c Thanh
____________________________________________________________________

4.4 LIEN HE NG SUAT VA BIEN DANG

II 2

4.4.1 nh luat Hooke tong quat


1- Lien he ng suat phap va bien dang z

dai
TTS n: trong chng 3, a co:

3
III

nh luat Hooke lien he gia ng suat phap


va bien dang dai :

H.4.25. TTS khoi

(4.19)

- bien dang dai tng oi theo phng .


Theo phng vuong goc vi cung co bien dang dai tng oi
ngc dau vi :

' = =

(4.20)

TTS khoi: vi cac ng suat chnh 1, 2 , 3 theo ba phng chnh


I, II, III (H.4.25). Tm bien dang dai tng oi 1 theo phng I .
Bien dang dai theo phng I do 1 gay ra:

1 (1 ) =

1
E

Bien dang dai theo phng I do 2 gay ra: 1 ( 2 ) =

Bien dang dai theo phng I do 3 gay ra: 1 ( 3 ) =

E
E

Bien dang dai tng oi theo phng I do ca ba ng suat 1, 2 , 3


sinh ra se la tong cua ba bien dang tren:
1 = 1 ( 1 ) + 1 ( 2 ) + 1 ( 3 ) =

1
[ 1 ( 2 + 3 )]
E

(4.21)

Tng t, bien dang dai tng oi theo hai phng chnh II , III con lai:
1
[ 2 ( 3 1 )]
E
1
3 = [ 3 ( 1 + 2 )]
E

2 =

(4.22)
(4.23)

TTS tong quat: Ly thuyet an hoi a chng minh oi vi vat lieu


an hoi ang hng, ch sinh ra bien dang dai ma khong sinh ra bien
dang goc , ch sinh ra bien dang goc ma khong sinh ra bien dang dai.
Trong trng hp phan to TTS tong quat, van co

______________________________________________________________
Chng 4: Trang thai ng suat

http://www.ebook.edu.vn
14

GV: Le c Thanh
____________________________________________________________________

1
x ( y + z )
(4.24)
E
1
y = y ( z + x )
E
1
z = z ( x + y )
E

x =

2-Lien he gia ng suat tiep va bien dang


goc

( nh luat Hooke ve trt)

Phan to TTS trt thuan tuy (H.4.26). Bien


dang goc (goc trt) bieu th o thay oi
H. 4.26 TT S tr t thuan tuygoc vuong.
Bien da ng goc
nh luat Hooke ve trt:
=

(4.25)

trong o: G - la moun an hoi trt. Th nguyen cua G la [lc/(chieu dai)2]


va n v thng dung la N/m2 hay MN/m2.
Lien he gia E, va G nh sau:

G=

E
2(1 + )

(4.26)
4.4.2 nh luat Hooke khoi
Tnh o bien oi the tch cua mot phan to hnh
hop co cac canh bang da1, da2 va da3 .
The tch cua phan to trc bien dang la:

II

x
z

2
1

Vo = da1 da2 da3

III

Sau bien dang, phan to co the tch la:

H.4.27. TTS khoi

V1 = (da1 + da1 )(da 2 + da2 )(da 3 + da3 )

Goi bien dang the tch tng oi la , ta co:


V V
= 1 o = 1 + 2 + 3
Vo

(4.27)

______________________________________________________________
Chng 4: Trang thai ng suat

http://www.ebook.edu.vn
15

GV: Le c Thanh
____________________________________________________________________

The (4.21)(4.22),(4.23) vao (4.27)


1 2
( 1 + 2 + 3 )
= 1 + 2 + 3 =
E
at tong ng suat phap la:
(4.28) thanh:

(4.28)

= 1 + 2 + 3

1 2

(4.29)

cong thc (4.29) c goi la nh luat Hooke khoi bieu th quan he tuyen
tnh gia bien dang the tch tng oi va tong ng suat phap.

Nhan xet :
T (4.29), neu vat lieu co he so Poisson = 0,5 ( cao su), th luon
luon bang khong tc la the tch khong oi di tac dung cua ngoai lc.
Cong thc tren cho thay phu thuoc vao tong ng suat phap ch
khong phu thuoc vao rieng tng ng suat phap. Nh vay, neu cung vi
phan to ay ta thay cac ng suat chnh bang mot ng suat trung bnh tb co
gia tr bang trung bnh cong cua ba ng suat chnh noi tren:
tb =

1 + 2 + 3
=
3
3

th bien dang the tch tng oi cua phan to tren van khong thay oi.
That vay, vi nhng ng suat chnh la tb , bien dang the tch bang:
1 =

1 2
( tb + tb + tb ) = 1 2
E
E

Ket qua tren co y ngha nh sau: vi phan to ban au la hnh lap


phng, trong hai trng hp tren ta thay the tch phan to eu bien oi nh
nhau.
- Tuy nhien, trong trng hp au khi cac ng suat chnh khac nhau,
phan to va bien oi the tch va bien oi hnh dang tc la tr thanh
phan to hnh hop ch nhat sau khi bien dang.
- Con trong trng hp th hai, khi thay cac ng suat chnh bang ng
suat trung bnh, phan to ch bien oi ve the tch ma khong bien oi hnh
dang, ngha la sau khi bien dang phan to van gi hnh lap phng.
- Ve mat ly luan, co the phan phan to TTUS khoi chu cac ng suat
chnh 1 , 2 , 3 thanh 2 phan to (H. 4.28). Phan to b) ch bien oi the tch,
phan to c) ch bien oi hnh dang.
______________________________________________________________
Chng 4: Trang thai ng suat

http://www.ebook.edu.vn
16

GV: Le c Thanh
____________________________________________________________________
2

tb

2 - tb

1 - tb

tb

tb

3 - tb

a)

c)

b)

H.4.28 Phn tch TTUS khoi thanh 2 TTUS

4.5 THE NANG BIEN DANG AN HOI


chng 3, phan to TTS n (thanh b keo hoac nen):
u = 2 (4.30)
The nang bien dang an hoi rieng
Trong TTS khoi, s dung nguyen ly oc lap tac dung, ta co the
nang bien dang an hoi rieng bang:
u=

1 1
2

2 2
2

3 3

(4.31)

thay 1, 2, 3 theo nh luat Hooke trong (4.21) - (4.23) vao ,


1
{ 1[ 1 ( 2 + 3 )] + 2 [ 2 ( 3 + 1 )] + 3 [ 3 ( 2 + 1 )]}
2E
1 2
(4.32)
u=
1 + 22 + 32 2 ( 1 2 + 2 3 + 3 1 )
2E

u=

hay

Ta co the phan tch the nang bien dang an hoi u thanh hai thanh phan:
-Thanh phan lam oi the tch goi la the nang bien oi the tch utt
-Thanh phan lam oi hnh dang goi la the nang bien oi hnh dang uhd
Ta co:
u = utt + uhd
e tnh the nang bien oi hnh dang, ta thay cac ng suat 1, 2 va 3
bang ng suat (1 -tb ), (2 -tb ), (3 -tb ), tac dung len cac mat phan to.
2

tb

2 - tb

1 - tb

tb

tb

3 - tb

H.4.29 Pha n tch TTS tha nh hai TTS

The vao (4.32) ta co the nang bien oi hnh dang bang:


uhd =

1 2
1 2
( 1 + 2 + 3 )2
1 + 22 + 32 2 ( 1 2 + 2 3 + 3 1 )
2E
6E

______________________________________________________________
Chng 4: Trang thai ng suat

http://www.ebook.edu.vn
17

GV: Le c Thanh
____________________________________________________________________

uhd =

hay :

1+ 2
1 + 22 + 32 1 2 2 3 1 3
3E

(4.33)

TTS n , thay 1 = ; 2 = 0; 3 = 0 vao (4.32) va (4.33), ta c the


nang rieng va the nang bien oi hnh dang nh sau:
u=

2
2E

uhd =

1+ 2

3E

(4.34

Th du 4.4: Cho phan to nh hnh ve:


trang thai ng suat phang.
Tnh x , y , u (phng utao vi truc x mot goc 30 0 .
Cho E=104kN/cm2 , =0,34 ,
Ta co x = 6kN / cm

=300

u
y

6kN/cm 2

y = 8kN / cm 2

= 2kN / cm 2
= 60

2kN/cm 2

8kN/cm 2

1
1
[
x y ] = 4 [6 (0,34)8] = 3,28 10 4
E
10
1
1
y = [ y yy ] = 4 [8 (0,34)6] = 5,96 10 4
E
10
+ y x y
u = x
+
cos 2 xy sin 2 = 9,232kN / cm 2
2
2
1
1
u = [ u v ] = u ( x + y u = 7,611kN / cm 2
E
E

x =

______________________________________________________________
Chng 4: Trang thai ng suat

http://www.ebook.edu.vn
18

GV: Le c Thanh
____________________________________________________________________

Th du 4.5:
Mot khoi lap phng bang be tong at va kht vao ranh cua vat the A
(tuyet oi cng) chu ap suat phan bo eu mat tren P= 1kN/cm2 (H.4.11).
Xac nh ap lc nen vao vach ranh, lien he gia ng suat va bien dang
dai tng oi theo cac phng. o bien dang the tch tuyet oi. Cho
canh a = 5 cm; E = 8.102 kN/cm2; = 0,36.
Chon he truc nh hnh ve.Ta co: khoi be tong TTSphang .

x 0; y = p kN/cm2 ;

z = 0

z 0; y 0 ;

a
P

x = 0

nh luat Hooke cho bien dang dai:


z

x =

H.4.11

y =

1
x ( y + z ) = 0
E

x = p = -(0,36 1) = 0,36 kN/cm 2

1
p
y ( x + z ) =
(1 - 2 )
E
E

z =

1
1
p
z ( x + y ) = [0 - (-p - p)] = (1 + )
E
E
E

Bien dang the tch tuyet oi:

1 2
x +y +z) V
E
1 - (2 0,36)
[ 0,36 1](5 5 5) = - 0,0559cm3
=
800
v = V =

______________________________________________________________
Chng 4: Trang thai ng suat

http://www.ebook.edu.vn
19

GV: Le c Thanh
____________________________________________________________________

Thdu4.6
Mot tam mong co kch thc nh tren H.4.5
chu tac dung cua ng suat keo = 30 kN/cm2
theo phng chieu dai cua tam
m
va ng suat tiep = 15 kN/cm2.
a) Xac nh ng suat phap theo phng
ng cheo mn va phng vuong goc vi ng cheo
b) Tnh bien dang dai tuyet oi cua ng cheo mn.

15 mm

25 mm

H45

Cho E = 2.104 kN/cm2, = 0,3


.Goi

u =

u =

u = mm ,

u =

lmm
lmm = lmm u
lmm

1
[ u v ]
E

30 + 0 30 0
+
cos 600 (15) sin 600 = 35,5kN / cm 2
2
2
1
[ u ( u u ) ] = 1,8575 . 10 3
= mm =
E

lu = lmm = 1,8575.10 3 50 = 0,093mm

______________________________________________________________
Chng 4: Trang thai ng suat

http://www.ebook.edu.vn
20

GV: Le c Thanh
____________________________________________________________________

BAI TAP CHNG 4


4.1 Tm gia tr ng suat phap va ng suat tiep tren mat cat AB cua phan to
nh tren H.4.1 bang phng phap giai tch va o th. n v ng suat tnh
bang kN/cm2.
c)

b)

2
B

3
4
50o

30o

b)

a)

c)

60o

7
60o

30o

A
e)

d)

H. 4.1

f)

4.2 Tren hai mat tao vi nhau mot goc = 60o va i


qua mot iem TTS phang co cac ng suat nh
tren H.4.2. Hay tnh cac ng suat chnh tai iem o,
ng suat phap u va bien dang tng oi u theo
phng u. Cho: E = 2.10 kN/cm ; = 0,3.
2

u
6 kN/cm2
60

5 kN/cm2
3 kN/cm2

H.4.2

4.3 Tren mat cat m - n i qua mot iem trong vat the

m
TTS phang co ng suat toan phan p = 3000 N/cm2,
p
60
ng suat nay co phng tao thanh goc 60o vi mat
45
cat. Tren mat vuong goc vi mat cat o ch co ng
n
suat tiep (H.4.3).
H. 4.3
Tnh ng suat phap va ng suat tiep tren mat cat hp
vi mat cat m - n mot goc 45o. Tnh ng suat phap ln nhat tai iem o.
o

______________________________________________________________
Chng 4: Trang thai ng suat

http://www.ebook.edu.vn
21

GV: Le c Thanh
____________________________________________________________________

4.4 Tai mot iem tren be mat cua vat the, ng


suat tac dung len phan to nghieng mot goc 30o
vi truc x co tr so va hng nh tren H.4.30.
a) Xac nh ng suat chnh va phng chnh.
b) Xac nh ng suat tiep cc tr va ng suat
phap tren be mat co ng suat tiep cc tr. Bieu
dien cac ng suat o tren H.4.4.

y
3 kN/cm2

5 kN/cm2

= 30o
x

H. 4.4

4.5 Mot tam mong co kch thc nh tren

H.4.5 chu tac dung cua ng suat keo


= 30 kN/cm2 theo phng chieu dai cua
tam va ng suat tiep = 15 kN/cm2.
m
25 mm
a) Xac nh ng suat phap theo phng
ng cheo mn va phng vuong goc
H45
vi ng cheo
b) Tnh bien dang dai tuyet oi cua ng cheo mn.

15 mm

Cho E = 2.104 kN/cm2, = 0,3.


4.6 Mot tam thep mong hnh ch nhat chu ng suat phap phan bo eu x
va y nh tren H.4.6. Cac tam ien tr A va B c gan len tam theo hai
phng x va y cho cac so o nh sau: x = 4,8.104 va y = 1,3.104.
Tnh x va y, biet E = 2.104 kN/cm2; = 0,3.
n
o

45
B

A
x

B
O

H. 4.6

45o
A
m

H. 4.7

4.7 Tai mot iem tren mat vat the chu lc, ngi ta gan cac tam ien tr
A, B, C e o bien dang ty oi theo cac phng Om, On va Ou (H.4.7).
Cac so o thu c: m = 2,81.104 ; n = 2,81.104 ; u = 1,625.104
Xac nh ng suat chnh, phng chnh tai iem o.
Cho : E = 2.104 kN/cm2 ; = 0,3.
______________________________________________________________
Chng 4: Trang thai ng suat

http://www.ebook.edu.vn
22

GV: Le c Thanh
____________________________________________________________________

4.8 Tai iem A cua mot dam cau co gan hai


tenxomet e o bien dang theo phng
nam ngang va phng thang ng (H.4.8).

Khi xe chay qua cau, ngi ta o c: x

y
x
y

= 0,0004; y = 0,00012.Tnh ng suat


phap theo phng doc va phng thang

H.4.8

ng cua dam. Cho biet E = 2.104 kN/cm2; = 0,3.

4.9

Co mot phan to hnh hop co cac canh: a = 2cm;

P1

b = 4 cm; c = 2 cm, chu tac dung cua cac lc P1, P2


tren bon mat cua phan to (xem H.4.9). Cho : P1 = 60
kN; P2 = 120 kN; E = 2.104 kN/cm2; = 0,3.
a) Xac nh cac bien dang dai a, b, c cua cac canh
a, b, c va bien oi the tch cua phan to hnh hop.
b) Muon bien oi the tch V = 0 th phai at them lc
phap tuyen P3 bang bao nhieu vao hai mat con lai?
Tnh max trong trng hp nay.

P2

P2

P1

H.4.9

4.10 Mot khoi hnh hop lam bang thep co kch thc cho tren H.4.10, c
at gia hai tam cng tuyet oi, chu lc nen P = 250 kN. Tnh lc tac
dung tng ho gia mat tiep xuc cua hnh hop vi cac tam cng. Cho =
0,3.
.
P
y
m
c
5

m
c
0
1

b)
a)

H. 4.10

5cm

______________________________________________________________
Chng 4: Trang thai ng suat

http://www.ebook.edu.vn
23

GV: Le c Thanh
____________________________________________________________________

4.11 Mot khoi lap phng bang be tong at va kht ranh cua vat the A
chu ap suat phan bo eu mat tren P = 1 kN/cm2 (H.4.11).
Xac nh ap lc nen vao vach ranh va o bien dang the tch tuyet oi.
Cho canh a = 5 cm; E = 8.102 kN/cm2; = 0,36.
. Vat the A coi nh cng tuyet oi.
4.12 Mot tam thep kch thc a b c at gia hai tam tuyet oi cng, hai
tam nay c lien ket vi nhau bang bon thanh nh H.4.12. Khi tam
thep chu ap lc p phan bo tren hai mat ben th ng suat keo cua thanh
la bao nhieu? Tnh ng suat chnh trong tam thep. Cho Etam = Ethanh va
dien tch F cua thanh.
p

p
z

H.4.12

______________________________________________________________
Chng 4: Trang thai ng suat

http://www.ebook.edu.vn
24

GV: Le c Thanh
Thanhg Tuan

Chng 5

LY THUYET BEN
5.1 KHAI NIEM VE LYTHUYET BEN
ieu kien ben thanh chu keo hoac nen ung tam ( chng 3),
( TTS n) :
max = 1 []k ;
min = 3 []n
trong o,

hiem cua vat lieu (o )


[ng suat cho phep ] = ng suat nguy
[
;
]= 0
He so an toan
n

ng suat nguy hiem 0 co c t nhng th nghiem keo (nen) ung tam:


- oi vi vat lieu deo la gii han chay ch
- oi vi vat lieu don la gii han ben b.
e viet ieu kien ben mot iem cua vat the TTS phc tap
(phang hay khoi), can phai co ket qua th nghiem pha hong nhng mau th
TTS tng t. Viec thc hien nhng th nghiem nh the rat kho khan v:
- ng suat nguy hiem phu thuoc vao o ln cua cac ng suat chnh va
phu thuoc vao t le gia nhng ng suat nay. Do o phai thc hien mot so
lng rat ln cac th nghiem mi ap ng c t le gia cac ng suat chnh
co the gap trong thc te
- Th nghiem keo, nen theo ba chieu can nhng thiet b phc tap, khong
pho bien rong rai nh th nghiem keo nen mot chieu
V vay, khong the can c vao th nghiem trc tiep ma phai da tren cac
gia thiet ve nguyen nhan gay ra pha hong cua vat lieu hay con goi la nhng
thuyet ben e anh gia o ben cua vat lieu.
nh ngha :Thuyet ben la nhng gia thuyet ve nguyen nhan pha hoai
cua vat lieu, nh o anh gia c o ben cua vat lieu moi TTS khi ch
biet o ben cua vat lieu TTS n ( do th nghiem keo, nen ung tam).
Ngha la, vi phan to TTS bat ky co cac ng suat chnh 1, 2, 3, ta
phai tm ng suat tnh theo thuyet ben la mot ham cua 1, 2, 3 roi so sanh
vi [] hay [] TTS n.
ieu kien ben cua vat lieu co the bieu dien di dang tong quat nh
sau: t = t = f ( 1 , 2 , 3 ) [ ]k ( hay t = f ( 1 , 2 , 3 ) [ ]n )
t , t c goi la ng suat tnh hay ng suat tng ng. Van e la
phai xac nh ham f hay la tm c thuyet ben tng ng.

Chng 5: Ly Thuyet Ben

http://www.ebook.edu.vn

GV: Le c Thanh
Thanhg Tuan

5.2 CAC THUYET BEN (TB) C BAN


1- Thuyet ben ng suat phap ln nhat (TB 1)
Nguyen nhan vat lieu b pha hong la do ng suat phap ln nhat cua
phan to TTS phc tap at en ng suat nguy hiem TTS n.
Neu ky hieu:
II
II
1 , 2 , 3 : ng suat chnh
2
cua TTS phc tap
0k
0k hay 0n - ng suat nguy
1
0k
I
I
hiem ve keo va nen
3
n - he so an toan
III
III
ieu kien ben theo TB 1:
H.5.1. TTS khoi H.5.2. Trang thai nguy

t1 = 1 0 k = []k (5.1a)
hiem cua TTS n
t1 = 3

n
0n

= []n
n

(5.1b)

trong o: t1 - la ng suat tnh hay ng suat tng ng theo TB 1


u khuyet iem: TB 1, trong nhieu trng hp, khong phu hp vi
thc te. Th du trong th nghiem mau th chu ap lc giong nhau theo ba
phng (ap lc thuy tnh), du ap lc ln, vat lieu hau nh khong b pha hoai.
Nhng theo TB 1 th vat lieu se b pha hong khi ap lc at ti gii han ben
cua trng hp nen theo mot phng.
TB 1 khong ke en anh hng cua cac ng suat khac cho nen TB nay
ch ung oi vi TTS n.
2- Thuyet ben bien dang dai tng oi ln nhat (TB 2)
Nguyen nhan vat lieu b pha hong la do bien dang dai tng oi ln
nhat cua phan to TTS phc tap at en bien dang dai tng oi ln
nhat trang thai nguy hiem cua phan to TTS n.
Goi 1 : bien dang dai tng oi ln nhat cua phan to TTS phc tap

0k : bien dang dai tng oi trang thai nguy hiem cua phan to b keo
theo mot phng ( TTS n).
Theo nh luat Hooke, ta co:
1 =
0k =

1
[ 1 ( 2 + 3 )]
E

(a)

0k
E

(b)

Chng 5: Ly Thuyet Ben

http://www.ebook.edu.vn

GV: Le c Thanh
Thanhg Tuan

Ket hp (a) va (b), ke en he so an toan n


ieu kien ben theo TB 2:
1
[ 1 ( 2 + 3 )] 1 0 k
E
n E

(c)

hay t 2 = 1 ( 2 + 3 ) [ ]k

II

II

(5.2a)

oi vi trng hp bien dang


co ngan, ta co
t 2 = 3 ( 2 + 3 ) [ ]k

0k

0k

3
III

III

(5.2b)

H.5.1. TTS khoi H.5.2. Trang thai nguy


u khuyet iem: TB bien
hiem cua TTS n

dang dai tng oi tien bo hn so vi TB ng suat phap v co ke en anh


hng cua ca ba ng suat chnh. Thc nghiem cho thay TB nay ch phu hp
vi vat lieu don va ngay nay t c dung trong thc te.
3- Thuyet ben ng suat tiep ln nhat (TB 3)
Nguyen nhan vat lieu b pha hong la do ng suat tiep ln nhat cua
phan to TTS phc tap at en ng suat tiep ln nhat trang thai nguy
hiem cua phan to TTS n.
Goi: max - ng suat tiep ln nhat cua phan to TTS phc tap ;

0k - ng suat tiep ln nhat trang thai nguy hiem cua phan to b


keo theo mot phng ( TTS n).
n He so an toan
ieu kien ben theo TB 3: max

ok
n

(d)

trong o, theo (4.18), chng 4, ta co:


max =

1 3
;
2

(e) vao (d),

0k =

1 3
2

0k
2

(e)

0k
2n

ieu kien ben theo TB 3:


t3 = 1 3 []k

(5.3)

u khuyet iem: TB ng suat tiep ln nhat phu hp vi thc nghiem


hn nhieu so vi hai TB 1 va TB 2 . Tuy khong ke ti anh hng cua ng
suat chnh 2 song TB nay to ra kha thch hp vi vat lieu deo va ngay
nay c s dung nhieu trong tnh toan c kh va xay dng. No cung
phu hp vi ket qua mau th chu ap lc theo ba phng.

Chng 5: Ly Thuyet Ben

http://www.ebook.edu.vn

GV: Le c Thanh
Thanhg Tuan

4- Thuyet ben the nang bien oi hnh dang (TB 4)


Nguyen nhan vat lieu b pha hong la do the nang bien oi hnh dang
cua phan to TTS phc tap at en the nang bien oi hnh dang trang
thai nguy hiem cua phan to TTS n.
Goi: uhd - The nang bien oi
hnh dang cua phan to TTS
phc tap

II

II

(uhd)o - The nang bien


0k
1
0k
I
oi hnh dang trang thai nguy
I
3
hiem cua phan to b keo theo mot
III
III
phng ( TTS n).
H.5.1. TTS khoi H.5.2. Trang thai nguy
n He so an toan
hiem cua TTS n
ieu kien e phan to TTS
phc tap khong b pha hong la ben theo TB 4 la:
uhd < (uhd)o
Theo 4.5 ,chng 4, ta a co:
1 +
12 + 22 + 32 1 2 2 3 3 1
uhd =
3E
(uhd )o = 1 + 02k
3E

(g)

(h)

The (h) vao (g) , lay can bac hai cua hai ve , ke en he so an toan n
ieu kien ben theo TB 4:
12 + 22 + 23 1 2 2 3 31 []k

hay la:

t4 =

12 + 22 + 23 1 2 2 3 31 []k

(5.4)

trong o: t4 - la ng suat tng ng theo thuyet ben th t.


u khuyet iem: TB the nang bien oi hnh dang c dung pho
bien trong ky thuat v kha phu hp vi vat lieu deo. Ngay nay c s
dung nhieu trong tnh toan c kh va xay dng .

Chng 5: Ly Thuyet Ben

http://www.ebook.edu.vn

GV: Le c Thanh
Thanhg Tuan

CAC KET QUA AC BIET:


1- TTS phang ac biet (H.5.3):
Cac ng suat chnh :

1,3

= + 2 ;
2
2

a)

Theo TB ng suat tiep (5.3):


t3 = 1 3 =

2 = 0

H. 5.3

(5.5)

2 + 4 2 [ ]

Theo TB the nang bien oi hnh dang (5.4):


t4 =

hay:

12 + 22 + 23 1 3 21 3 2 []

(5.6)

2 + 3 2 []

2- TTS trt thuan tuy (H.5.4):


Cac ng suat chnh :

1 = 3 = | |; 2 = 0

Theo TB ng suat tiep:


t3 = 1 3 = 2 | | []

hay:

[ ]
| |
2

H.5.4

(5.7)

Theo TB the nang bien oi hnh dang:


t4 =

hay:

| |

32 []
[]
3

Chng 5: Ly Thuyet Ben

(5.8)

http://www.ebook.edu.vn

GV: Le c Thanh
Thanhg Tuan

5- Thuyet ben ve cac TTS gii han (TB 5 hay la TB Mohr)


TB Mohr c xay dng tren c s cac ket qua thc nghiem, khac vi
cac TB trc xay dng tren c s cac gia thuyet.
chng 4, ta a biet mot TTS khoi vi ba ng suat chnh 1, 2 va
3 co the bieu dien bang ba vong tron Morh 1, 2 va 3 vi ng knh tng
ng la 2 3 , 1 3 va 1 2 nh Hnh.4.22. Neu vat lieu trang thai
nguy hiem th nhng vong tron tng ng vi TTS nguy hiem c goi la
nhng vong tron Mohr gii han. Thc nghiem cho thay, ng suat phap 2 t
anh hng en s pha hoai cua vat lieu nen ta ch e y en vong tron Mohr
ln nhat goi la vong tron chnh xac nh bi ng knh 1 3.
ng bao

O
Cn

H. 5.5

Cac vong tron Mohr gii

han va ng cong gii han

Ck

H. 5.6

ng bao gii han


n gian hoa

Tien hanh th nghiem cho cac TTS khac nhau va tm trang thai gii han
tng ng cua chung, tren mat phang toa o , ta ve c mot ho cac
ng tron chnh gii han nh H.5.5. Neu ve ng bao nhng vong tron
o ta se thu c mot ng cong gii han, ng cong nay cat truc hoanh
iem tng ng vi trang thai co ba ng suat chnh la ng suat keo co gia
tr bang nhau. Gia thiet rang ng bao la duy nhat oi vi moi loai vat lieu,
ta nhan thay neu TTS nao bieu th bang mot vong tron chnh nam trong
ng bao th vat lieu am bao ben, vong tron chnh tiep xuc vi ng bao
th TTS o gii han ben con neu vong tron chnh cat qua ng bao th
vat lieu b pha hong.
Viec phai thc hien mot so lng ln cac th nghiem e xac nh cac
vong tron gii han va ve chnh xac ng cong gii han la khong n
gian.V vay, ngi ta thng ve gan ung ng bao bang cach da tren c
s hai vong tron gii han keo va nen theo mot phng vi ng knh tng
ng la [] k va []n. ay, e cho tien ta thay the cac ng suat nguy hiem

0 va 0n bang ky hieu ng suat cho phep []k va []n tc la a co ke ti he


Chng 5: Ly Thuyet Ben

http://www.ebook.edu.vn

GV: Le c Thanh
Thanhg Tuan

so an toan. ng bao c thay the bang ng thang tiep xuc vi hai


vong tron gii han nh tren H.5.6.
[]k

[]n
1

Cn

Ck

M1

N1

M
N

H. 5.7

Trang thai n g suat gii han va ng bao

Xet mot TTS khoi co vong tron Mohr ln nhat 1 va 3 tiep xuc vi
ng bao, nam gii han ve o ben. Tren H.5.7, vong tron nay c ve
bang ng net t. Sau ay, ta thiet lap lien he gia nhng ng suat chnh
1 va 3 vi cac ng suat cho phep []k va []n. T hnh ve ta co ty le thc:
NN1
MM1
=
KN1
KM1

Thay the cac tr so:


1
NN1 = ([[] n [] k ) ;

2
1
MM1 = (1 3 []k ) ;
2

1
([]n + []k )
2
1
([]k (1 + 3 ))
KM1 =
2
KN1 =

vao ty le thc tren, ta nhan c ieu kien gii han:


3 [ ] k
[ ] n [ ] k
= 1
[ ] n + [ ] k
[]k (1 + 3 )

hoac:

[]k
3 = [ ] k
[]n

Nh vay, ieu kien ben theo TB Mohr (TB 5) c viet la:

1 3 []k

(5.9a)
vi he so:

[ ] k
[ ] n

(5.9b)

Tuy bo qua anh hng cua ng suat chnh 2 va n gian hoa ng


cong gii han thanh ng thang, thuyet ben Mohr co u iem hn nhng
thuyet ben tren v no khong da vao gia thuyet nao ma can c trc tiep vao
trang thai gii han cua vat lieu. Thc te cho thay TB nay phu hp vi vat
lieu don, tuy nhien no cho ket qua chnh xac ch khi vong tron gii han cua
TTS ang xet nam trong khoang hai vong tron gii han keo va nen.

Chng 5: Ly Thuyet Ben

http://www.ebook.edu.vn

GV: Le c Thanh
Thanhg Tuan

5.3 VIEC AP DUNG CAC TB


Tren ay la nhng TB c dung tng oi pho bien. Viec ap dung TB
nay hay TB khac e giai quyet bai toan cu the phu thuoc vao loai vat lieu s
dung va TTS cua iem kiem tra.
oi vi TTS n, ngi ta dung TB 1 e kiem tra o ben.
oi vi TTS phc tap, neu la vat lieu don, ngi ta thng dung TB 5
(TB Mohr) hay TB 2, neu la vat lieu deo ngi ta dung TB 3 hay TB 4.
Hien nay, co nhieu TB mi c xay dng, tong quat hn va phu hp
hn vi ket qua thc nghiem. Tuy vay, nhng TB nay cung co nhng nhc
iem nhat nh nen cha c s dung rong rai.
Th du: Kiem tra ben phan to vat the TTS khoi nh tren H.5.8. ng
suat cho theo kN/cm2. Cho biet: [] = 16 kN/cm 2 .
Giai.

Chon he toa o nh tren H.5.8.

Theo quy c ta co:

x = -5 kN/cm , y = 6 kN/cm , zy = -yz = 4 kN/cm

x
z

z =0 , xz = zx =yx = xy =0
Mat vuong goc vi truc x la mat chnh vi ng
suat chnh

x = 5 kN/cm 2 .

H. 5.8

Hai ng suat chnh con lai

nam trong mat phang vuong goc vi ng suat chnh a cho va co gia tr
bang:
max =
min

Do o:

z + y
2

2
8 kN/cm 2
z y

+ 2zy = 3 5 =
2
2 kN/cm 2

1 = 8 kN/cm 2 ; 2 = 2 kN/cm 2 ; 3 = 5 kN/cm 2

Theo TB ng suat tiep:


t3 = 1 3 = 8 (5) = 13 kN/cm 2 < 16 kN/cm 2

Theo TB the nang bien oi hnh dang:


t4 =
=

12 + 22 + 23 1 2 1 3 2 3
82 + 22 + 52 ( 2) 8 8( 5) ( 2)( 5)

= 11,79 kN/cm 2 < 16 kN/cm 2

Nh vay, theo ca hai TB phan to nay am bao ben.

Chng 5: Ly Thuyet Ben

http://www.ebook.edu.vn

GV: Le c Thanh
Thanhg Tuan

BAI TAP CHNG 5

5.1 Khi nen vat lieu theo ba


z
phng cung vi tr so ng

y x
suat phap (H.5.1), ngi ta

thay vat lieu khong b pha


a)
hoai. Hay kiem tra ben oi
H. 5.2
H. 5.1
vi phan to tren bang TB
ng suat tiep ln nhat va TB the nang bien oi hnh dang ln nhat.

b)

5.2 Dung TB ng suat tiep ln nhat e tnh ap lc p ln nhat tac dung tren
khoi thep tren H.5.2. Khoi thep o c at kht vao trong khoi thep ln.
Cho E = 2.107 N/cm2; = 0,28;
[ ] = 16 kN/cm2.
5.3 Cho TTS nh H.5.3. Tnh ng
suat tng ng (ve trai cua cong
thc kiem tra ben) theo TB the nang
bien oi hnh dang va TB Mohr. Cho

a
a

8 kN/cm

H. 5.3
2

5.4 Cho TTS tai mot iem cua vat


the chu lc nh H.5.4:

Kiem tra o ben theo TB 3 va TB 4.

2 kN/cm2

ok/on = 0,25.

1 = 20 kN/cm2; 2 = 40 kN/cm2;
3 = 80 kN/cm2

4 kN/cm2

p
1

H. 5.4

H. 5.5

Biet [] = 120 kN/cm2.


5.5 Mot tru tron bang thep (= 0,3) at kht gia hai tng cng nh H.5.5.
Phan gia cua tru chu ap lc p phan bo eu. Tnh ng suat tng ng
theo TB 4 phan gia va phan au cua hnh tru.

Chng 5: Ly Thuyet Ben

http://www.ebook.edu.vn

GV: Le c Thanh

Chng 6

AC TRNG HNH HOC CUA MAT CAT NGANG


6.1 KHAI NIEM
chng 3, khi tnh o ben cua thanh chu keo (nen) ung tam, ta thay
ng suat trong thanh ch phu thuoc vao o ln cua dien tch mat cat ngang
F (mat cat F, dien tch F). Trong nhng trng hp khac, nh thanh chu
uon, xoan th ng suat trong thanh khong ch phu thuoc vao dien tch F ma
con phu thuoc vao hnh dang, cach bo tr mat cat ngha con nhng yeu to
khac ma ngi ta goi chung la ac trng hnh hoc cua mat cat ngang.

P
P

a)

b)

H.6.1. Dam chu uon


a) Tiet dien ng; b) Tiet dien nam ngang

Xet thanh chu uon trong hai trng hp mat cat at khac nhau nh tren
H.6.1. Bang trc giac, de dang nhan thay trng hp a), thanh chu lc tot
hn trng hp b), tuy rang trong trong hai trng hp dien tch cua mat cat
ngang thanh van nh nhau. Nh vay, kha nang chu lc cua thanh con phu
thuoc vao cach sap at va v tr mat cat ngang oi vi phng tac dung cua
lc.(ng suat nho 04 lan o vong nho 16 lan ). Cho nen s chu lc khong
nhng phu thuoc F, ma can phai nghien cu cac ac trng hnh hoc khac
cua mat cat ngang e tnh toan o ben, o cng, o on nh va thiet ke mat
cat cua thanh cho hp ly.
6.2 MOMEN TNH - TRONG TAM
_________________________________________________________________
Chng 6: ac trng hnh hoc cua mat cat ngang http://www.ebook.edu.vn
1

GV: Le c Thanh

Xet mot hnh phang bieu dien mat


cat ngang F ( mat cat F ) nh tren H.6.2.
Lap mot he toa o vuong goc Oxy trong
mat phang cua mat cat. M(x,y) la mot
iem bat ky tren hnh. Lay chung quanh
M mot dien tch vi phan dF.

yo

yo
y

C
xC

dF
xo

xo

yC

x
Momen tnh cua mat cat F oi vi
x
O
truc x (hay y) la tch phan:
H.6.2 Mat cat F va trong tam C
S x = ydF , S y = xdF
(6.1)
F

v x, y co the am hoac dng nen


momen tnh co the co tr so am hoac dng.
Th nguyen cua momen tnh la [(chieu dai)3].
Truc trung tam la truc co momen tnh cua mat cat F oi vi truc o
bang khong.
Trong tam la giao iem cua hai truc trung tam.
Momen tnh oi vi mot truc i qua trong tam bang khong.
Cach xac nh trong tam C cua mat cat F:
Dng he truc x o Cy o song song vi he truc xOy ban au (H.6.2). Ta co
x = xC + xo ;

y = yC + yo , vi C(x c ,y c )

Thay vao (6.1),


S x = ( yC + yo )dF = yC dF + yo dF = yC F + S xo
F

v truc xo la truc trung tam nen S xo = 0 ,


S x = yC F , va

: S y = xC F

(6.2)

T (6.2)
xC =

Sy
F

yC =

Ket luan: Toa o trong tam

Sx
F

C( xC , yC )

(6.3)
c xac nh trong he truc xOy ban

au theo momen tnh Sx , Sy va dien tch F theo (6.4).


Ngc lai, neu biet trc toa o trong tam, co the s dung (6.2), (6.3)
e xac nh cac momen tnh.

Nhan xet 1:
_________________________________________________________________
Chng 6: ac trng hnh hoc cua mat cat ngang http://www.ebook.edu.vn
2

GV: Le c Thanh

Mat cat co truc oi xng, trong tam nam tren truc nay v momen tnh
oi vi truc oi xng bang khong (H.6.3a,b).
Mat cat co hai truc oi xng, trong tam nam giao iem hai truc oi
xng (H.6.3c).
y

a)

b)

c)

H.6.3 Mat cat co truc oi xng

Thc te, co the gap nhng mat cat ngang co hnh dang phc tap c
ghep t nhieu hnh n gian.
Tnh chat: momen tnh cua hnh phc tap bang tong momen tnh cua
cac hnh n gian.
Vi nhng hnh n gian nh ch nhat, tron, tam giac hoac mat cat cac
loai thep nh hnh I, U, V, L ta a biet trc (hoac co the tra theo cac bang
trong phan phu luc ) dien tch, v tr trong tam, t o de dang tnh c
momen tnh cua hnh phc tap gom n hnh n gian:
S x = F1 y1 + F2 y2 + ... + Fn yn =

F y
i

S y = F1 x1 + F2 x2 + ... + Fn xn =

Fx

(6.4)

i i

trong o: Fi , xi , yi - dien tch va toa o trong tam cua hnh n gian th i,


n - so hnh n gian.
Toa o trong tam cua mot hnh phc tap trong he toa o xy.
n

xC =

Sy
F

Fx
i =1
n

F
i =1

; yC =

Sx
=
F

Fy
i =1
n

F
i =1

(6.5)

_________________________________________________________________
Chng 6: ac trng hnh hoc cua mat cat ngang http://www.ebook.edu.vn
3

GV: Le c Thanh
y

Th du 6-1 Xac nh trong tam


mat cat ch L ch gom hai hnh
ch nhat nh tren H.6.4. Toa o

x1

trong tam C cua hnh tren la:


xF +x F
xC =
= 1 1 2 2 ;
F
F1 + F2
S
y F + y2 F2
yC = x = 1 1
F
F1 + F2

xC

Sy

F
C

y1

yC

Th du 6.2 Mot mat cat thanh


ghep, gom thep ch so hieu

x C
O

y2

x2

H . 6 . 4 T r o n g t a m h n h p h c t a p

No55, thep ch [ so hieu No27,


va thep ch nhat 15 1,2 cm (H.6.5). Xac nh trong tam C cua mat cat.
Giai.
Tra bang (OCT 8239-89) so lieu sau:
- oi vi thep ch No55:
h2 = 55 cm
t = 1,65 cm
F2 = 118 cm2

yY
I

x
IoN55

x
x

C2

yC

C
y3
o

[ N 27

z3
C3

F1 = 15 cm x 1,2 cm = 18 cm2

y1

II

- oi vi thep ch [ N 27:
h3 = 27 cm
F3 = 35,2 cm2
z3 = 2,47 cm
- Hnh ch nhat:

150 12 mm

C1

III

H.6.5. Trong tam C cua hnh ghep

Chon he truc toa o xy qua goc C2 toa o trong tam cua ba hnh tren la:
y1 =

55 1,2
55
+
= 28,1 cm ; y2 = 0 ; y3 =
+ 2,47 = 29,97 cm
2
2
2

Dien tch va momen tnh cua toan mat cat la:


F = F1 + F2 + F3 = 18 + 118 + 35,2 = 171,2 cm2

S x = y1 F1 + y2 F2 + y3 F3 = (28,1)(18) + 0 (29,97 )(35,2 ) = 549,144cm3

v y la truc oi xng, trong tam C se nam tren truc nay.


Toa o iem C la:

xC = 0 ; yC =

S x 549,144
=
32 cm
F
171,2

Dau () cho thay trong tam C nam pha di truc x.


Chu y rang, truc x co the chon tuy y song th du nay ta at truc x i
qua trong tam C2 cua mat cat ch cho tien tnh toan.
_________________________________________________________________
Chng 6: ac trng hnh hoc cua mat cat ngang http://www.ebook.edu.vn
4

GV: Le c Thanh

Th du 6.3. Cho mat cat hnh ch U .Tm trong tam C


Chon he truc x,y nh hnh ve, truc x qua ay mat cat (truc y la truc oi
xng, C nam tren truc y)
yC =

S x 24 4 2 + 2(4 12 10)
=
= 6cm
(24 4) + 2(4 12)
F

4cm

Hay :
yC =

S x1 _ S x 2
F1 _ F2

(24 16 8) _(16 12 10)


= 6cm
(24 16) _(16 12)

y
4cm
16cm
12cm

4cm

1
H. 6.12

6.3 MOMEN QUAN TNH- HE TRUC QUAN TNH CHNH TRUNG TAM
1- Momen quan tnh (MMQT)
Momen quan tnh oc cc

( MMQT oi vi iem) cua mat cat F


oi vi iem O c nh ngha la
bieu thc tch phan:

dF
F

(6.6)

J = dF
2

vi : - khoang cach t iem M en


goc toa o O,
Momen quan tnh oi vi truc

H. 6.6 Hnh phan g F

y va x cua mat cat F c nh ngha:


J y = x 2 dF ; J x = y 2 dF
F

(6.7)

Momen quan tnh ly tam cua mat cat F oi vi he truc x,y c nh


ngha:
(6.8)

J xy = xydF
F

T nh ngha cac momen quan tnh, ta nhan thay:


- MMQT co th nguyen la [chieu dai]4
- Jx , Jy , Jp > 0
- MMQT ly tam Jxy co the dng, am hoac bang khong.
- V

2 = x2 + y2

nen

J = Jx + Jy

(6.9)

_________________________________________________________________
Chng 6: ac trng hnh hoc cua mat cat ngang http://www.ebook.edu.vn
5

GV: Le c Thanh

Chng 7

UON PHANG THANH THANG


7.1 KHAI NIEM CHUNG
Thanh chu uon la thanh co

P2

truc b uon cong doi tac dung


cua ngoai lc. Thanh co truc nam ngang
chu uon c goi la dam.
(Thanh co truc thang ng goi la cot)

P3
02

P1
P5
01

P4

Ngoai lc: Lc tap trung P,


H.7.1. Tai trong tac dung len dam
lc phan bo q tac dung vuong goc
vi truc dam hay momen (ngau lc)
M nam trong mat phang cha truc dam (H.7.1).
Mat phang tai trong: Mat phang ( ) cha ngoai lc va truc dam.
ng tai trong: Giao tuyen cua mat phang tai trong vi mat cat ngang.
Gii han bai toan:
+ Ch khao sat cac thanh mat cat ngang co t nhat mot truc oi xng.
Truc oi xng nay va truc thanh hp thanh mat phang oi xng.
Tai trong nam trong mat phang oi xng.
Mat phang tai trong trung mat phang oi xng,
ng tai trong cung la truc oi xng cua mat cat ngang
Truc dam sau khi b cong van nam trong mat phang ( )c goi la
uon phang.
+ Mat cat ngang dam co chieu rong be so vi chieu cao.
H.7.3 ,7.4,7.5 : gii thieu mot so loai dam n gian thng gap

_________________________________________________________________
http://www.ebook.edu.vn 1
Chng 7: Uon phang thanh thang

GV: Le c Thanh

b)
a

L
c)

a)

H.7.3. Cac loai dam: a) Dam n gian


b) Dam chen kep; c) Dam co au mut tha
P

a)

L-2a

P
b)

+
_

a)

P
c)

M
Pa

Pa

H.7.4. Dam vi vung gia chu


uon thuan tuy

b)

H.7.5. Dam chu uon thuan tuy

Noi lc: Tuy theo ngoai lc tac dung ma tren mat cat ngang dam co cac
noi lc la lc cat Qy va momen uon Mx .
Phan loai:
Uon thuan tuy phang: Noi lc ch co momen uon Mx=hang so.
Uon ngang phang : Noi lc gom lc cat Qy va momen uon Mx
Dam H.7.4 co oan gia CD chu uon thuan tuy, dam H. 7.5 chu
uon thuan tuy. oan dam AC va DB cua dam H.7.4 chu uon ngang
phang.

_________________________________________________________________
http://www.ebook.edu.vn 2
Chng 7: Uon phang thanh thang

GV: Le c Thanh

7.2 UON THUAN TUY PHANG


7.2.1 nh ngha: Thanh chu uon thuan tuy phang khi tren moi mat cat
ngang ch co mot noi lc Mx.
Dau cua Mx : Mx > 0 khi cang (keo) th di ( th y > 0 ) cua dam
7.2.2 Tnh ng suat tren mat cat ngang:
1. Th nghiem va quan sat bien dang:

H.7.6. a) Thanh trc khi bien dang


b) Sau bien dang; c) Mat cat ngang sau bien dang

H. 7.6 a) Thanh trc khi bien dang


b) Sau bien dang; c) Mat cat ngang sau bien dang
Ke len mat ngoai mot thanh thang chu uon nh H.7.6a, nhng ng
song song vi truc thanh tng trng cho cac th doc va nhng ng
vuong goc vi truc thanh tng trng cho cac mat cat ngang; cac ng
nay tao thanh cac li o vuong (H.7.6a).
Sau khi bien dang (H.7.6b), truc thanh b cong, cac ng thang song
song vi truc thanh thanh cac ng cong song song vi truc thanh; nhng
ng vuong goc vi truc thanh van con vuong goc vi truc thanh, ngha la
cac goc vuong c bao toan trong qua trnh bien dang.
Ngoai ra, neu quan sat thanh th thay cac th ben di dan ra(b keo)
va cac th ben tren co lai (b nen). Nh the, t th b dan sang th b co se
ton tai cac th ma chieu dai khong thay oi trong qua trnh bien dang, goi la
th trung hoa. Cac th trung hoa tao thanh lp trung hoa. Giao tuyen cua
lp trung hoa vi mat cat ngang tao thanh ng trung hoa. V mat cat
ngang co chieu rong be nen ng trung hoa xem nh thang (H.7.6.c)
_________________________________________________________________
http://www.ebook.edu.vn 3
Chng 7: Uon phang thanh thang

GV: Le c Thanh

Mat phang tai trong

Phan b nen

Lp trung hoa

z
x

Phan b keo

ng trung hoa
ng tai trong

Sau bien dang cac mat cat ngang 1-1 va 2-2 ban au cach nhau mot oan
vi phan dz se cat nhau tai tam cong O (H.7.7b) va hp thanh mot goc d.
Goi la ban knh cong cua th trung hoa, tc khoang cach t O en th
trung hoa. o dan dai tng oi cua mot th ab cach th trung hoa mot
khoang cach y cho bi:
z =

ab 010 2 ( + y ) d dz ( + y ) d d y
=
=
= = y
01 0 2
dz
d

(a)

trong o: - la o cong cua dam.


He thc nay chng to bien dang doc truc dam t le vi o cong va bien
thien tuyen tnh vi khoang cach y t th trung hoa
O

1
O1
M

a
1

2
O2
y
dz

b
2

a) Truc bie n dang

O1

O2
b

b) Sau bien da ng

H.7.7 oan dam vi phan dz

_________________________________________________________________
http://www.ebook.edu.vn 4
Chng 7: Uon phang thanh thang

GV: Le c Thanh

2. Thiet lap cong thc tnh ng suat:


Moi th doc cua dam ch chu keo hoac
nen (cac iem bat ky tren mat
cat ngang trang thai ng suat n).
nh luat Hooke ng vi trang thai
ng suat n cho ta:
z = E z = Ey
(b)
ng suat phap tac dung tren
mat cat ngang bien thien bac nhat

Mx
0

ng trung hoa

y
z

dF

H.7.8. ng suat phap va mo


men uon tren mat cat
ngang cua dam chu uon

vi khoang cach y t th trung hoa.


Xet hp lc cua cac ng suat phap tren toan mat cat ngang.
+ Lien he gia z va Nz
F z dF = F EyF = 0 (nh ngha N z =0) (c)

V o cong va moun an hoi E la hang so nen co the em ra ngoai


dau tch phan,

ydF = 0

(d)

(d) cho thay momen tnh cua dien tch mat cat ngang oi vi truc trung
hoa x bang khong truc trung hoa x i qua trong tam mat cat ngang.
Tnh chat nay cho phep xac nh truc trung hoa cua bat ky mat cat
ngang nao. Neu truc y la truc oi xng, th he truc (x,y) chnh la he truc
quan tnh chnh trung tam.
+ Lien he gia z va Mx
trong o:

Mx =

Jx =

z ydF = E y 2 dF = EJx
F

(e)

y 2 dF

(g)

la momen quan tnh cua mat cat ngang oi vi truc trung hoa x.
Bieu thc (e) c viet lai nh sau:
=

Mx
EJ x

(7.1)

EJx goi la o cng uon cua dam.


The(7.1) vao (b) Cong thc tnh ng suat phap tai mot iem tren mat cat
ngang dam:

z =

Mx
y
Jx

(7.2)

ng suat bien thien bac nhat theo tung o y.va y la khoang cach cua iem
tnh ng suat ke t truc trung hoa x .(M x va y mang dau ai so)
_________________________________________________________________
http://www.ebook.edu.vn 5
Chng 7: Uon phang thanh thang

GV: Le c Thanh

Cong thc ky thuat:


Neu momen uon dng, dam b cang ( b keo ) th di, cac th tren
b nen . Ket qua ngc lai neu momen uon am. Do vay trong thc hanh, ta
co the s dung cong thc ky thuat e tnh ng suat,
M
z = x | y |
(7.3)
Jx
ta se lay: dau (+) neu Mx gay keo tai iem can tnh ng suat.
dau () neu Mx gay nen tai iem can tnh ng suat.
.
7.2.3 Bieu o ng suat phap - ng suat phap cc tr:
Bieu o ng suat phap:
+Nhng iem cang xa truc trung hoa co tr so ng suat cang ln.
+Nhng iem cung co khoang cach ti th trung hoa se co cung tr so
ng suat phap.
Bieu o phan bo ng suat phap la o th bieu dien gia tr cac ng suat
tai cac iem tren mat cat ngang.
*Trng hp mat cat ngang co hai truc oi xng (Hnh tron, ch nhat..) cho
bi H.7.9
*Trng hp mat cat ngang ch co mot truc oi xng (ch I,U) cho bi
H.7.10.
Dau (+) ch ng suat keo.
Dau (-) ch ng suat nen.

_________________________________________________________________
http://www.ebook.edu.vn 6
Chng 7: Uon phang thanh thang

GV: Le c Thanh

H. 7.9 Bieu o ng suat phap cho cac mat cat co hai truc oi xng

H. 7.10 Bieu o ng suat phap cho cac mat cat co mot truc oi xng
ng suat phap cc tr:
Tnh ng suat phap khi keo va khi nen ln nhat tren mat cat ngang dam
nhng iem xa ng trung hoa nhat.
k
n
Goi ymax , ymax lan lt la khoang cach th chu keo va th chu nen

xa ng trung hoa nhat. Khi o ng suat chu keo ln nhat max va ng


suat chu nen ln nhat min se tnh bi cac cong thc:
M
M
k
max = x ymax
= kx
(7.4a)
Jx
Wx

min =

Mx

n
=
ymax

Mx

Jx
Wxn
J
J
Wxk = k x ; Wxn = n x
ymax '
ymax

vi:

(7.4b)
(7.5)

n
k
Cac ai lng Wx va Wx goi la cac suat tiet dien hoac momen chong uon

cua mat cat ngang.


Trng hp at biet: Neu truc x (truc trung hoa) cung la truc oi xng
(mat cat ch nhat, tron, ,) th:
k
n
ymax
= ymax
=

khi o:

h
2

Wxk = Wxn = Wx =

2J x
h

(7.6)

va ng suat nen va keo cc ai co tr so bang nhau:


max = min =

Mx
Wx

(7.7)

Mat cat ngang hnh ch nhat vi be rong b va chieu cao h :


_________________________________________________________________
http://www.ebook.edu.vn 7
Chng 7: Uon phang thanh thang

GV: Le c Thanh

Jx =

bh3
bh 2
; Wx =
12
6

(7.8)

Mat cat ngang hnh tron:


d 4
Jx =

64

0,05d 4 ; Wx =

d 3
32

0,1d 3

(7.9)

Mat cat ngang hnh vanh khan : ng knh ngoai D, trong, d


Jx =

D 4
64

(1 4 ) ; Wx =

D 3
32

(1 4 )

vi

= d/ D

Mat cat ngang hnh , C: Tra bang thep nh hnh.


Y ngha vat ly cua momen chong uon: khi momen chong uon cang ln
dam chu c momen uon cang ln.
7.2.4 ieu kien ben- Ba bai toan c ban
ieu kien ben:
+ Dam bang vat lieu don: [] k [] n
min [] n
max [] k
+ Dam bang vat lieu deo: [] k = [] n = []
max z []
Ba bai toan c ban:

(7.10a)
(7.10b)

+Bai toan kiem tra ben,(ay la bai toan tham ke.)


+Bai toan chon kch thc mat cat ngang,(bai toan thiet ke).
+Bai toan chon tai trong cho phep.(bai toan sa cha,nang cap)
Bai toan c ban 1: Kiem tra ben- Kiem tra thanh chu lc co am bao
o ben hay khong. Dung (7.10a) hay (7.10b) e kiem tra.
Th du 7.1 Tren mat cat ngang cua mot dam ch T ngc (H.7.11), momen
uon Mx = 7200 Nm. Dam lam bang vat lieu co ng suat cho phep khi keo
va nen khac nhau:
`
[ ]k = 20 MN/m2; []n = 30 MN/m2 `
Mx
125
`
Kiem tra ben biet rang: Jx = 5312,5 cm4
O
Giai.
x

Ta co:

ykmax = 75 mm= 7,5.102 m


y

max

= 125 mm = 12,5.10 m

75

Dam ch T chu uon


_________________________________________________________________
http://www.ebook.edu.vn 8
Chng 7: Uon phang thanh thang
H. 7.11

GV: Le c Thanh

Wxk =

5312,5 10 8
Jx
=
= 708,3 10 6 m 3
k
7,5 10 2
ymax

Wxn =

5312,5 108
Jx
=
= 425 10 6 m3
2
n
12,5 10
ymax

max =

Mx
7200
=
= 10,20 106 N/m 2 = 10,20 MN/m 2 < [ k ]
708,3 10 6
Wxk

min =

Mx
7200
=
= 17 106 N/m 2 = 17 MN/m 2 < [ n ]
n
Wx
425 10 6

vay dam u ben.


Bai toan c ban 2: Chon kch thc mat cat ngang sao cho dam thoa ieu
kien ben.
T ieu kien ben tong quat (7.10a,b) momen chong uon va kch thc
cua mat cat ngang se c xac nh.
Th du 7.2 Cho dam chu lc nh H.7.12.
Dam lam bang hai thep ch , Chon so
hieu cua thep ch  e dam thoa ieu kien
ben. Biet [ ] = 16 kN/cm2.

M = 60 KNm



Giai.
Dam chu uon thuan tuy; tren moi mat cat

H.7.12

ngang cua dam co momen uon Mx=60


kNm.
Ap dung cong thc (7.7) va (7.10b) ta c:

M = 60 kNm

Wx

M max
60.100
=
= 375 cm 3
[]
16

Tra bang thep hnh ta chon 2  20 co Wx = 2 184 = 368 cm3.


Kiem tra lai ieu kien ben ta co:
max =

M max
60.100
=
= 16,3 kN/cm2
368
Wx

sai so tng oi: 16,3 16 100%


16

= 1,9% ;

vay dam u ben. Chon 2  20

Bai toan c ban 3: nh tai trong cho phep [P] e dam thoa ieu kien
ben.
Th du 7.3 Mot dam bang gang co mat cat ngang nh H.7.13. Xac nh tr
so momen uon cho phep(momen co chieu nh hnh ve). Biet:
[ ] = 1,5 kN/cm2 .
Hoi vi tr so momen uon cho phep o, ng suat nen ln nhat trong
dam la bao nhieu?
Cho biet Jx = 25470 cm4
_________________________________________________________________
http://www.ebook.edu.vn 9
Chng 7: Uon phang thanh thang

GV: Le c Thanh

Giai.
T ieu kien ben max =

[M x ] = [ k ]

Mx
Jx

k
=
ymax

Mx

[] k

Wxk

Jx
25470
= 1,5
= 3537,5 kNcm
k
ymax
10,8

192mm
x
108mm

Tng ng ta co:
min =

[M x ] y n

max

Jx

3537,5
19,2 = 2,67 kN/cm 2
25470

H.7.13

7.2.5 Hnh dang hp ly cua mat cat ngang.


Hnh dang hp ly la sao cho kha nang chu lc cua dam la ln nhat
nhng ong thi t ton vat lieu nhat. ieu kien:
max =

Mx
Jx

y max =

Lap t so cac ng suat :


- Neu vat lieu don: < 1 v :

min

y max
n

y max

Mx
Jx

[ ]k

y max =

=
k
n
nen y max p y max

Ta chon mat cat ngang khong oi xng qua truc trung hoa.
k
n
- Neu vat lieu deo: =1 nen y max = y max
Ta chon mat cat ngang oi xng qua truc trung hoa.
Theo bieu o ng suat ta thay cang gan truc trung hoa ng suat cang nho,
nen tai o vat lieu lam viec t hn nhng iem xa truc trung hoa, v vay
thng cau tao hnh dang mat cat sao cho vat lieu xa truc trung hoa . v du
hnh ch I,U,vanh khan ,hnh rong

_________________________________________________________________
http://www.ebook.edu.vn 10
Chng 7: Uon phang thanh thang

GV: Le c Thanh

7.3 UON NGANG PHANG


7.3.1 nh ngha- Dam goi la chu uon ngang phang khi tren mat cat
ngang co 2 noi lc la: momen uon Mx va lc cat Qy ( H 7.14).
1
1

P
Qy

Mx

Qy

Mx

PL

H.7.14. So o dam
chu uon ngang

H.7.15 Mat cat ngang dam


chu uon ngang phang

7.3.2 Cac thanh phan ng suat:


1- Th nghiem va quan sat bien dang
Ke nhng ng song song va vuong goc vi truc thanh (H.7.16a).
Sau bien dang cac goc vuong khong con vuong ( H.7.16b).
1

yz

a)

b)

dz

zy

c)
H. 7.16. a) Thanh trc bien dang
b) Thanh sau bien dang
c) Trang thai ng suat phang

2- Trang thai ng suat:


Khac vi trng hp uon thuan tuy, ngoai ng suat phap z do momen
Mx gay ra con co ng suat tiep zy do lc cat Qy gay ra. Trang thai ng suat
cua mot phan to co cac mat song song cac truc toa o bieu dien nh hnh
7.15 va 7.16c
_________________________________________________________________
http://www.ebook.edu.vn 11
Chng 7: Uon phang thanh thang

GV: Le c Thanh

3. Cong thc tnh ng suat phap:


Chap nhan vi sai so khong ln dung cong thc (7.2 ) e tnh ng suat
phap trong thanh chu uon ngang phang.(Ly thuyet an hoi a chng minh)
z =

Mx
y
Jx

(7.2 )

4. Cong thc tnh ng suat tiep:


Gia thiet:
- Mat cat ngang dam co chieu rong be so vi chieu cao.
- ng suat tiep phan bo eu theo be rong cua mat cat va cung chieu
vi lc cat (ngha la moi iem nam cach eu ng trung hoa th co cung tr
so ng suat tiep).
dz
Mx

dz
Q y1
Mx

M x + dM x

Q y1

y
G

02

01
X

01
Q y2

z1

yz

zy

Ta xac nh quy luat phan bo ng suat tiep doc theo chieu cao cua mat cat
ngang.
Xet oan dam gii han bi 2 mat cat 1-1 va 2-2 cach nhau dz (H.7.17a).
e khao sat ng suat tiep tai iem K cach ng trung hoa x mot khoang y,
ta dung mat cat i qua K vuong goc vi lc cat.
Xet can bang cua phan di ABCDEFGH ( H.7.17b)
Theo cac gia thiet a neu, cac ng suat tiep zy thang ng co phng song
song vi lc cat th phan bo eu tren mat thang ng ABCD. Ngoai ra theo
nh luat oi ng cua ng suat tiep , tren mat vuong goc vi mat cat ngang
ABFE cung co ng suat tiep yz co gia tr bang vi zy ( H.7.17b).
_________________________________________________________________
http://www.ebook.edu.vn 12
Chng 7: Uon phang thanh thang

GV: Le c Thanh

Nh vay, ton tai ng suat tiep theo phng ngang gia cac lp song
song vi truc dam cung nh cac ng suat tiep thang ng tren cac mat cat
ngang cua dam. Tai mot iem, cac ng suat nay co gia tr bang nhau.
Phng trnh can bang theo phng z doc truc thanh cho:
N1 N 2 + T = 0
(a)
trong o: N1 - la hp cua cac lc tac dung tren mat 1-1 c tnh bi:
N1 =

Fc

z1dF =

M
ydF
Fc J
x

(b)

N2 - la hp cua cac lc tac dung tren mat 2-2 c tnh bi:


N2 =

Fc

z 2 dF =

Fc

M x + dM x
ydF
Jx

(c)

T - la hp cua cac lc tac dung tren mat tren ABEF cua phan t:
T = yz b c dz
(d)
Thay (b), (c), (d) vao (a)

Fc

Mx
M x + dM x
ydF
ydF + yzb c dz = 0
Fc
Jx
Jx

zy = yz =

(e)

dM x 1

ydF
dz J xb c Fc

(f)

thay Qy = dMx/dz ta c:
zy = yz =

at:

Qy
J xb c

Fc

S = ydF
c
x

(g)

ydF

Fc

zy = yz =

Q y S xc
J xb c

(7.11)

Cong thc (7.11) goi la cong thc D.I. Zhuravski


S cx :momen tnh cua phan dien tch b cat (F c )oi vi truc trung hoa.
bc: be rong tiet dien cat.
J x :Momen quan tnh cua tiet dien.
Q y : Lc cat tai tiet dien ang tnh.

_________________________________________________________________
http://www.ebook.edu.vn 13
Chng 7: Uon phang thanh thang

GV: Le c Thanh

5-Phan bo ng suat tiep tren mot so mat cat thng gap:


+ Mat cat ngang ch nhat (H.7.18):

m1

max

h/2

M+dM

M
p

p1

x
y

h/2
n1

Fc

a)

y
b)

c)

H.7.18. Phan bo cua ng suat tiep tren mat cat


ngang ch nhat

Dien tch b cat Fc la hnh ch nhat , nen


h / 2 y b h2
h

2
S = b y y +
= y
2 2 4
2

(i)

c
x

Thay vao (7.11)

zy =

Qy h 2

y 2
2J x 4

(7.12)

He thc nay chng to ng suat tiep trong dam tiet dien ch nhat bien
thien theo quy luat bac hai theo khoang cach y t truc trung hoa va bieu o
theo chieu cao cua dam co dang nh tren H.7.18c.
zy = 0 khi

y = h / 2

( cac iem bien tren, di cua mat cat)

zy = max khi y= 0 ( cac iem tren truc trung hoa):

max =

Qy h 2
8J x

3 Qy
2 F

(7.13)

trong o: F = bh - la dien tch cua mat cat ngang.


Th du 7.4 Tnh ng suat phap va ng suat tiep cc ai tren dam co mat
q
cat ngang hnh ch nhat bxh (H. 7.19)
h p
Cho biet: q = 12 kN/m , l = 4 m; h = 27 cm, b = 18 cm, ng suat cho phe
l
[ ] = 1,1 kN/cm2, [] = 0,22 kN/cm2.
b
Giai.
Momen cc ai gia dam:
M max =

ql
12 4 4 10
=
= 2400 kNcm
8
8
2

Q ql/2

ql/2

_________________________________________________________________
ql2/8
http://www.ebook.edu.vn
14
Chng 7: Uon phang thanh thang

GV: Le c Thanh

Lc cat cc ai hai goi ta:


Qmax =

12 4
ql
=
= 24 kN
2
2

ng suat cc ai:
max =
max =

M max
2400 6
=
= 1,095 kN/cm2 < 1,1 kN/cm2
W
18 27 2

3Qmax
3 24
= 0,075 kN/cm 2 < 0,22 kN/cm 2
=
2bh
2 18 27

H.7.19

+ Mat cat ngang hnh tron va hnh vanh khan (H.7.20)

Qy

max

b(y)

b()
b(y)
C
a)

b)

c)

H.7.20. ng suat tiep tren mat cat ngang hnh tron

Khi dam co mat cat ngang la hnh tron, ng suat tiep tren mat cat ngang
khong con song song vi lc cat na. Neu khong co lc tac dung tren mat
ngoai cua dam, ng suat tiep tren hai dien tch vi phan tai cac iem 1 va 2
tren vung sat chu vi cua mat cat ngang phai hng theo phng tiep tuyen
vi chu vi nay (H.7.20a).
Cac tiep tuyen nay co phng ong quy tai iem C tren phng tac
dung cua lc cat. Bi v lc cat Qy la hp cua cac ng suat tiep (H.7.20),
nen cac ng suat tiep tai cac dien tch vi phan tai 3 va 4 co cung khoang
cach y ti truc trung hoa se co phng i ngang iem C.
Moi ng suat tiep nay co the phan thanh hai thanh phan: thanh phan
thang ng 1, va nam ngang 2. Cac thanh phan nam ngang tac dung tren
hai phan trai va phai se t can bang nhau do tnh oi xng, trong khi cac
thanh phan thang ng hp lai thanh lc cat Qy.
_________________________________________________________________
http://www.ebook.edu.vn 15
Chng 7: Uon phang thanh thang

GV: Le c Thanh

Nh vay, trong dam co mat cat ngang tron, thanh phan 1 se ong vai
tro cua trong dam co mat cat ngang hnh ch nhat.
Momen tnh cua phan dien tch gii han bi bien di mat cat ngang va
mat cat song song vi mat trung hoa khoang cach y t truc trung hoa x
cho bi:
(j)
S xc = dF = b( )d
F
F
c

ta co:

(k)

bc = b( ) = 2 R2 y2

trong o: R - la ban knh cua hnh tron mat cat ngang.


Do vay:

Sxc =

R 2 y2 .d =

y2

2 2
R y2
3

(l)

3/2

va thanh phan ng suat tiep theo phng thang ng co tr so:


zy =

zy = 0 khi

4 Qy
y2
1 2
3 F
R

y = h / 2

(7.14)

( cac iem bien tren, di cua mat cat)

zy = max khi y= 0 ( cac iem tren truc trung hoa):

max =

4 Qy
,( F:dien tch hnh tron)
3 F

(7.15)

+ Mat cat ngang hnh ch , hay ch T

h1/2

h1/2

a)

max

h1/2

h1/2

b)

H.7.17. ng suat tiep trong long cua dam ch I


_________________________________________________________________
http://www.ebook.edu.vn 16
Chng 7: Uon phang thanh thang

GV: Le c Thanh

Cac mat cat ngang ch  hay ch T c xem nh cau tao bi cac hnh
ch nhat ghep nen vi mc o chnh xac nhat nh, cac cong thc dung cho
dam mat cat ngang ch nhat cung dung c cho cac loai mat cat nay. ng
suat tiep c tnh bang cong thc Zhuravski :

Q y Sxc
I x bc

zy trong ban bung: Xet iem co tung o y ( H.7.21a)


bc chnh la be rong ban bung: bc = d
Sxc la momen tnh cua phan dien tch gach cheo di mc ef oi vi
truc trung hoa x. Sxc co the tnh bang momen tnh cua na hnh ( trong
bang ghi la Sx ) tr momen tnh cua phan dien tch (y x d)
y
S xc = S x (d y )
2

(o)

ng suat tiep zy trong ban bung cua dam ch la


zy =

Qy
y2
S x (d )
Jxd
2

(p)

(p) ch rang ng suat tiep trong ban bung cua dam ch I bien thien
theo quy luat parabol doc theo chieu cao cua dam.
zy = max khi y = 0 ( cac iem tren truc trung hoa):

max =

Qy
J xd

zy = 1 khi y =

va:

1 =

(7.17)

Sx
h
2

t = h1 ( iem tiep giap gia bung va canh). 1 kha ln

Qy
h2
S x d 1
J xd
2

(7.18)

zy trong ban canh: Xet mot iem trong ban canh, be rong cat bc = b kha
ln so vi d, nen zy trong canh be, co the bo qua (H.7.21)
zx trong ban canh: Xet mot iem trong canh (H7.21), bc = t
b
h t
S xc = t x
2
2 2

b
h t
Qy x
2
2 2
zx =
Jx

(7.19)

ng suat tiep zx phan bo bac nhat theo x , bieu o phan bo nh H.7.21

_________________________________________________________________
http://www.ebook.edu.vn 17
Chng 7: Uon phang thanh thang

GV: Le c Thanh

Th du 7.5 Tnh ng suat tiep cac iem tren truc trung hoa trong than cua
dam ch T co mat cat ngang nh tren H.7.22 . Cho b = 8 cm, d = 2 cm, h =
16 cm, h1 = 14 cm, va Q = 20 kN.
Giai
b = 8 cm

Khoang cach c ti trong tam cua mat cat ngang


c xac nh bi:
c=

8 2 1 + 14 2 9
= 6,09 cm
8 2 + 14 2

h = 16 cm

Momen quan tnh Jx cua mat cat ngang:


8 23
2 143
Jx =
+ 8 2 (6,09 1) 2 +
+ 14 2 (9 6,09) 2
12
12
= 1144,3 cm 4

=14cm

cm

+ ng suat tiep cac iem tren truc trung hoa:


H.7.22
bc = 2 cm
Momen tnh cua phan dien tch di truc trung hoa oi vi truc nay la:
20 98,208
2 (16 6,09)2
=
= 0,858 kN/cm 2
Sc =
= 98,208 cm 3
x

max

1144,3 2

+ ng suat tiep cac iem tiep giap canh va bung : bc = 2 cm

S xc = 2 8 (6,09 1) = 81, 44 cm 3

1 =

20 81,44
= 0,712 kN/cm 2
1144,3 2

7.4 KIEM TRA BEN DAM CHU UON NGANG PHANG


Tren mat cat ngang cua dam chu uon ngang phang co 2 ng suat:
- ng suat phap z do momen uon Mx gay ra.
- ng suat tiep zy do lc cat Qy gay ra.
Bieu o phan bo ng suat phap va ng suat tiep theo chieu cao cua mat
cat ngang hnh ch nhat (H.7.23b,c), ta thay co ba loai phan to trang thai
ng suat khac nhau (H.7.23a):
- Nhng iem bien tren va di = 0, ch co z 0 nen trang thai ng
suat cua cac phan to nhng iem nay la trang thai ng suat n
- Nhng iem nam tren truc trung hoa a = 0, ch co max nen trang
thai ng suat cua nhng phan to nhng iem nay la trt thuan tuy.
_________________________________________________________________
http://www.ebook.edu.vn 18
Chng 7: Uon phang thanh thang

GV: Le c Thanh

- Cac iem khac, z 0 va zy 0, nen chung trang thai ng suat


phang at biet.

min
min
max

max
Mmax

max
+

max

Qmax

b)

a)

c)

H. 7.23 a) Cac phan to trang thai ng suat khac nhau


b) S phan bo ng suat phap; c) S phan bo ng suat tiep

Khi kiem tra ben toan dam, phai bao am moi phan to eu thoa ieu
kien ben. (u 3 ieu kien ben)
a) Phan to trang thai ng suat n (nhng iem tren bien tren va
di cua dam), xet tai mat cat co M max va s dung thuyet ben ng suat phap
ln nhat ta co:
+ Dam lam bang vat lieu deo,

[ k ] = [ n ] = [] ,

(7.20)

max []

+ Dam lam bang vat lieu don,

ieu kien ben:

[ k ] [ n ] ,

ieu kien ben :

max [ k ]

(7.21)

min [ n ]

b) Phan to trang thai ng suat trt thuan tuy (nhng iem nam tren
truc trung hoa), xet tai mat cat co Qy

max

ta co

max =

Q ymax .S x
J x .b c

[ ]

+ Dam bang vat lieu deo:


Theo thuyet ben ng suat tiep ln nhat (TB 3): max [] =

[]
2

(7.22)

_________________________________________________________________
http://www.ebook.edu.vn 19
Chng 7: Uon phang thanh thang

GV: Le c Thanh

Theo thuyet ben the nang bien oi hnh dang (TB 4):
max [] =

[]

(7.23)

+ Dam bang vat lieu don: s dung thuyet ben Mohr (TB 5):
[ ]
1+m
[ k ]
m =
[ n ]

(7.24)

max [] =

trong o:

(7.25)

c) Phan to trang thai ng suat phang ac biet:


-Xet tai mat cat co momen uon Mx va lc cat Qy cung ln,(co the nhieu
mat cat).
-Chon iem nguy hiem tren mat cat e co z va zy tng oi ln (ch
can kiem tra tai nhng ni nguy hiem nh ni tiep giap gia long va e cua
mat cat ch , ch C)cho thay oi tiet dien. Cac ng suat cua phan to nay
c tnh bi cac cong thc quen thuoc:
z =

Mx
y
Jx

va

zy =

Qy S xc
J x bc

-Tnh ng suat chnh cua phan to. 1,3 =

1
2 + 4 2
2

ieu kien ben (chng 5):


+ Dam lam bang vat lieu deo:
Theo TB 3: (7.26)
Theo TB 4:

t4 =

t 3 = 1 3 =

2z + 4 2zy []

2z + 3 2zy []

(7.27)

+ Dam lam bang vat lieu don: Dung TB 5


t5 =

1m
1+ m
z +
2z + 42zy []
2
2

(7.28)

T ay cung co ba bai toan c ban:


Bai toan c ban 1: Kiem tra ben
Bai toan c ban 2: Chon kch thc mat cat ngang
Da vao ieu kien ben cua phan to trang thai ng suat n e chon
s bo kch thc mat cat ngang dam. Sau o, tien hanh kiem tra ben oi vi
cac phan to trang thai ng suat khac . Neu khong at th thay oi kch
thc mat cat ngang.
Bai toan c ban 3: nh tai trong cho phep.
T ieu kien ben cua phan to trang thai ng suat n, xac nh s
bo tai trong cho phep sau o tien hanh kiem tra ben cac phan to con lai
_________________________________________________________________
http://www.ebook.edu.vn 20
Chng 7: Uon phang thanh thang

GV: Le c Thanh

Th du 7.9 Cho dam co mat cat ngang va chu lc nh hnh ve.


1/ Ve bieu o Mx va Qy.
2/ Tnh ng suat phap va ng suat tiep ln nhat tai mat cat m-m
(ben trai c).
3/Tnh ng suat chnh tai iem K(mat cat tiep giap long va e)mat m-m,
Theo TB3.
3ql
m

ql 2

A
B m
ql
qa

qL

3L

11
qa
4
15
qL
4

7,6cm

8cm

X
4,4cm

4cm 4

13
qL
4

3cm

12cm 3cm

11
qL
4
_

13
qL
4

1
qL
4

J X = 1388,4 cm 4
ql

ql 2

17 2
qL
4

Tai mat cat m-m conoi lc : M x =


Qy =

17 2 17
qa = 10 1 1 = 42,5kN m
4
4

11
11
qL = 10 1 = 27,5kN
4
4

k
n
y max
= 4,4, cm, y max
= 7,6cm

mmax
m =

Mx k
4250
y max =
4,4 = 13,47kN / cm 2
Jx
1388,4

_________________________________________________________________
http://www.ebook.edu.vn 21
Chng 7: Uon phang thanh thang

GV: Le c Thanh

mmin
m =

mm

max

Mx n
4250
y max =
7,6 = 23,26kN / cm 2
Jx
1388,4

Q y S xc
Jxb

, vi

= 0,572 kN / cm 2

7,6

S XC = 2(3 7,6
) = 173,28cm 3
2

Tnh ng suat chnh tai K.


k =

27,5 158,4
= 0,174cm 2 ,
1388,4 18

kz =

S xc 18 4 2,2cm 3 = 158,4cm 3

4250
(4,4 4 ) = 1,22 kN/cm 2
1388,4

Theo thuyet ben 3:


t 3 = K2 + 4 K2 =

(1,22)2 + 4(0,74)

= 2,22kN / cm 2

Th du 7.6 Xac nh kch thc mat cat ngang hnh ch nhat ,


cho[ ] = 1 kN/cm2., L=1m ,h=2b .Tnh max
q=2kN/m

h=2b

B
P=3qa
L

b
qa

qa
2qa
qa 2
2

qa 2

max =

M x ,max
W

qa 2 6 2 1 100 6
=
=
1
b h2
b (2b) 2

b=7cm,h=14cm

_________________________________________________________________
http://www.ebook.edu.vn 22
Chng 7: Uon phang thanh thang

GV: Le c Thanh

max = 1,5

Qy
F

1,5 2qa 1,5 2 2 1


=
= 0,06 kN / cm 2
7 14
98

Th du 7.7 Xac nh so hieu mat cat ngang theo yeu cau o ben, neu
[ ] = 16 kN/cm2.
zo

60 kN

60 kN

6m

1m

h/2

1m

d
t

60 kN
h/2

Qy

60 kN

60 kNm

Mx

H.7.22
H.7.21

Giai.

Mo men uon cc ai va lc cat cc ai xay ra tai cung mot mat cat di tac
dung cua tai trong:
Mmax = 60 kNm; Qmax = 60 kN
Mo men chong uon can thiet la:
Wx =

M x ,max

[ ]

6000
= 375cm 3
16

Tra bang thep hnh mat cat [

OCT 8240-56 ta chon 2[22 vi:

mot [22 co d = 5,3 mm, F = 26,7 cm2; Wx = 193 cm2; Sx = 111 cm3;
Jx = 2120 cm4; h = 22 cm; t = 0,96 cm; b = 8,2 cm.
Kiem tra ben thep hnh mi chon:
* Phan to trang thai ng suat n: ng nhien thoa
* Phan to trang thai ng suat trt thuan tuy: tai mat cat co:
Qy,max = 60 kN

_________________________________________________________________
http://www.ebook.edu.vn 23
Chng 7: Uon phang thanh thang

GV: Le c Thanh

max =

Q y S xc
I xbc

vi

S xc = 2 S x = 2 111 cm3
J x = 2 2120 cm4
b c = 2d = 2 0,53 cm
Qy = 60 kN

60 2 111

2
Suy ra: max = 2 2120 2 0,53 = 2,96 kN/cm

Theo thuyet ben ng suat tiep cc ai:

[ ] = [ ] = 16 = 8kN/cm2 > max


2

vay phan to nay thoa ieu kien ben.


* Phan to trang thai ng suat phang ac biet: o la phan to ni tiep giap
gia long va e tai mat cat nay co:
M x ,max = 60 kNm
xA =

va Q y ,max = 60 kN

6000
(11 0,96) = 14,21 kN/cm2
2 2120

0,96

3
S xc = 2 8,2 0,96 11
= 165,626 cm
2

A =

60 165,626
= 2,21 kN/cm 2
2 2120 2 0,53

Theo thuyet ben ng suat tiep cc ai:


t 3 = A2 + 4 A2 =

(14,21)2 + 4(2,21)

= 14,38kN / cm 2

vay phan to nay thoa ieu kien ben.


Ket luan: Chon 2 [ 22.
Th du 7.8 Xac nh tai trong cho phep [P] cua dam cho tren H.7.25.
Cho: a = 80 cm, [ ] = 16 kN/cm2
Giai

2P

Bieu o lc cat Qy va momen uon Mx


a

(H.7.25). Mat cat nguy hiem co:


Mx =

7
Pa
4

va

Qy =

7
P
4

2a
5/4P

10

P/4

Mat cat I 10 co:h = 10 cm; Jx = 198 cm4


Wx = 39,7cm3; Sx = 23cm3 ,

7/4P

Qy

_________________________________________________________________
Mx http://www.ebook.edu.vn 24
Chng 7: Uon phang thanh thang
5/4Pa

7/4Pa

GV: Le c Thanh

d = 0,45 cm; t = 0,72 cm; b = 5,5 cm


T ieu kien ben cua phan to
TTS n nguy hiem ta co:
7 Pa
[]
4Wx

4 16 39,7
4 []Wx
=
= 4,537 kN
7
80
7 a

Ta chon [P] = 4,53 kN.


Vi tr so cua P a chon, ta kiem tra ben cac phan to con lai TTS
trt thuan tuy va TTS phang ac biet.
++ Phan to TTS trt thuan tuy ; truc trung hoa cua mat cat co:
7
7
P = 4,53 = 7,923 kN
4
4
7 4,53 23
[ ]
=
= 2,046 kN/cm 2 < [] =
= 8 kN/cm 2
4 198 0,45
2

Qy =

max

phan to nay thoa ieu kien ben.

++ Phan to TTS phang ac biet; ni tiep giap gia long va e tai


mat cat co:
7
7
7
P = 7,923 kN
M x = Pa = 4,53 0,8 = 6,342 kNm va Qy =
4
4
4
Sxc = 5,5 0,72

zy

(10 0,72)
= 18,37 cm 3
2

7
4,53 18,37
= 4
= 1,634 kN/cm 2
198 0,45

z =

634,2 10

0,72 = 13,71 kN/cm2


198 2

Theo thuyet ben ng suat tiep cc ai:


t3 =

2z + 4 2zy =

(13,71)2 + 4 (1,634)2

= 14,09 kN/cm2 < [] = 16 kN/cm2

Ket luan: Tai trong cho phep [P] = 4,53 kN

_________________________________________________________________
http://www.ebook.edu.vn 25
Chng 7: Uon phang thanh thang

GV: Le c Thanh

Th du 7.10:Cho dam ABC chu lc nh hnh ve .


nh [q] cho[ ] = 16 kN/cm2. [ ]=9kN/cm2
16 1cm
2ql

2ql

ql

A
B
4l

2,2ql
2,2ql

N 0 20

Y 16 1cm

4,8ql

1,8ql

3,8ql
3ql2
0,8ql

ql

h=20cm,b=0,76cm
d =0,72cm,t=0,9cm

JX=1520cm4 ,WX=152cm3
SX=87,8cm3,

ql2

2,42ql2

Tnh:

J X = 2(

Max z =

M xmax
Wx

16 13
12

+ (10 , 5 ) 2 16 1 ) + 2 J x = 6570 cm

[ ] , vi

Wx =

J X 6570
=
= 597,3cm 3
H
11
2

_________________________________________________________________
http://www.ebook.edu.vn 26
Chng 7: Uon phang thanh thang

GV: Le c Thanh

[q] [ ]2Wx
3l

16 597,3
= 14,2kN / m , vi
3(1,5) 2

M xmax = 3ql 2

Kiem tra lai ng suat tiep vi q va tm.


S xc = 2 S x + (16 1 10 ,5) cm 3

max =

Qy S xc
J x bc

= 4,07 kN / cm 2 [ ] , vi

J x = 6570 cm 4 , l = 1,5 m
b c = 2 d = 2 0,52 cm
Q y = 3,8ql = 3,8 14,2 1,5 = 80,94kN

_________________________________________________________________
http://www.ebook.edu.vn 27
Chng 7: Uon phang thanh thang

GV: Le c Thanh

7.5 QU AO NG SUAT CHNH


Trong phan ben tren chung ta ch mi xac nh tr so cua ng suat
chnh oi vi mot phan to bat ky ma cha e cap en phng cua chung.
Nhng ket qua at c kha tot oi vi vat lieu co ng suat cho phep khi
keo va khi nen la nh nhau. Tuy nhien, oi vi cac vat lieu nh be tong cot
thep, viec xac nh phng cua ng suat chnh tai moi iem rat can thiet, e
t o co the at cot thep gia cng theo cac phng nay.
Ta co the xac nh phng cua ng suat chnh thong qua vong tron
Mohr. Gia s va la cac thanh phan ng suat phap va ng suat tiep tren
mat phang vuong goc vi truc dam va co tr so dng:
= + z =

Qy S xc
Mx
y va = + zy =
Jx
J xb c

Phng 3

Phng 1

B
C

= 0
= zy

a = z

= zy

zy

H. 7.26

Sau khi ve vong tron Mohr ng suat chung ta nhan thay phng chnh
la phng noi t iem cc P(0,+zy) vi hai iem A va B hai au ng
knh cua vong tron Mohr: PA ch phng ng suat chnh 1, con PB ch
phng ng suat chnh 3.
H.7.26 cho thay, cac vong tron Mohr ng suat va cac phng chnh tai
nhieu iem khac nhau tren mat cat ngang. Ta gia s rang momen uon va
lc cat tai mot mat cat mang dau dng. ng suat chnh thay oi vi bien
mat cat ngang. Gan nhng bien, mot trong cac ng suat chnh bang khong,
trong khi ng suat chnh kia co phng song song vi truc dam; con truc
trung hoa, cac ng suat chnh co phng hp vi truc dam mot goc 45o.
Bang phng phap tng t, ta co the xac nh c phng cua ng suat
chnh nhieu iem tren dam (H.7.27) Ta ve cac ng cong co tiep tuyen
_________________________________________________________________
http://www.ebook.edu.vn 28
Chng 7: Uon phang thanh thang

GV: Le c Thanh

la phng cua ng suat chnh va goi cac ng o la quy ao ng suat


chnh cua dam chu uon. Cac quy ao nay hp thanh hai ho ng cong
vuong goc nhau, mot ho la quy ao ng suat keo va mot ho la quy ao ng
suat nen. Cac phng cua ng suat chnh tuy thuoc vao loai tai trong va
ieu kien bien cua dam.
Tren H.7.28, quy ao ng suat keo c bieu dien bang ng net
am con quy ao ng suat nen bieu dien bang ng net t.
Ngi ta thng dung cac phng phap thc nghiem e xac nh quy
ao ng suat chnh nh phng phap quang an hoi, phng phap dung
sn don.

min

min

max

max

Mx

Phng
nen 3

max

Qy

Phng nen 3

Phng
keo 1

max

Phng
keo 1

max

Phng
nen 3

Phng
keo 1

H.7.25
q
A

H. 7.28. Quy ao ng suat chnh cua dam


ta n chu tai phan bo eu

7.6 THE NANG BIEN DANG AN HOI CUA DAM CHU UON PHANG

_________________________________________________________________
http://www.ebook.edu.vn 29
Chng 7: Uon phang thanh thang

GV: Le c Thanh

Trong chng TRANG THAI NG SUAT, ta a co cong thc tnh the


nang rieng bien dang an hoi cua mot phan to la:
u=

U
1 2
=
1 + 22 + 32 2 ( 1 2 + 2 3 + 3 1 )
V 2E

(7.29)

Trng hp dam chu uon ngang phang, trang thai ng suat cua phan
to la phang nen mot thanh phan ng suat chnh bang khong, 2 chang han,
khi o bieu thc cua the nang rieng bien dang an hoi co dang:
u=

1 2
dU
=
1 + 32 2 1 3
dV 2 E

(7.30)

trong o: 1 va 3 la cac ng suat chnh c suy t z va zy theo cong thc:


2

1 =

z

+ z + 2zy
2
2

3 =

z

z + 2zy
2
2

(7.31)

(7.32)

thay vao (7.30)


2

z 2 z 2

z 2
1 z
2
2
u=
2 + 2 + zy 2 zy
2 E 2
2

2 2

rut gon ta c:
u=

z2
2E

zy2 2(1 + )
2

(7.33)

Ngoai ra, gia cac hang so cua vat lieu E, G, ton tai he thc sau:
G=

E
2(1 + )

(7.34)

thay vao (7.33) va rut gon, cuoi cung ta c:


u =

2
2z zy
+
2 E 2G

(7.35)

thay bieu thc cua z va zy bang (7.2) va (7.11) ta c:


u=

( )
( )

Qy2 S xc
M x2 2
+
y
2 EJ x2
2GJ x2 b c

(7.36)

The nang bien dang an hoi trong mot oan thanh dz la:

_________________________________________________________________
http://www.ebook.edu.vn 30
Chng 7: Uon phang thanh thang

GV: Le c Thanh
2
M2
Qy2 (S xc )
2
x

dU = udz.dF = dz udF = dz
y +
dF
c 2
2
F
F 2 EJ x2

(
)
2
GJ
b
x

vi:

F
J x2

(a)

y 2 dF = J x va neu ta ky hieu:

(S )
(b )

c 2
x
c 2

(b)

dF =

Qy2
M x2
dz +
dz
2 EJ x
2GF

ta c: dU =

(c)

Do o, the nang bien dang an hoi trong ca thanh vi chieu dai L la:
U =

L
o

M x2
dz +
2 EJ x

L
o

Qy2
2GF

(7.37)

dz

Vi thanh co o cng thay oi tng oan hay luat bien thien cua Mx va
Qy thay oi tng oan thanh, cong thc tren co the rut gon lai:
n

U =
i =1

Li
0

n
Li
Qy2
M x2
dz +
dz
0
2 EJ x
2GF
i =1

(7.38)

trong o: Li - chieu dai moi oan thanh, n - so oan thanh

he so ieu chnh s phan bo khong eu cua ng suat tiep.


Bang cach ap dung cong thc tnh ta co the tnh c he so nay oi
vi mot so tiet dien thong thng
- Mat cat ngang hnh ch nhat: = 1,2
- Mat cat ngang hnh tron:

= 10 / 9

- Mat cat ngang ch : = F / Flong


trong o: F - la dien tch toan bo mat cat.
Flong la dien tch phan long (phan ban bung) cua ch .

7.7 DAM CHONG UON EU

Trong trng hp dam co mat cat


ngang khong oi, ta a chon kch thc
cua theo mat cat co mo men uon ln
nhat. Cach s dung vat lieu nh vay cha
hp ly v khi ng suat tai nhng iem
nguy hiem tren mat cat co mo men uon

P/2

l/2

l/2

Pl/4

H. 7.29

_________________________________________________________________
http://www.ebook.edu.vn 31
Chng 7: Uon phang thanh thang

P/2

GV: Le c Thanh

ln nhat at en tr so ng suat cho phep th ng suat tai nhng iem nguy


hiem tren cac mat cat khac con nho hn rat nhieu so vi ng suat cho
phep. Nh vay ta cha s dung het kha nang chu lc cua vat lieu cac
mat cat khac. e tiet kiem c vat lieu ta phai tm hnh dang hp ly cua
dam sao cho ng suat tai nhng iem nguy hiem tren moi mat cat ngang
eu cung at en gia tr ng suat cho phep. Dam co hnh dang nh vay goi
la dam chong uon eu.
Ta xet vai th du cu the sau ay.
Gia s, ta co dam chu lc nh tren hnh ve (H.7.29), mo men uon Mx
va lc cat Qy tren mat cat 1-1 nao o cach goi ta A ben trai mot khoang
cach co tr so la:
p
z
2
p
Qy =
2

Mx =

Gia thiet mat cat ngang co hnh dang la mot hnh tron. Nh vay tr so
ng suat phap ln nhat tren mat cat c tnh vi cong thc:
max =

Mx
P.z
=
Wx 0,1d 3

Vi ieu kien ng suat cc ai tren moi mat cat cung at ti tr so ng


suat cho phep [], ta tm c luat bien thien cua ng knh d theo bien so
z nh sau:
d=

p.z
0,1[ ]

d1

(a)
H. 7.30

Nh vay hnh dang cua thanh phai co dang ng net t nh tren


hnh ve (H. 7.30).
Ta thay tai hai au mut, mat cat co dien tch bang khong, ieu o hoan
toan phu hp vi ieu kien bien thien cua mo men uon, v tai o mo men
uon bang khong. Song, nh vay khong thoa man ieu kien ben cua lc cat
Qy. Qua vay, tren moi mat cat cua dam ta eu co mot tr so lc cat Qy =
lc cat o sinh ra ng suat tiep ln nhat max =

P
va
2

4 Qy
. V the dien tch cua mat
3 F

cat can phai u e chu cat. Do o phai chon ng knh vi ieu kien:
_________________________________________________________________
http://www.ebook.edu.vn 32
Chng 7: Uon phang thanh thang

GV: Le c Thanh

max =

4 Qy
[ ]
3 F

ng knh co tr so be nhat cung phai la:


d = d1 =

4 Qy
3 [ ] .

(b)

V ieu kien che tao, rat kho gia cong e thanh co the co hnh dang
ng cong c bieu dien theo bieu thc (a), nen trong thc te ngi ta
thng lam cac truc hnh bac, ngha la ng knh cua cac mat cat thay oi
tng oan mot, gan sat vi ng chong uon eu (H. 7.31).
Cac lo xo co s o chu lc nh (H.7.31), thng c ghep bi cac la
thep nh (H.7.32). Cac la thep c ghep theo hnh dang cua dam chong
uon eu, hnh dang o lam lo xo co trong lng nho va chuyen v ln. Loai
lo xo nay thng dung lam dp cua cac truc banh xe.

H. 7.31

H. 7.32

oi vi dam co s o chu lc nh (H.7.33), neu chieu cao cua dam


khong oi th dam chong uon eu co hnh dang nh tren (H. 7.34). Mat cat
au t do co dien tch khac khong v dam con chu lc cat. Dien tch o
c xac nh tuy theo tr so cua lc cat.

H. 7.33

H. 7.34

_________________________________________________________________
http://www.ebook.edu.vn 33
Chng 7: Uon phang thanh thang

GV: Le c Thanh

BAI TAP CHNG 7


7.1 Xac nh chieu dai nhp ln nhat cho dam ta n co mat cat ngang
hnh ch nhat (140 mm 240 mm) chu tac dung cua tai phan bo eu
cng o q = 6,5 kN/m neu ng suat cho phep la 8,2 MPa (trong lng
cua dam a ke trong q.
Tra li: 3,68 m
7.2 Mot dam thep mat cat ngang hnh ch I ta n va co hai au mut tha
nh tren H.7.2. Dam chu tac dung cua lc phan bo eu cng o q = 10
kN/m moi au mut tha. Gia s mat cat ngang ch co so hieu 16 co
momen chong uon (hay suat tiet dien) la 109 cm3. Xac nh ng suat
phap cc ai trong dam do uon, max do q.
q = 10 kN/m

2m

q = 10 kN/m

4m

2m

H 7.2

7.3 Mot dam bang go ABC co mat cat ngang hnh vuong canh b, ta n
tai A va B chu tai trong phan bo eu q = 1,5 kN/m tren phan mut tha
BC (H.7.3). Tnh canh cua hnh vuong b, gia s chieu dai nhp L = 2,5 m
va ng suat cho phep [] = 12 MPa. Hay ke en trong lng rieng cua
dam biet rang trong lng rieng cua go la = 5,5 kN/m3.
q = 1,5 kN/m

A
L = 2,5 m

B
L = 2,5 m

H. 7.3

7.4 Mot mang nc co mat cat ngang nh H.7.4.


Mang at len hai cot cach nhau 6 m. Vat lieu lam
mang co trong lng rieng = 18 kN/m3. Hoi khi
cha ay nc th ng suat phap va ng suat tiep
cc ai la bao nhieu?

16

12
4

H. 7.4

_________________________________________________________________
http://www.ebook.edu.vn 34
Chng 7: Uon phang thanh thang

GV : Le c Thanh

Chng 8

CHUYEN V CUA DAM CHU UON


8.1 KHAI NIEM CHUNG
Khi tnh mot dam chu uon ngang phang, ngoai ieu kien ben con phai
chu y en ieu kien cng. V vay, can phai xet en bien dang cua dam.
Di tac dung cua cac ngoai lc, truc dam b uon cong, truc cong nay c
goi la ng an hoi cua dam (H.8.1).

K
ng an hoi

ng an hoi
y

P v y(z)

P
K

H.7.1

H.7.2

Xet mot iem K nao o tren truc dam trc khi bien dang. Sau khi bien
dang, iem K se di chuyen en v tr mi K. Khoang cach KK c goi la
chuyen v thang cua iem K. Chuyen v nay co the phan lam hai thanh
phan:
Thanh phan v vuong goc vi truc dam (song song vi truc y) goi la
chuyen v ng hay o vong cua iem K.
Thanh phan u song song vi truc dam (song song vi truc z) goi la
chuyen v ngang cua iem K.
Ngoai ra , sau khi truc dam bien dang, mat cat ngang K b xoay i
mot goc , ta goi goc xoay nay la chuyen v goc (hay la goc xoay ) cua
mat cat ngang iem K. Co the thay rang, goc xoay chnh bang goc gia
truc cha bien dang cua dam va tiep tuyen iem K cua ng an hoi
(H.8.1).
Chong 8: Chuyen v cua dam chu uon

http://www.ebook.edu.vn

GV : Le c Thanh

Ba ai lng u, v, la ba thanh phan chuyen v cua mat cat ngang


iem K.
Trong ieu kien bien dang cua dam la be th thanh phan chuyen v
ngang u la mot ai lng vo cung be bac hai so vi v, do o co the bo qua
chuyen v u va xem KK la bang v, ngha la v tr K sau khi bien dang nam
tren ng vuong goc vi truc dam trc bien dang (H.8.2).
Goc xoay co the lay gan ung:

tg =

dv
.
dz

Neu chon truc dam la z, truc y vuong goc vi truc dam, th chuyen v v
chnh la tung o y cua iem K. Tung o y cung chnh la o vong cua iem
K. Ta thay ro neu K co hoanh o z so vi goc nao o th cac chuyen v y,
cung la nhng ham so cua z va phng trnh an hoi la:
y(z) = v(z)
Phng trnh cua goc xoay se la:
(z) =

dv
dy
=
= y' (z)
dz
dz

hay, phng trnh cua goc xoay la ao ham cua phng trnh ng
an hoi.
Quy c dng cua chuyen v:
- o vong y dng neu hng xuong.
- Goc xoay dng neu mat cat quay thuan chieu kim ong ho.
ieu kien cng: Trong ky thuat, khi tnh toan dam chu uon, ngi ta
thng khong che o vong ln nhat cua dam khong c vt qua mot gii
han nhat nh e am bao yeu cau ve s lam viec, my quan cua cong
trnh..., ieu kien nay c goi la ieu kien cng. Neu goi f la o vong ln
nhat cua dam th ieu kien cng thng chon la:
1
1
f
L = 300 1000

trong o: L - la chieu dai nhp dam.


Tuy loai cong trnh ma ngi ta quy nh cu the tr so cua [ f L] .

Chong 8: Chuyen v cua dam chu uon

http://www.ebook.edu.vn

GV : Le c Thanh

8.2 PHNG TRNH VI PHAN CUA NG AN HOI


Xet 1 iem bat ky K tren truc dam.
Trong chng 7 (cong thc 7.1) ta a lap c moi lien he gia o cong
cua truc dam tai K sau bien dang vi momen uon noi lc Mx tai K la:
1

Mx
EJ x

(a)

Mat khac, v ng an hoi c bieu dien bi phng trnh ham so y(z)


trong he truc (yz) nen o cong cua o th bieu dien cua ham so 1 iem K
co hoanh o z c tnh theo cong thc:
1

(a) va (b)

(1 + y )
2

(1 + y' )
2

3
2

(b)

3
2

Mx
EJ x

(c)

o la phng trnh vi phan tong quat cua ng an hoi, tuy nhien phai
chon sao cho hai ve cua phng trnh tren eu thoa man.

z
Mx

Mx

Mx

Mx

Mx > 0

Mx < 0

y < 0

y > 0

H.8. 3

Khao sat mot oan dam b uon cong trong hai trng hp nh H.8.3. Trong
ca 2 trng hp momen uon Mx va ao ham bac hai y luon luon trai dau,
cho nen phng trnh vi phan cua ng an hoi co dang:
y' '

(1 + y' )
2

3
2

Mx
EI x

Vi gia thiet chuyen v la be (o vong va goc xoay be), co the bo qua


(y)2 so vi 1 va khi o phng trnh vi phan cua ng an hoi co dang
gan ung nh sau:
Chong 8: Chuyen v cua dam chu uon

http://www.ebook.edu.vn

GV : Le c Thanh

y' ' =

Mx
EI x

(8.1)

trong o: Tch so EJx la o cng khi uon cua dam .


8.3

LAP PHNG TRNH NG AN HOI BANG PHNG PHAP

TCH PHAN KHONG NH HAN


Ve phai cua phng trnh vi phan (8.1) ch la mot ham so cua z nen (8.1)
la phng trnh vi phan thng.
Tch phan lan th nhat (8.1) phng trnh goc xoay:
= y' =

Mx
dz + C
EJ x

(8.2)

Tch phan lan th hai phng trnh ng an hoi:

M
y = x dz + C dz + D
EJ x

(8.3)

Trong (8.2) va (8.3), C va D la hai hang so tch phan se c xac nh


cac ieu kien bien. Cac ieu kien nay phu thuoc vao lien ket cua dam va
phu thuoc vao s thay oi tai trong tren dam.
A
yA = A = 0

A
a)

yA = 0

C
b)

yB = 0

H. 8.4

oi vi dam n gian, co the co cac ieu kien nh sau:


+ au ngam cua dam console co goc xoay va o vong bang khong
(H.8.4a):
yA = A = 0
+ Cac au lien ket khp o vong bang khong (H.8.4b):
y A = yB = 0
+ Tai ni tiep giap gia hai oan dam co phng trnh ng an hoi
khac nhau, o vong va goc xoay ben trai bang vi o vong va goc xoay
ben phai ( iem C tren H.8.4b):

yCtr = yCph;

Chong 8: Chuyen v cua dam chu uon

Ctr = Cph

http://www.ebook.edu.vn

GV : Le c Thanh

Th du 8.1 Viet phng trnh ng an hoi va goc xoay cho dam cong
son (console) nh H.8.5. T o suy ra o vong va goc xoay ln nhat. Cho
EJx = hang so.
Giai.

Phng trnh momen uon tai

mat cat co hoanh o z la:


Mx=Pz

yB = B = 0

z
L

(a)
y

H.7.5

the vao (8.1) phng trnh vi phan cua ng an hoi :


Mx
Pz
=
EJ x EJ x

y' ' =

Pz 2
+C
2 EJ x

(c)

Pz 3
+ Cz + D
6 EJ x

(d)

tch phan hai lan, = y ' =


y=

(b)

C va D c xac nh t cac ieu kien bien ve o vong va goc xoay tai


ngam:
z = L; = 0 va y = 0
thay cac ieu kien nay vao (c) va (d)
C=

PL2
PL3
; D=
2 EJ x
3EJ x

Vay phng trnh ng an hoi va goc xoay la:


y=

Pz 3
PL2
PL3

z+
;
6 EJ x 2 EJ x
3EJ x
Pz 2
PL2

2 EJ x 2 EJ x

o vong va goc xoay ln nhat au t do A cua dam; ng vi z = 0, ta co:


ymax =

PL3
PL2
; =
3EJ x
2 EJ x

ymax > 0 ch rang o vong cua iem A hng xuong


< 0 ch rang goc xoay cua iem A ngc kim ong ho.

Chong 8: Chuyen v cua dam chu uon

http://www.ebook.edu.vn

GV : Le c Thanh

Th du 8.2 Tnh o vong va goc xoay ln nhat cua dam (H.8.6).


Cho EJx = hang
Giai.

Phng trnh momen uon tai

mat cat co hoanh o z la:


Mx =

yB = B = 0

z
L

qz2
2

(a)

H.8.6

qz 2
y' ' =
2 EJ x

the vao (8.1),

tch phan hai lan, = y ' =


y=

(b)
qz 3
+C
6 EJ x

(c)

qz 4
+C z+ D
24 EJ x

(d)

hai ieu kien bien au ngam z = L; = 0 va y = 0 cho :


qL3
qL4
C=
; D=
6 EJ x
8 EJ x

Vay phng trnh an hoi va goc xoay la:


qL4
qL3
qL4
y=

z+
;
24 EJ x 6 EJ x
8 EJ x

qL3
qL3

6 EJ x 6 EJ x

o vong va goc xoay ln nhat au t do A cua dam; ng vi z = 0, ta


co:

ymax

qL4
=
8EJ x

va

qL3
A =
6 EJ x

Th du 8.3 Tnh o vong va goc xoay ln nhat cua dam n gian chu tai
phan bo eu (H.8.7). o cng EJx cua dam khong oi.
Giai.

Phng trnh momen uon tai


mat cat ngang co hoanh o z la:
Mx =

qL
qz2
q
=
z
Lz z2
2
2
2

Chong 8: Chuyen v cua dam chu uon

L/2

(a)

thay vao (8.1), phng trnh vi


phan cua ng an hoi nh sau:

L
y

H.8.7

http://www.ebook.edu.vn

GV : Le c Thanh

y' ' =

q
Lz z 2
2 EJ x

tch phan hai lan, = y ' =


y=

(b)

q Lz 2 z 3

+ C
2 EJ x 2
3

(c)

q Lz 3 z 4

+C z + D
2 EJ x 6 12

(d)
khi : z = 0; y = 0

ieu kien bien cac goi ta trai va phai cua dam: khi : z = L; y = 0

D = 0; C =

qL3
24 EJ x

Nh vay phng trnh ng an hoi va goc xoay la:


y=

qL3
24 EJ x

= y' =

z2 z3
z 1 2 2 + 3
L L

(e)

qL3
z2
z3
1 6 2 + 4 3
24 EJ x
L
L

(g)

o vong ln nhat cua dam tai mat cat ngang gia nhp ng vi:
z=

L
2

(tai ay y = 0)

thay z =

L
2

vao (e), ymax = y

L
z=
2

5qL4
384 EJ x

Goc xoay ln nhat, nho nhat (ymax , ymin) tai mat cat ngang co y = 0
(hay Mx = 0), tc cac goi ta trai va phai cua dam. Thay z = 0 va z = L
lan lt vao (g)

max = y 'max =

1 qL3
24 EJ x

min = y 'min =

1 qL3
24 EJ x

Goc xoay cua mat cat goi ta trai thuan chieu kim ong ho, goc xoay
cua mat cat goi ta phai ngc chieu kim ong ho.

Chong 8: Chuyen v cua dam chu uon

http://www.ebook.edu.vn

GV : Le c Thanh

Th du 8.4 Lap phng trnh o vong va goc xoay cua dam tren hai goi ta
chu lc tap trung P nh H.8.8 cho biet EJx = hang so.
P
B

z1
a

Z2
L

Giai.

Pab/L

H.8.8

Dam co hai oan, bieu thc momen uon trong hai oan AC va CB
khac nhau nen bieu thc goc xoay va o vong trong hai oan cung khac
nhau. Viet cho tng oan cac bieu thc Mx, y, y, y nh sau:
Momen uon Mx trong cac oan sau:
oan AC (0 z1 a):

M x(1) =

Pb
z1
L

(a)

oan CB (a z2 L):

M x(2) =

Pb
z2 P (z2 a )
L

(b)

Phng trnh vi phan cua ng an hoi trong moi oan:


oan AC:
oan CB:

y1 ' ' =

Pb
z1
LEJ x

y2 ' ' =

Pb
P
( z2 a )
z2 +
EJ x
LEJ x

(c)
(d)

Tch phan lien tiep cac phng trnh tren, ta c:


oan AC (0 z1 a):
y1 ' =

Pb 2
z1 + C1
2 LEJ x

(e)

y1 =

Pb 3
z1 + C1 z1 + D1
6 LEJ x

(g)

oan CB (a z2 L):

Chong 8: Chuyen v cua dam chu uon

http://www.ebook.edu.vn

GV : Le c Thanh

y2 ' =

Pb 2
P
( z 2 a ) 2 + C2
z2 +
2 LEJ x
2 EJ x

y2 =

Pb 3
P
(z2 a ) 3 + C2 z2 + D2
z2 +
6 LEJ x
6 EJ x

(h)
(i)

Xac nh cac hang so tch phan C1, D1, C2, D2 t cac ieu kien bien
- goi ta A, B o vong bang khong
- mat cat ngang C noi tiep hai oan, o vong va goc xoay cua hai
oan phai bang nhau.

khi:

z1 = 0; y1 = 0
z2 = 0; y2 = 0
z1 = z2 = a; y1 = y2; y1 = y2

T bon ieu kien nay :


D1 = 0

Pb L3 + P (L a ) 3 + C2 L + D2 = 0
6 LEJ x
6 EJ x

Pb 3
Pb 3

6 LEJ a + c1a + D1 = 6 LEJ a + c2 a + D2


x
x

Pb
Pb

2
2
2 LEJ a + c1 = 2 LEJ a + c2
x
x

Giai he phng trnh tren,


D1 = D2 = 0; C1 = C2 =

Pb
L2 b 2
6 LEJ x

Vay phng trnh goc xoay va o vong trong tng oan la:
oan AC (0 z1 a):

Pb L2 b 2 z12
'


1 = y1 =
6
2
LEJ

2
2
3
y = Pb L b z z1
1
1

6
LEJ x 6

oan BC (a z2 L):
2

Pb z22 L(z2 a ) L2 b 2
'

2 = y2 =
2

2
6
LEJ
b
x

3
2
2
3
L b
z2
Pb (z2 a )

y2 = LEJ 6b L + 6 z2 6
x

Tnh o vong ln nhat trong dam bang cach da vao ieu kien y = 0,
Chong 8: Chuyen v cua dam chu uon

http://www.ebook.edu.vn

GV : Le c Thanh

Gia s a > b. Trc het ta se xet o vong ln nhat trong oan nao
goi ta A (z1 = 0) goc xoay bang:
1 A =

va C (z1 = a):

PbL b 2
1 > 0
6 EJ x L2

1C =

PbL
(a b ) < 0
3EJ x

Nh vay, gia hai iem A va C goc xoay 1 oi dau, ngha la se b triet


tieu mot lan. ieu o cho thay o vong co gia tr ln nhat trong oan AC.
e tm hoanh o z1(0) cua mat cat ngang co o vong ln nhat, ta cho
phng trnh 1 = 0:

0,500L

2
Pb L b 2 (z1 (0 ))
1 [z1 (0)] =

=0
LEJ x 6
2

z1 (0) =

L b
3

0,577L

(o)

H.8.9

Sau o a vao bieu thc (l) cua o vong, gia tr ln nhat cua o
vong

ymax = y1(z1( 0 ) ) =

3Pb L2 b 2
27 EJ x

b2
1 2
L

(p)

Cac he qua:
- Neu P at gia nhp dam (b = L / 2) , th t (o) va (p) , ta c:
z1 (0) =

L
PL3
= 0,500 L ; ymax =
2
48EJ x

- Khi P gan goi B, tc b 0 ta co: z1(0) =

L
3

= 0577L

Nh vay, neu tai trong di chuyen t trung iem D gia nhp dam en
goi ta B (H.8.9) th hoanh o z1(0) se bien thien t 0,5L en 0,577L, tc la
t iem D en iem E. Trong thc te ngi ta thng quy c la khi tai
trong P tac dung mot v tr nao o th van co the coi o vong ln nhat
gia nhp dam.
Th du, neu tai trong P tac dung v tr nh H.8.8 th o vong gia
nhp dam se bang:
So sanh hai gia tr ymax va

y(l 2 ) =
y(l 2 ) thay

Pb
3L2 4b 2
48 EJ x

hai gia tr nay khac nhau va rat t

.
Chong 8: Chuyen v cua dam chu uon

http://www.ebook.edu.vn 10

GV : Le c Thanh

Nhan xet: Neu dam co nhieu oan, can phai lap phng trnh vi phan
ng an hoi cho nhieu oan tng ng. moi oan , phai xac nh hai
hang so tch phan, neu dam co n oan th phai xac nh 2n hang so, bai
toan tr nen phc tap neu so oan n cang ln, v vay phng phap nay t
dung khi tai trong phc tap hay o cng dam thay oi.

8.4 XAC NH O VONG VA GOC XOAY BANG PHNG PHAP TAI


TRONG GIA TAO (PHNG PHAP O TOAN)

Phan trc, a co lien he vi phan gia noi lc va ngoai lc ( CH. 2):

dQ
= q
dz
dM x
= Q
dz
d2 M x
= q
dz2

(a)

oi vi viec khao sat ng an hoi cua dam , cung co phng trnh


vi phan:
d2y
M
= x
2
dz
EJ x

(b)

oi chieu cac phng trnh (a) va (b), ta thay co s tng t sau:


y

Mx

dy
= y'
dz

dM x
=Q
dz

d2y
M
= y' ' = x
2
dz
EJ x

d 2M x
=q
dz 2

Ta nhan thay muon tnh goc xoay y va o vong y th phai tch phan
lien tiep hai lan ham so

Mx
EJ x

Tng t muon co lc cat Qy va momen uon Mx th phai tch phan lien


tiep hai lan ham so tai trong q.
Tuy nhien phan trc ( CH.2), ta a tnh lc cat Qy va momen uon
Mx theo tai trong q t viec khao sat cac phng trnh can bang.
Chong 8: Chuyen v cua dam chu uon

http://www.ebook.edu.vn 11

GV : Le c Thanh

Nh vay, cung co the tnh goc xoay y va o vong y theo ham y=-

Mx
EJ x

ma khong can tch phan. o cung chnh la phng phap tai trong gia
tao.

Phng phap tai trong gia tao:


Tng tng mot dam gia tao (DGT) co chieu dai giong dam thc
M
(DT), tren DGT co tai trong gia tao qgt giong nh bieu o x tren dam
EJ x

that, th se co s tng ng:


y' ' =

Mx
= qgt
EJ x

; y =Qgt ;

y = Mgt

trong o: qgt - Tai trong gia tao


Qgt - Lc cat gia tao- Lc cat trong DGT

M gt - Momen gia tao- Momen uon trong DGT


Muon tnh goc xoay y va o vong y cua mot dam thc (DT)
(dam ang khao sat) th ch can tnh lc cat Qgt va momen uon Mgt do tai
trong gia tao tac dung tren DGT gay ra.
Tuy nhien, e co c s ong nhat ng an hoi y va Momen uon
Mgt th ieu kien bien cua chung phai giong nhau: y = Qgt ; y = Mgt tai bat
ky iem tren hai DT va DGT; Ngoai ra neu xet tai iem bat ky tren dam
phai khao sat en s giong nhau cua bc nhay goc xoay y va bc
nhay lc cat

Qgt .

Cach chon dam gia tao (DGT)


DGT c suy t DT vi ieu kien la ni nao tren DT khong co o
vong va goc xoay th ieu kien lien ket cua DGT nhng ni o phai tng
ng sao cho qgt khong gay ra Mgt va Qgt.
Chieu dai cua DT va DGT la nh nhau.
Bang 8.1 cho mot so DGT tng ng vi mot so DT thng gap.

Chong 8: Chuyen v cua dam chu uon

http://www.ebook.edu.vn 12

GV : Le c Thanh

Cach tm tai trong gia tao qgt


V qgt =

Mx
, nen qgt bao gi cung ngc dau vi momen uon Mx. Do o:
EJ x

- Neu: Mx > 0 th qgt < 0, ngha la neu bieu o Mx nam pha di truc
hoanh (theo qui c Mx > 0 ve pha dc truc thanh) th qgt hng xuong
- Neu: Mx < 0 th qgt hng len.

qgt luon co chieu hng theo th cang cua bieu o mo men Mx

Bang 8.1
Dam thc

Dam gia tao

y = 0
0

y = 0

Mgt = 0

Mgt = 0

Qgt 0

Qgt 0

0
0

y=0

=0

B
y= 0

y 0

y = 0
0

0
tr= ph

y 0

y= 0
0

y= 0
0

y 0
0

Chong 8: Chuyen v cua dam chu uon

Mgt 0
Qgt 0

Mgt = 0
Qgt = 0

Mgt 0
Qgt 0

A
Mgt 0
Qgt 0

Mgt = 0
Qgt 0
Qtr = Qph

Mgt = 0
Qgt 0

Mgt = 0
Qgt 0

Mgt = 0
Qgt 0

D
Mgt 0
Qgt 0

http://www.ebook.edu.vn 13

GV : Le c Thanh

Ngoai ra trong qua trnh tnh cac noi lc Mgt, Qgt cua DGT, can phai
tnh hp lc cua lc phan bo qgt tren cac chieu dai khac nhau. Do o, e
tien li ta xac nh v tr trong tam va dien tch cua nhng hnh gii han
bi cac ng cong nh bang 8.2 di ay
Bang 8.2

()

x1

Lh
2

L
3

2L
3

nh

Lh
3

L
4

3L
4

nh

Lh
n +1

L
n+2

L(n + 1)
n+2

2Lh
3

3L
8

5L
8

2Lh
3

L
2

L
2

C
x1

V tr trong tam

Dien tch

Hnh ve

x2

x2
L

Bac 2

C
x1

x2
L

Bac n

x2

x1
L
nh
h

Bac 2

C
x1

x2
L
nh

C
x1

x2
L

a)

b)

Th du 8.5 Tnh o vong va goc xoay


Chong 8: Chuyen v cua dam chu uon

c)

L
qL2
2
qL2
2EI x

Mx

DGT

http://www.ebook.edu.vn 14
H. 8.10

GV : Le c Thanh

au t do B cua dam cong xon chu tai trong phan bo eu q (H.8.10a). o


cng cua dam EJx = const
Giai.
+ Bieu o momen uon Mx cua DTco dang ng bac 2 c ve tren
H.810b.
+ DGT tng ng vi lc phan bo qgt nh H.8.10c.
+ o vong va goc xoay tai B cua DT chnh bang momen uon Mgt va
lc cat Qgt tai B cua DGT.Dung mat cat sat B cua dam gia tao, tnh noi
lc mat cat ngang nay va c:
qL3
1 qL2
L =
B = QgtB =
;
3 2 EJ x
6 EJ x

yB = M gtB =

Th du 8.6 Tnh o vong


va
goc xoay tai C cua dam
cho
tren H.8.11a. oan dam
AB

1 qL2
3
qL4

L L =
3 2 EJ x
4
8EJ x
qL

S2
a)

3L

qL
8

qL2

b)

co o cng 2EJ, oan

qL2

dam BC co o cng EJ.


Giai.

c)

+ Bieu o momen uon


c ve tren H.8.11b.
e de dang trong
viec tnh toan ta phan tch
Mx thanh tong cua cac
bieu o momen uon co
dang n gian nh
H.8.11c.
+ DGT vi lc
H.8.11d.

qL2
EJx

qL2
2EJx

9qL
8

d)

9qL 2
16EJx

qL2
2EJx

e)

B
Vgt

qgt nh

(chu y la o cng trong


AB va BC khac nhau).

qL
EJx

H. 8.11

B
Vgt

+ Tnh noi lc C cua DGT.


Chong 8: Chuyen v cua dam chu uon

http://www.ebook.edu.vn 15

GV : Le c Thanh

Chia DGTthanh hai DGT nh H.8.11e, phan lc B cua DGT AB la:


VgtB =

1 qL3
16 EJ x

Phan lc nay tac dung len DGT BC va de dang tnh c:


QgtC =

1 qL3 1 qL2
7 qL3
+ L
=+
16 EJ x 2 EJ x
16 EJ x

M gtC =

1 qL3
1 qL2 2
13 qL4
L+ L
L=
16 EJ x
2 EJ x 3
48 EJ x

o vong va goc xoay tai C cua DT


yC =

M gtC

13 qL4
;
48 EJ x

C =

C
Qgt
=

7 qL3
16 EJ x

8.5 BAI TOAN SIEU TNH (BTST)


Tng t cac bai toan ve thanh chu keo, nen ung tam, ta con co cac
BTST ve uon.
o la cac bai toan ma ta khong the xac nh toan bo noi lc hoac phan
lc ch vi cac phng trnh can bang tnh hoc, v so an so phai tm cua bai
toan ln hn so phng tnh can bang tnh hoc co c.
e giai c cac BTST, can tm them mot so phng trnh phu da
vao ieu kien bien dang cua dam.
Xet cu the th du sau:
Th du 8.6 Ve bieu o noi lc cho dam nh H.8.12a. Biet EJ = hang so.
Giai.
+ Dam a cho co bon phan lc can tm (ba ngam A va mot goi ta
B). Ta ch co ba phng trnh can bang tnh hoc, nen can tm them mot
phng trnh phu ve ieu kien bien dang cua dam.
+ Tng tng bo goi ta au B va thay vao o mot phan lc VB
(H.8.12b), ta c mot he mi. He nay ch co the lam viec giong nh he
tren khi VB phai co tr so va chieu the nao e o vong tai B, do tai trong q
va VB sinh ra, phai bang khong
ieu kien bien dang ( chuyen v):

Chong 8: Chuyen v cua dam chu uon

yB (q, VB ) = 0

http://www.ebook.edu.vn 16

GV : Le c Thanh

q
B

a)
L

VB =
q
B

b)

h)

5
qL
8

i)

Qy

VB
c)

d)

k)

qL2
2

3
qL
8

3
qL
8

1 2
qL
8

Mx

9qL2
128

VBL

+ Ta tnh o vong tai B bang phng phap tai trong gia tao (hay mot
phng phap khac).
Bieu o momen uon cua dam H.8.12b do tai trong q va phan lc VB
gay ra ve nh H.8.12c,d, DGT va qgt nh H.8.12 e, g. Ta co:
o vong yB cua he 8.12b chnh la Momen gia tao tai B cua DGT
yB = M Bgt =

2
1
L qL 3
3 2 EJ 4

ieu kien o vong yB = 0, VB =

1
2

L VB L
EJ

2
L
3

3
qL
8

Sau khi tm c VB, de dang ve c cac bieu o noi lc cua dam a


cho nh H.8.12 i, k.

7.4. PHNG PHAP DIEN TCH MOMEN


1. Noi dung phng phap
Chong 8: Chuyen v cua dam chu uon

http://www.ebook.edu.vn 17

GV : Le c Thanh

Mx
nh H.8.10b, ng an hoi (net t) nh H.8.10a.
EI x

Xet dam co bieu o

A
a)

H
yB

yA

zA

z
dz

zB

LAB
zC

zC

b)

Mx
EI x

Mx
dz
EI x

S AB

H.8.10 Phng phap dien tch mo men


ZB
ZB
M
M
Xet oan dam AB: d = x dz , suy ra: Z d = Z x dz
A
A
EI x
EI x

B A = AB = S AB
vi S AB la dien tch cua bieu o

(8.18)

Mx
gom gia hai mat cat A va B.
EI x

nh ly 1. o thay oi goc xoay gia hai mat cat cua mot dam (th du gia A va B)
th bang dau tr dien tch cua bieu o
T hnh 8.10d:

dt = z d = z

Mx
gia hai mat cat ay.
EI x

ZB
ZB
Mx
M
dz suy ra: t BA = dt = z x dz = z C S AB (8.20)
ZA
ZA
EI x
EI x

z C la khoang cach t trong tam cua dien tch S AB en B

nh ly 2. o sai lech gia tiep tuyen mot iem B tren ng an hoi vi mot
tiep tuyen mot iem A khac cung tren ng an hoi bang vi dau tr mo men
Chong 8: Chuyen v cua dam chu uon

http://www.ebook.edu.vn 18

GV : Le c Thanh

tnh cua dien tch cua bieu o

Mx
oi vi ng thang ng i qua B.
EI x

T H.8.10d ta co:
yB = yA + ALAB + tBA = yA + A(zB zA) + tBA

yB = yA + A(zB zA) z C S AB

(8.21)

(7.21) chnh la cong thc dung e xac nh o vong cua iem B neu biet o vong
cua mot iem A (zB > zA) va bieu o

Mx
gia hai iem nay.
EI x

T (8.21 co the tnh o vong cua iem A khi biet o vong cua iem B (zB > zA).
A = B + S AB va yA = yB A(zB zA) + z C S AB
vi:

z C = L AB z C

ta viet:

y A = y B B + S AB L AB + (L AB z C ) S AB

Khai trien va rut gon, ta c:

yA = yB BLAB zC S AB

(8.22)

zC - la khoang cach t trong tam C cua S AB ke t A.


Th du 8.5. Dung phng phap dien tch mo men xac nh goc xoay au trai A va
o vong iem D gia dam (H.8.11). EIx = hang so.
Giai. Theo nh ly 1, cong thc (7.4), xet hai iem
A (z = 0) va D (z = L/2)
D = A S AD

Chu y rang D = 0 v bai toan oi xng va S AD co


the phan chia thanh S 1 + S 2 + S 3 .
A (S 1 + S 2 + S 3 ) = 0
ta suy ra:
A = S1 + S 2 + S 3 =

13 qL3

648 EI x

Goc xoay cua mat cat A thuan chieu kim ong ho.
Ap dung cong thc (8.21), ta viet
yD = y A + A

(1)
(2 )
(3 )
qL2
13 qL3 L
77
L
z C S AD = 0 +

zC S1 + zC S 2 + zC S 3 =

2
648 EI x 2
11664 EI x

BAI TAP CHNG 8

Mo
2m

Chong 8: Chuyen v cua dam chu uon

6m

http://www.ebook.edu.vn 19
H.8.1

GV : Le c Thanh

8.1 Xac nh ng an hoi dam bang phng phap tch phan khong nh
han, biet Mo = 20 kNm, EJ khong oi. H.8.1.
8.2

Xac nh goc xoay hai au dam va o vong tai gia dam bang
phng phap tch phan khong nh han, EJ khong oi. H.8.2.

8.3 Dam mat cat ngang thay oi va chu lc


nh H.8.3. Tnh o vong tai dam t do va
goc xoay tai mat cat ngang gia dam.

q
h
C

8.4 Dam co o cng khong oi nh H.8.4.


Xac nh:
- o vong va goc xoay tai C

L/2L/2
b

H. 8.2B

L/2

L/2

H. 8.3 4qa

H.8.4

8.6 Tm o vong tai B, goc xoay tai A cua


dam nh H.8.6, biet EJ= hang.

8.5 Tm o vong tai mat cat C, goc xoay ben


trai va phai khp A cua dam nh H.8.5,
biet o cng EJ = hang .

C
a

H. 8.5
P

B
a

8.7 Xac nh o vong va goc xoay tai C. H.8.7

H. 8.6

40 kN
A

- o vong tai mat cat D

qa

- Goc xoay tai A va B

L/2L/2

2EJ

3m

EJ

1m

H. 8.7

8.8 Mot he thong gom ba cong xon, au t


do c lien ket vi nhau bang nhng
giang cng nh H.8.8. Tnh ng suat
cc ai moi dam khi co lc treo
Chong 8: Chuyen v cua dam chu uon

http://www.ebook.edu.vn 20
H. 8.8

GV : Le c Thanh

dam, biet o cng EJ la hang so.


8.9 Ve bieu o noi lc cua dam sieu tnh nh H.8.9. Viet phng trnh
ng an hoi, biet o cng EJ la hang so.

Mo

L/2

H. 8.9

EJ = ha ng so

L/2

H. 8.10

8.10. Xac nh phan lc cua dam sieu tnh nh H.8.10.


8.11. Thanh thep dai 1 m, mat cat ch nhat 2036 mm, ngam au A, chu
lc P = 30 N at gia nhp. Kiem tra o ben cua dam.
Biet [] = 16 kN/cm2. au B co khe h = 20 mm.
Cho E = 2.104 kN/cm2.
P

6 mm

0,5 m

0,5 m

20 mm

H. 8.11

8.5. PHNG PHAP DIEN TCH MOMEN (DTMM)


1. Noi dung phng phap

Chong 8: Chuyen v cua dam chu uon

http://www.ebook.edu.vn 21

GV : Le c Thanh

Xet dam chu uon co bieu o

Mx
nh H.8.13b, ng an hoi (net t)
EJ x

nh H.8.13a.

A
a)

yA

yB

y
z

z
dz

zB

zC

zC

b)

Mx
EJ x

c)

dz

Mx
dz
EJ x

S AB

dt

A
yA

d)

yB

tBA
B

H.8.13

Xet oan dam AB, ta a co:


y" =

Mx
EJ x

ZB

ZA

dy ' d
M
=
= x
dz
dz
EJ x

d =

ZB

ZA

d =

Mx
dz
EJ x

Mx
dz
EJ x

Chong 8: Chuyen v cua dam chu uon

http://www.ebook.edu.vn 22

GV : Le c Thanh

(8.4)

B A = AB = S AB

vi

S AB

Mx
gom gia hai mat cat A va B.
EJ x

la dien tch cua bieu o

nh ly 1. o thay oi goc xoay gia hai mat cat cua mot dam (th du gia
A va B) th bang dau tr dien tch cua bieu o

Mx
gia hai mat cat ay.
EJ x

T H.8.13c ta co the viet:


dt = z d = z

suy ra:
zC

t BA =

ZB

ZA

dt =

Mx
dz
EJ x
ZB

ZA

Mx
dz = zC S AB
EJ x

la khoang cach t trong tam cua dien tch

S AB en

nh ly 2. o sai lech gia tiep tuyen mot iem B tren ng an hoi vi


mot tiep tuyen mot iem A khac cung tren ng an hoi bang vi dau
tr momen tnh cua dien tch cua bieu o

Mx
oi vi ng thang ng i
EJ x

qua B.
T H.8.13d ta co:
yB = yA + ALAB + tBA
= yA + ALAB

(8.5)

z C S AB

(8.5) chnh la cong thc dung e xac nh o vong cua iem B neu biet o
vong cua mot iem A (zB > zA) va bieu o

Mx
gia hai iem nay.
EJ x

T (8.5) ta cung co the tnh o vong cua iem A khi biet o vong cua
iem B (zB > zA). That vay theo phan tren ta co:
A = B + S AB

va:

yA = yB ALAB +

vi:

z C = L AB z C

ta viet:

y A = y B B + S AB L AB + (L AB zC ) S AB

z C S AB

Khai trien va rut gon, ta c:


yA = yB BLAB zC S AB

(8.5)

trong o: zC - la khoang cach t trong tam C cua

Chong 8: Chuyen v cua dam chu uon

S AB

ke t A.

http://www.ebook.edu.vn 23

GV : Le c Thanh

Dung phng phap DTMM can biet dien tch va trong tam cua mot so
hnh ( bang 8.2 ).
Th du 8.7. Dung phng phap DTMM xac
nh goc xoay au trai A va o vong iem

B
D

D gia dam (H.8.14). EJx = hang so.

L/3

L/3

Giai.

L/3

+ Theo nh ly 1, cong thc (8.4), xet hai iem

Mx
EJ x

A (z = 0) va D (z = L/2) :

S3

D = A S AD

Chu y rang D = 0 v bai toan oi xng va


co the phan chia thanh S 1 + S 2 + S 3 (H.8.14).

5 qL2
72 EJ x
H.8.14

S AD

4 qL2
72 EJ x

A (S 1 + S 2 + S 3 ) = 0

A = S1 + S 2 + S 3
=

4 qL2 1 L 4 qL2 L 2
qL2
L 13 qL3

+
+
=

72 EJ x 2 3 72 EJ x 6 3 72 EJ x 6 648 EJ x

Goc xoay cua mat cat A thuan chieu kim ong ho.
+ Ap dung cong thc (8-5), ta viet
yD = y A + A

= 0+

L
z C S AD
2

(2 )
(3 )
13 qL3 L (1)

z C S 1 + z C S 2 + z C S 3

648 EJ x 2

13 qL3 L L 1 L 1 4qL2 L

+
+
648 EJ x 2 6 3 3 2 72 EJ x 3
+

1 L L 4qL2
3 L 2
qL2
L

+

2 6 6 72 EJ x 8 6 3 72 EJ x 6
=

77
qL2

11664 EJ x

3EJ

EJ

L/2

L/2

Mx

o vong mat cat D hng xuong di.


Th du 8.8
Xac nh goc xoay A,B va o vong D
cua dam cho nh H.8.15
Chong 8: Chuyen v cua dam chu uon

S1 =

1 qL3
72 EJ x

1 2
qL
8

S2 =

1 qL3
24 EJ x M x
EI x

1 qL
1 qL2
24 EJ x
http://www.ebook.edu.vn
24
8 EJ x
H.8.15

GV : Le c Thanh

Giai
+ Bieu o mo men uon Mx va

Mx
EJ

ve nh H.8.15
+ Theo cong thc 8.5, ta co:
yB

= yA + AL

z C S AB

= 0 + AL

zC

A =

1
L

( zC S 1 +
(1)

(1)

( 2)

zC

S 1

( 2)

zC

S 2

S 2)

3 L L
5L
qL3
qL3
= 1
+
+

=
L
82 2
72 EJ
8 2 24 EJ

13 qL
576 EJ x

+ Bay gi ap dung nh ly 1, cong thc (8.4)

B = A
=

= A

S AB
3

13 qL
576 EJ x

S1

S2

qL3
qL3
qL3

= 19
576 EJ x
72 EJ x
24 EJ x

Goc xoay mat cat B ngc chieu kim ong ho.


+ Cuoi cung xac nh o vong D bang cong thc 8.5 ap dung cho hai
iem A va D
yD = yA + A
=0+

13
576

L
2

z C S AD

qL3 L
qL3
qL4
3 L
= 5
8 2 72 EJ x
576 EJ x
EJ x 2

+ Ta co the kiem tra lai ket qua cua yD bang cach khao sat oan DB,
ap dung (8.5)
yD = yB B

L
2

ZC

S BD

19 qL3 L

= 0

576

EJ x

Chong 8: Chuyen v cua dam chu uon

3
L
8
2

qL3
qL4
= 5
576
24 EJ x
EJ x

http://www.ebook.edu.vn 25

GV : Le c Thanh

a)

8.5 BAI TOAN SIEU TNH


(BTST)
Tng t cac bai toan
ve thanh chu keo, nen ung
tam, ta con co cac BTST ve
uon.
o la cac bai toan ma ta
khong the xac nh toan bo
noi lc hoac phan lc ch
vi cac phng trnh can
bang tnh hoc, v so an so
phai tm cua bai toan ln hn
so phng tnh can bang
tnh hoc co c.
e giai c cac BTST,
can tm them mot so phng
trnh phu da vao ieu kien
bien dang cua dam.
Xet cu the th du sau:
Th du 8.10. Ve bieu o noi
lc cho dam nh H.8.16a.

b)

A
VB

c)

d)

qL2
2

VBL

e)

qL2
2 EJ

g)

VB L
EJ

q
B

h)

VB =

i)

5
qL
8

k)

1 2
qL
8

Chong 8: Chuyen v cua dam chu uon

3
qL
8
Qy

3
qL
8
Mx

9qL2

http://www.ebook.edu.vn
128
26
H.8.16

GV : Le c Thanh

Biet EJ = const.
Giai
Giai.
+ Dam a cho co 4 phan lc can tm (ba ngam A va mot goi ta B).
Ta ch co 3 phng trnh can bang tnh hoc, nen can tm them 1 phng
trnh phu ve ieu kien bien dang cua dam.
+ Tng tng bo goi ta au B va thay vao o mot phan lc VB
(H.8.12b), ta c mot he mi. He nay ch co the lam viec giong nh he
tren khi VB phai co tr so va chieu the nao e o vong tai B, do tai trong q
va VB sinh ra, phai bang khong
ieu kien bien dang ( chuyen v):

yB (q, VB ) = 0

+ Ta tnh o vong tai B bang phng phap tai trong gia tao (hay mot
phng phap khac).
Bieu o momen uon cua dam H.8.16b do tai trong q va phan lc VB
gay ra ve nh H.8.16c,d, DGT va qgt nh H.8.16 e, g. Ta co:
o vong yB cua he 8.16b chnh la Momen gia tao tai B cua DGT
yB = M Bgt =

2
1
L qL 3
3 2 EJ 4

L VB L

1
2

ieu kien o vong yB = 0, VB =

EJ

2
L
3

3
qL
8

Sau khi tm c VB, de dang ve c cac bieu o noi lc cua dam a


cho nh H.8.16 i, k.
Th du 8.11. Tnh phan lc VB
cua dam sieu tnh nh H.8.17a.

Cho biet : EJx = hang


Giai.
Tng t th du tren, cung

b
L
P

co ieu kien yB = 0

b)

Tnh yB bang phng phap


dien tch mo men
Bieu o Mx/ EJx do tai trong

a)

A
B

c)

P va phan lc VB c ve
H.8.17c
Ap dung cong thc (8.5), ta
Chong 8: Chuyen v cua dam chu uon

VB

Pa
EJ x

Mx
EJ x

VB L
EJ x
H.8.17

http://www.ebook.edu.vn 27

GV : Le c Thanh

co:
yA = yB AL +

z S AB

a 1 Pa 2 1 V L
0 = yB 0L + L a + L L B

yA =

3 2 EJ

3 2

EJ

Pa 2 3L a VB L3

+
2 EJ 3 3EJ

ieu kien yB = 0 cho ta


0=
suy ra

VB =

Pa 2 3L a VB L3

+
2 EJ 3 3EJ

Pa 2
(3L a )
2 L3

BAI TAP CHNG 8


8.1. Xac nh ng an hoi dam bang phng phap tch phan khong nh
Mo
6m

2m

H.8.19

han, biet Mo = 20 kNm


8.2. Xac nh goc xoay hai au dam va o vong tai gia dam bang
phng phap tch phan khong nh han
8.3. Dam mat cat ngang thay oi va chu lc nh H.8.21. Tnh o vong tai
dam t do va goc tai mat cat ngang gia dam.
h
C

EI

L/2

2EI

L/2

L/2

L/2

H.8.20

H.8.21
Chong 8: Chuyen v cua dam chu uon

http://www.ebook.edu.vn 28

GV : Le c Thanh

8.4. Dam co o cng khong oi. Xac nh:


- o vong va goc xoay tai C
- Goc xoay tai A va B
- o vong tai mat cat D
q

4qa
qa2
B

C
a

H.8.22

8.5. Tm o vong tai mat cat C, goc xoay ben trai va phai khp A cua dam
P
A

C
a

nh H.8.23.
H.8.23
P
A

C
a

8.6. Tm o vong tai B, goc xoay tai A cua dam nh H.8.24.


H.8.24
8.7. Xac nh o vong va goc xoay tai C
A

2EI

40 kN

EI

C
3m

1m

H.8.25
8.8. Mot he thong gom ba cong xon, Dam t do c lien ket vi nhau
Chong 8: Chuyen v cua dam chu uon

http://www.ebook.edu.vn 29

GV : Le c Thanh

bang nhng gang cng. Tnh ng suat cc ai moi dam khi co lc P


treo dam

H.8.26

8.9. Ve bieu o noi lc cua dam sieu tnh nh H.8.27. Viet phng trnh
ng an hoi.
q

Hnh
7.9
H.8.27

8.10. Xac nh phan lc cua dam sieu tnh nh H.8.28.


Mo
EI = hang so
L/2

L/2

H.8.28

8.11. Thanh thep dai 1 m, mat cat ch nhat 2036 mm, ngam dam A, chu
lc P = 30 N at gia nhp. Kiem tra o ben cua dam.
Biet [] = 16 kN/cm2. dam B co khe h = 20 mm, cho E = 2.105
P
A

60 mm

0,5 m

0,5 m

Chong 8: Chuyen v cua dam chu uon

20 mm

http://www.ebook.edu.vn 30

GV : Le c Thanh

MN/m2
H.8.29

Chong 8: Chuyen v cua dam chu uon

http://www.ebook.edu.vn 31

GV: Le c Thanh

Chng 9

XOAN THUAN TUY

. KHAI NIEM

Mz

1- nh ngha: Thanh chu xoan thuan tuy


khi tren cac mat cat ngang ch co mot thanh

phan noi lc la momen xoan Mz (H.9.1).

Dau cua Mz : Mz > 0 khi t ngoai mat cat


nhn vao thay Mz quay thuan kim ong ho

H. 9.1

Ngoai lc: Gom cac ngau lc, momen


xoan Mz, nam trong mat phang vuong goc truc thanh.
Thc te: truc truyen ong, thanh chu lc khong gian, dam ovang...
2- Bieu o noi lc momen xoan Mz
Bieu o momen xoan c ve bang cach xac nh noi lc theo phng
phap mat cat va ieu kien can bang tnh hoc: M/OZ = 0.
Th du 1: Ve bieu o Mz cho truc truyen ong chu tac dung cua ba
ngau lc xoan ( momen xoan)
(H.9.2.a).
M1=10kNm

a)

M2=7kNm

Mz

C
b)

10 kNm
+
d)

M1=10kNm

M3=3kNm

H.9.2

3 kNm

Mz

M1=10kNm

M2=7kNm
Mz

c)

Giai: Thc hien mot mat cat ngang trong oan AB, xet can bang phan
trai (H.9.2.b), de thay rang e can bang ngoai lc la ngau lc xoan M1 , tren
tiet dien ang xet phai co noi lc la momen xoan Mz :
M /z = 0 Mz 10 = 0 Mz = 10kNm
Tng t, cat qua oan BC, xet phan trai (H.9.2.c):
M /z = 0 Mz + 7 10 = 0 Mz = 3
Momen tai cac tiet dien cua hai oan au thanh bang khong, bieu o
noi lc ve H.9.2.d.
Chng 9: XOAN THUAN TUY

http://www.ebook.edu.vn

GV: Le c Thanh

Th du 2: Ve bieu o momen xoan Mz (H.9.3.a)


Giai: Phan tch thanh tong
cua hai trng hp tac dung
rieng le ( H.9.3b va H.9.3c ).
Trong moi trng hp,
ngoai lc la mot ngau lc gay
xoan, do o noi lc trong
thanh cung la momen xoan.
Bieu o noi lc cua tng
thanh ve ngay tren H.9.3.b,c.

M 1 = 8 kNm

M 1 = 5 kNm

a)

c)

Mz = 5

b)

Mz = 8

Mz = 5

+
d)

Bieu o Mz cua thanh la tong


ai so hai bieu o tren
(H.9.3.d).

Mz = 3

M z (kNm)

H.9.3

Nhan xet: Dau cua noi lc la dng khi t ngoai nhn vao au
thanh thay ngoai lc quay thuan chieu kim ong ho va ngc lai.
3- Cong thc chuyen oi cong suat ong c ra ngau lc xoan
(momen xoan ngoai lc) tren truc
Khi tnh toan cac truc truyen ong, thng ta ch biet cong suat truyen
cua mot tnh bang ma lc hay kilooat va toc o truc quay bang vong/phut,
do o can chuyen oi cong suat truyen ra ngau lc xoan tac dung len truc.
Gia s co mot ngau lc xoan Mo (n v la N.m) tac dung lam truc quay
mot goc (radian) trong thi gian t, cong sinh ra la:
A = Mo.

(i)

M o
A

=
= Mo
= M o
W =
t
t
t

cong suat la:

(ii)

trong o: - la van toc goc (rad/s), n v cua cong suat la N.m/s.


Goi n la so vong quay cua truc trong mot phut (vong/phut), ta co:
=

2n n
=
60
30

(iii)

t (ii) va (iii)
a)

Neu W tnh bang ma lc (CV, HP) ;1ma lc = 750N.m/s = 0,736 kW:


Mo =

b)

30W 30.750.W
W
=
= 7162 ( Nm)
n
n
n

(9.1)

Neu W tnh bang kilowat (KW), 1 KW 1020 N.m/s:


Mo =

30W 30.1020.W
W
=
= 9740 (Nm)
.n
n
n

Chng 9: XOAN THUAN TUY

(9.2)

http://www.ebook.edu.vn

GV: Le c Thanh

. XOAN THUAN TUY THANH THANG TIET DIEN TRON


Mz

Mz

b)

a)

H. 9.4

dz

1- Th nghiem - Nhan xet


Lay mot thanh thang tiet dien tron, tren mat ngoai co vach nhng
ng song song va nhng ng tron thang goc vi truc, tao thanh li
o vuong (H.9.4.a). Tac dung len hai au thanh hai ngau lc xoan Mz ngc
chieu, ta thay truc thanh van thang, chieu dai thanh khong oi, nhng
ng tron thang goc vi truc van tron va thang goc vi truc, nhng ng
song song vi truc thanh nhng ng xoan oc, li o vuong thanh li bnh
hanh (H.9.4.b).
2- Cac gia thiet
a) Mat cat ngang van phang, thang goc vi truc thanh va khoang cach
khong oi trong qua trnh bien dang,
b) Cac ban knh van thang va khong oi trong qua trnh bien dang,.
c) cac th doc khong ep va ay lan nhau trong qua trnh bien dang.
3- Cong thc ng suat tiep
Ta tnh ng suat tai mot iem bat ky tren mat
cat ngang co ban knh (H.9.1).
Co the nhan thay, theo th nghiem tren, bien
dang cua thanh chu xoan thuan tuy ch la s xoay
tng oi gia cac mat cat ngang quanh truc.

Mz

H. 9.1

e xet bien dang xoan cua mot phan to tai mot iem bat ky ban knh
trong thanh, ta tach phan to bang ba cap mat cat nh sau:

Chng 9: XOAN THUAN TUY

http://www.ebook.edu.vn

GV: Le c Thanh

d
Mz

Mz

H. 9.6
Phan to trt thuan tuy

dz

1 dz 2

b)

a)

H. 9.5 Bien dang cua phan to chu xoan


- Hai mat cat (1-1) va (2-2) thang goc vi truc cach nhau oan dz
(H.9.5.a).
- Hai mat cat cha truc hp vi nhau mot goc d be(H.9.5.b).
- Hai mat cat hnh tru ong truc z (truc thanh) ban knh va + d
(H.9.5.a).
Theo cac gia thiet, trong qua trnh bien dang, so vi cac iem E, F, G,
H thuoc mat cat (1-1), cac iem A, B, C, D cua phan to tren mat cat (2-2) di
chuyen en A, B, C, D phai nam tren cung tron ban knh va + d,
ong thi OAB va OCD phai thang hang.
Goi d la goc gia hai ng thang OAB va OAB, o la goc xoay cua
mat cat (2-2) so vi mat cat (1-1) quanh truc z, d cung chnh la goc xoan
tng oi gia hai tiet dien lan can cach nhau dz.
oi vi phan to ang xet, goc AEA bieu dien s thay oi goc vuong
cua mat ben phan to goi la bien dang trt (goc trt) cua phan to.
T (H.9.5.b), ta co:
tan =

Chng 9: XOAN THUAN TUY

AA
d
=
EA
dz

(a)

http://www.ebook.edu.vn

GV: Le c Thanh

Theo gia thiet a) khong co bien dang dai theo phng doc truc, theo
gia thiet c) cac th doc khong tac dung vi nhau nen khong co ng suat
phap tac dung len cac mat cua phan to.
Theo gia thiet a) cac goc vuong cua mat CDHG va mat BAEF khong
thay oi nen khong co ng suat tiep hng tam tren mat A, B, C, D. Do gia
thiet b), moi ban knh van thang nen khong co ng suat tiep hng tam tren
mat A, B, E, F.
Nh vay, tren mat cat ngang cua thanh chu xoan thuan tuy ch ton tai
ng suat tiep theo phng vuong goc ban knh, goi la va phan to ang xet
trang thai trt thuan tuy (H.9.6).
Ap dung nh luat Hooke ve trt cho phan to nay, ta co:

= G

b)

(a) vao (b)


p = G

d
dz

(c)

Goi dF la mot dien tch vo cung be bao quanh iem ang xet, th .dF
la lc tiep tuyen tac dung tren dien tch o va .dF. la momen cua lc

dF oi vi tam O. Tong cac momen nay phai bang Mz, nen ta co the viet:
(d)

M z = p dF
F

(c) vao (d)

M z = G
F

d
dF
dz

(e)

V G.d/dz la hang so oi vi moi iem thuoc mat cat F, nen ta co the


a ra ngoai dau tch phan, khi o tch phan 2..dF chnh la momen quan
F

tnh cc Jp cua mat cat ngang oi vi tam O, ta c:


Mz = G

d
d
2 dF = G
Jp

dz
dz F

(f)

d
Mz
=
dz GJ

t (f) ta co:

(g)

Co the thay rang, d/dz chnh la goc xoan tren mot n v chieu dai
( con goi la goc xoan t oi ) (rad/m). at
=

Mz
GJ

d
dz

, ta co:
(9-3)

thay (g) vao (c) ta c cong thc tnh ng suat tiep:

Chng 9: XOAN THUAN TUY

http://www.ebook.edu.vn

GV: Le c Thanh

Mz

(9.4)

ng suat tiep thay oi theo quy luat bac nhat, bang khong tai tam O va
cc ai tai nhng iem tren chu vi.
Bieu o phan bo ng suat tiep tai moi iem tren mat cat ngang the hien
tren H.9.7.a. Tren H.9.7.b, the hien ng suat tiep oi ng tren cac mat cat
cha truc.

Mz

Mz

O
max
a)

b)

max

H.9.7. Phan bo ng suat tiep tren mat cat

Va ng suat tiep oi ng

ng suat tiep cc ai cac iem tren chu vi ( = ban knh R)


max =

at:

W =
max =

J
R

Mz
R
J

; Wp goi la momen chong xoan cua mat cat ngang


(9.5)

Mz
W

* Vi tiet dien tron ac va D la ng knh tiet dien:


W =

J
R

R 3
2

D 3
16

(9.6)

0,2 D 3

* Vi tiet dien tron rong:


W =

J
R

D 4 (1 4 ) 1
32

D 3
16

(1 4 ) 0,2 D 3 (1 4 )

(9.7)

trong o: la ty so gia ng knh trong va ng knh ngoai ( = d/D).

Chng 9: XOAN THUAN TUY

http://www.ebook.edu.vn

GV: Le c Thanh

4- Cong thc tnh bien dang khi xoan


Goc xoan tng oi gia hai mat cat cach nhau dz la d =

Mz
dz
GJ

(g)

Goc xoan tng oi gia hai mat cat cach nhau mot oan dai L la:
L

Mz
dz
o GJ

(9.8)

= d =
o

Khi oan thanh co Mz/GJp la hang so =

Khi thanh gom nhieu oan, moi oan co Mz/GJp la hang so:
= (
i

M zL
GJ p

MzL
)i
GJ

(9.9)

(9.10)

Goc xoan c quy c dng theo chieu dng cua Mz .


5- Tnh toan thanh tron chu xoan thuan tuy:
ieu kien ben:
+

max [ ]

= o

(9.11)

vi: o - la ng suat tiep nguy hiem cua vat lieu, xac nh t th nghiem
n - la he so an toan.
+ Theo thuyet ben ng suat tiep ( chng 5 ):
max

[ ]
2

(9.12)

+ Theo thuyet ben the nang bien oi hnh dang ( chng 5 ):


max

[ ]
3

(9.13)

ieu kien cng:

max [ ]

(9.14)

[ ] : Goc xoan ty oi cho phep, c cho t cac so tay ky thuat, n v


cua [ ] la (radian/ n v chieu dai )
Ba bai toan c ban:
- Kiem tra ben, cng (bai toan kiem tra)
- Xac nh tai trong cho phep
- Xac nh ng knh (bai toan thiet ke).

Chng 9: XOAN THUAN TUY

http://www.ebook.edu.vn

GV: Le c Thanh

6- The nang bien dang an hoi


The nang rieng tch luy trong mot n v the tch la:
u=

1
[ 12 + 22 + 32 2 ( 1 2 + 2 3 + 3 1 )]
2E

Thanh chu xoan thuan tuy, TTS trt thuan tuy vi ng suat tiep , nen
1 = ; 2 = 0 va 3 = , ta c:
u=

1+ 2

(a)

vi: E = 2 G/(1 + ), thay vao (a), ta c:


1
2 G

u=

(b)

The nang tch luy trong mot oan dz la:


(c)

dU = udV = udFdz
V

thay (b) vao (c), ta c:


dU =

1p
1 M z2 2 dF .dz
1 M z2
=

=
dz 2 dF
F 2 G F 2 J p2
G
2G J p2 F

dU =

1 M z2
dz
2 GJ p

hay:

(d)

Vay the nang tren oan thanh co chieu dai L la:


L

U =

1 M z2
dz
2 o GJ p

(9.15)

+ Khi oan thanh co Mz/GJp la hang so U =

1 M z2 L
2 GJ p

(9.16)

+ Khi thanh gom nhieu oan, moi oan co Mz/GJp la hang so


U =

M z2 L
1
(
)i

2 i GJ p

Chng 9: XOAN THUAN TUY

(9.17)

http://www.ebook.edu.vn

GV: Le c Thanh

7- Dang pha hong cua cac vat lieu


3

max
1

a)

P
b)

H. 9.8 Trang thai ng suat ta i mo t iem


tre n ma t ngoa i cua thanh chu xoan
Nghien cu trang thai ng suat cua truc tron chu xoan, ta thay tai mot
iem tren mat ngoai, phan to trang thai trt thuan tuy chu ng suat tiep
cc ai max (H.9.a), trang thai nay, theo hai phng nghieng 45o so vi
truc co ng suat keo chnh va ng suat nen chnh 1 = 3 = (H.9.8.b).
Mat khac, qua th
nghiem, ta cung biet
H. 9.9 Dang n t gay cua va t lieu de o
rang vat lieu deo (nh
thep) chu keo, chu nen tot nh nhau, con chu cat th kem hn, do o, khi
mot truc thep b xoan se b gay theo mat cat ngang, do ng suat tiep max
tren mat cat ngang (H.9.9).
Vi vat lieu don nh
H. 9.10 Dang nt gay cua vat lieu don
gang, chu nen va chu
cat rat tot, con chu
keo rat kem nen khi xoan se b gay theo mat nghieng 45o so vi truc do ng
suat keo chnh 1 (H.9.10).
Vi vat lieu co cau tao th nh go, chu cat doc th rat kem nen khi
xoan se b nt doc theo ng sinh do ng suat ng suat tiep oi ng vi
ng suat tiep tren mat cat ngang (H.9.11).
Mz

Mz
H. 9.11 Dang nt gay cua go chu xoan

Chng 9: XOAN THUAN TUY

http://www.ebook.edu.vn

GV: Le c Thanh

Th du 9.3 Mot ong c cong suat 10kW, truyen mot momen xoan len mot
truc tron ng knh D tai tiet dien A, van toc truc n = 1400 vg/phut. Gia s
hieu suat truyen la 100%. Khi o tai tiet dien B, C nhan c cong suat
truyen 3kW va 7kW (H.9.12.a). nh ng knh D, sau o tnh goc xoan

AC . Biet: [] = 16 kN/cm2 ; [ ] = 0,250/m; a = 50cm; G = 8.103 kN/cm2.


Giai.
Goi ngau lc xoan tac dung tai A, B, C lan lt la M1, M2, M3. Ap
dung cong thc chuyen oi, ta c:
M1 = 9740 x 10 / 1400 = 69,57 N.m = 6957 Ncm
M2 = 9740 x 3 / 1400

= 20,87 N.m = 2087 Ncm

M3 = 9740 x 7/ 1400

= 48,70 N.m = 4870 Ncm

S o tnh cua truc (H.9.12.b), bieu o momen ve (H.9.12.c).


nh ng knh D:
+ Theo ieu kien ben max [ ] =
vi: [] =

[]
=
2

[ ] M z
2

8 kN/cm2 ;

Mz = 4870 Ncm
D 14,49 cm
(a)
+ Theo ieu kien cng:
max [ ]
D4

Wp

Mz
Mz
=
[ ]
GJ p G.0,1D 4

Mz
Mz
[ ] D 3
3
0,2 D
0,2.[ ]
10 KW

3 KW
B

a)

7 KW
A

20,87 Nm
B

b)

69,57 Nm

48,70 Nm

D4

Mz
G.0,1.[ ]

Mz
G.0,1.[ ]

vi: [ ] = 0,250/m
0,25
=
rad / cm ;
180 10 2

c)

+
20,87

Mz = 4870 Ncm;
G = 8.103 kN/cm2 D 11,17cm

H. 9.12

48,70

Mz (N.m)

(b)

e thoa ca hai yeu cau (a), (b), ta chon D = 15 cm.


Tnh goc xoan AC: Ap dung cong thc (9.6), ta c:
AC =

M zL
4870 50
=
= 0,006 rad

8 103 0,1 154


p i

GJ
i

Chng 9: XOAN THUAN TUY

http://www.ebook.edu.vn

10

GV: Le c Thanh

Th du 9.4 Mot thanh tiet dien tron


ng knh D hai au ngam chu
lc nh (H.9.13). Ve bieu o Mz va

MA

Mo
B

nh gia tr Mo theo ieu kien ben.

a/2

ME

Mo

2Mo

a/2

H. 9.13
Giai: Ngoai lc la momen
xoan trong mat phang thang goc vi truc thanh th phan lc phat sinh tai
cac lien ket ngam A va E phai la cac momen xoan MA, ME trong cac mat
phang thang goc vi truc thanh. Gia s MA, ME co chieu nh tren H.9.13.
e xac nh momen phan lc, viet phng
M
2M
2
M
M
Mo

Mo

B
a/2
/2
a

E
a/2
a/2

a)

trnh can bang M/z = 0, ta co:

MA - Mo +2Mo + Mo - ME =0
(a)
Phng trnh (a) khong u e nh c phan

ME

Mo

b)
2M
2Mo

lc MA, ME : Bai toan sieu tnh.


Can bo sung mot (hay nhieu) phng trnh
thiet lap t ieu kien bien dang cua bai toan
(phng trnh ieu kien bien dang).
Thng cach giai nh sau:

Mo
(4/3)M
(4/3)M0

c)

Mz

(5/3)M
(5/3)M0

(2/3)M
(2/3)
M0

+Tng tng bo ngam E, thay bang phan lc


tng ng ME (H.9.15.a).
+Viet phng trnh ieu kien bien dang: E = 0

Hnh
9.15
H.
9.15

(Tai E lien ket ngam do o goc xoay E = 0 )


+Tnh E : Ap dung nguyen ly cong tac dung, bieu o momen xoan do
tng trng hp tai gay ra c ve H.9.15.b. Tnh E theo (9.10) nh sau:
E = EA = (
i

M zL
M .3a M o 5a 2 M o 3a M o a
+
+

) = E
GJ p
G.J p
GJ p 2 GJ p 2 GJ p 2

+ Cho E = 0, ta c :

ME =

5
Mo
3

Ket qua dng, ME ung chieu chon.


+ Xac nh c ME , ta ve c bieu o momen xoan Mz nh H.9.15.c.
T bieu o noi lc Mz, ta thay: Mz,max= (5/3)Mo.
T ieu kien ben, ta co:

max []

M z max
[]
0,2.D 3

5M o
3.0,2.D3
[] M o []
3
5
3.0,2.D

Chng 9: XOAN THUAN TUY

http://www.ebook.edu.vn

11

GV: Le c Thanh

. XOAN THANH THANG TIET DIEN CH NHAT

Th nghiem xoan thanh tiet dien ch nhat, bien


dang cua thanh nh (H.9.16).
Ly thuyet an hoi cho cac ket qua nh sau:
ng suat: Tren mat cat ngang ch co ng suat tiep.
+ Tai tam va cac goc, ng suat tiep bang khong.
+ Tai iem gia canh dai, ng suat tiep at gia tr ln

nhat :

max =

Mz
hb 2

(9.18)

a
)

S venh cua tiet


H. 9.16
dien ch nhat khi xoan
1
1

+ Tai iem gia canh ngan, ng suat 1


be hn: 1 = max (9.19)

Mz

+Phan bo ng suat tiep tai cac iem tren


cac truc oi xng, cac canh tiet dien va
cac ng cheo c bieu dien H.9.17.
Goc xoan tng oi:
=

Mz
hb3

b)

max
z

max
h

1
b

H. 9.17 Phan bo ng suat tiep


tren tiet dien ch nhat

(9.20)

trong o: , , la cac he so phu thuoc


ty so (canh dai h /canh ngan b) c cho trong bang 1.
Bang 9.1 Gia tr , ,
h
b

1,5

1,75

2,5

10

0,203

0,231

0,239

0,246

0,258

0,267

0,282

0,299

0,307

0,313

0,333

0,141

0,196

0,214

0,229

0,249

0,263

0,281

0,299

0,307

0,313

0,333

1,000

0,859

0,820

0,795

0,766

0,753

0,745

0,743

0,742

0,742

0,742

Chng 9: XOAN THUAN TUY

http://www.ebook.edu.vn

12

GV: Le c Thanh

V. TNH LO XO HNH TRU BC NGAN CHU LC DOC TRUC

Lo xo la mot bo phan c dung rong rai trong ky thuat, c lap at tai


nhng cho can giam chan do tai trong ong nh e mong thang may, he
thong nhun trong oto, e mo t cong suat ln...
Lo xo hnh tru c cau tao bang cach quan mot si day thep tiet dien
vuong, ch nhat hoac tron quanh mot loi hnh tru, ta ch tnh lo xo chu lc
theo phng truc cua hnh tru nay; truc cua hnh tru cung la truc cua lo xo,
ngoai ra ch xet lo xo co cac vong gan nhau goi la lo xo hnh tru bc ngan
(H.9.18.a).
1- Cac ac trng cua lo xo:
+ d: ng knh day lo xo.
+ D: ng knh trung bnh lo xo.
+ n: So vong lam viec cua lo xo.
+ G: Mo un an hoi trt cua vat
lieu lam lo xo.
2- ng suat trong day lo xo:
Dung mot mat cat cha truc cua
loi hnh tru cat qua mot si day lo
xo, tach lo xo lam hai phan, xet
ieu kien can bang cua mot phan
lo xo nh tren H.9.18.b, ta c:

D
P = Qy
h

Mz

a)

b)

H. 9.18. a) Cac ac trng cu a lo xo


b) Noi lc tren tiet die n day lo xo
Qy = P

Y = 0 Q y = P
M / o = 0 M z = P.

lc cat Qy va momen xoan Mz, chung


eu gay ng suat tiep:

= M + Q

Tai mot iem bat ky tren mat


cat ngang, cac thanh phan ng suat
c bieu dien nh (H.9.19). Bo qua
o nghieng cua day lo xo, coi tiet
dien ang xet la tron, co the thay
Chng 9: XOAN THUAN TUY

D
2

Tren mat cat ang xet ( xem


nh mat cat ngang cua day lo xo) co

dF

a)

b)

D/2

o
Mz

M
M

d/2

H. 9.19 Noi lc va ng suat tren


mat cat day lo xo

http://www.ebook.edu.vn

13

GV: Le c Thanh

rang, tai mep trong cua mat cat day lo xo, iem A tren H.9.19, ng suat tiep
at gia tr cc ai, du lc P la tac dung keo hay nen lo xo.
Mot cach gan ung, ng suat tiep tai iem nguy hiem co the tnh nh
sau:
max

max

D
M
P
= Q + M =
+ z =
+ 2
F W p d 2 d 3
4
16
8 PD d
8PD
= 3
+ 1
d 2 D d 3
P

Qy

(9.21)

Thc chat Q khong phan bo eu, con cong thc tnh M nh tren
khong chnh xac v tiet dien khong tron do o nghieng cua day lo xo cung
nh si day lo xo khong la thanh thang, cho nen trong tnh toan thc hanh,
ke en ket qua do thc nghiem, ta co the lay:
max

D
8PD
= k 23 = k
.d
.d 3
16
P

vi

D
+ 0,25
k = d
D
1
d

(9.22)

2- Bien dang cua lo xo: Tnh o co, dan cua lo xo khi chu lc doc
truc.
Dung nguyen ly bao toan nang lng, bo qua cac mat mat nang lng,
cong ngoai lc T hoan toan bien thanh the nang bien dang an hoi U.
Ta co:
+ Cong cua ngoai lc P tren o co, dan cua lo xo la:

T =

1
P
2

(a)

+ The nang bien dang an hoi tch luy trong lo xo (bo qua the nang do Qy):
U =

1
M z2 L
GJ
2
p

U =

1 P 2 D 2 Dn
1 8P 2 D 3n
=
2 4 Gd 4 / 32 2 Gd 4

ve gia tr, T = U,
vi:
trong o:

C =

8PD 3 n P
=
Gd 4
C

Gd 4
8 D3n

(b)
(9.24)
(9.25)

C - la o cng cua lo xo

Chng 9: XOAN THUAN TUY

http://www.ebook.edu.vn

14

GV: Le c Thanh

Th du 9.5 Hai lo xo co o cng C1 = 8 kN/cm va C2 = 5 kN/cm cung chieu


cao H, c ghep ong truc , cung chu lc P = 50 kN (H.9.20.a). Tnh lc
tac dung tren tng lo xo, tnh chuyen v cua iem at lc.
P
P
C1
C2
1

1
R1

a)

H. 9.20

b)

R2

a) Hai lo xo ghep o ng truc


b) No i l c trong lo xo

Giai.
Cat 2 lo xo bang mat cat (1-1), xet can bang phan tren, goi noi lc cua
lo xo la R1 , R2, (H.9.20.b),
Y = 0

R1 + R2 = P

(a)

Mot phng trnh cha hai an so, ta gap bai toan sieu tnh.
ieu kien bien dang: o co ngan cua lo xo 1 phai bang lo xo 2:

1 = 2

C
R1
R
= 1 R1 = 1 R2
C2
C2
C1

(b)
(c)

(c) va (a)
R2 =

R1 =

P
C2
P
=
C1
C
C
+
1
2
1+
C2

(d)

C1
P
C1 + C2

thay gia tr P, C1, C2 vao (d): R1 = 30,77 kN; R2 = 19,23 kN


Chuyen v cua iem at lc chnh la o co cua lo xo 1 hoac lo xo 2
1 = 2 = = R1 / C1 =30,77/8 = 3,85cm.

Chng 9: XOAN THUAN TUY

http://www.ebook.edu.vn

15

GV: Le c Thanh

Th du 9.6 Mot thanh co EJ rat ln c xem la bang , c at tren ba lo


xo co o cng lan lt la C1 = 5 kN/cm, C3 = C2 = 10 kN/cm chu tac dung
cua lc P = 50 kN nh tren H.9.21.a. Tm lc tac dung tren cac lo xo, tnh
goc nghieng cua thanh ABC. Cho a = 50cm.
P

C1

C2

A
C2

C
R3

R2

R1

b)

a)

H. 9.21 a) Thanh ABC tuye t o i c ng a t tre n ba lo xo


b) Ngoai l c va ca c phan lc cua ca c lo xo

Giai.
Goi phan lc cua cac lo xo lan lt la R1, R2, R3 (H.9.21.b).
ieu kien can bang:
Y=0

R1 + R2 + R3 = P

(a)

M/A = 0 -R2.a- R3.3a + P.2a = 0


R2 + 3R3 = 2P

hay:

(b)

ieu kien bien dang: gia s, di tac dung cua ngoai lc, thanh ABC
co v tr mi nh (H.9.22):
2
1

H. 9.22 S o chuyen v cua thanh ABC va bien dang cua cac lo xo

Ta co: 2 =

2
1
3

1
3
3

R2
2 R1 1 R3
=
+
C2
3 C1 3 C3

(c)

Giai he (a), (b), (c), ta c phan lc cua cac lo xo, cung chnh la lc
tac dung len cac lo xo:

R1 =

P
9

; R2 =

1
P;
3

R3 =

5
P
9

T o, ta tnh c bien dang cua cac lo xo:


1 = 1,11cm; 2 = 1,67cm; 3 = 2,78cm
Goc nghieng cua thanh ABC la:
tan = (3 1)/3a = 0,0111 rad

Chng 9: XOAN THUAN TUY

http://www.ebook.edu.vn

16

GV: Le c Thanh

BAI TAP CHNG 9

4M0

9.1 Ve bieu o momen xoan, tnh ng


suat tiep ln nhat va goc xoan tai au t
do cua thanh tiet dien tron co khoan lo
doc truc nh H.9.1. Cho:
Mo = 360 Nm; a=50 cm;
G = 8.106 N/cm2, d = 3 cm.
9.2 Ve bieu o noi lc, kiem
tra o ben va o cng cua truc
tron(H.9.2).Biet: a =40 cm
[]=3000N/cm2; [] = 0,5o/m;
G=8.106 N/cm2; Mo = 1 kNm;
Tnh goc xoan tai B va C.

2M0

M0

d
2

a/2

H. 9.1

4M0

9.3Ve bieu o momen xoan va


tnh ng suat tiep ln nhat tren cac
mat cat ngang nguy hiem cua truc
tron nh tren H.9.3.
Cho: G = hang so.

m
c
8

m
c
0
1

M0

2M0

H. 9.2
3M
A

d
2

9.3 Mot truc chu xoan nh H.9.4.


Xac nh ng suat tiep max cua truc AB,
goc xoan AB , noi lc trong hai thanh CD
va CE.Cho: E=2.107N/cm2, G = 8.106
N/cm2;
M = 2kNm; a =2cm; F=4 m2; d=6 cm.
Xem puli tai C la tuyet oi cng.
9.5 Mot truc truyen ong tiet dien tron,
ng knh d. Tai puli A, truc nhan c
cong suat truyen 15 kW. Gia s hieu
suat truyen la 1, khi o tai cac puli B, C,
H truc truyen i cac cong suat lan lt la
4kW, 8kW va 3kW (H.9.5). Tnh d theo
ieu kien ben va ieu kien cng.

H. 9.3

a
F

4a

0
8
=
D

H. 9.4

H. 9.5

Cho:[]=2kN/cm2;[]=0,40/m;
G=8.103kN/cm2; toc o mot n = 150 vg/ph.

Chng 9: XOAN THUAN TUY

http://www.ebook.edu.vn

17

GV: Le c Thanh

9.6 Tren mat ngoai cua mot truc tron chu xoan
thuan tuy, ngi ta dung tam ien tr va o c
bien dang dai tng oi theo phng 45o so vi
truc la = 30.10-5 (H. 9.6)
Tnh momen xoan tac dung len truc.
Cho: E = 2.104 kN/cm2; = 0,3.

45o

H. 9.6

V tr bu long

9.7 Ngi ta noi hai truc cung ng knh D


= 8 cm bang mat bch va bon bu long 20 bo
tr oi xng tren ng tron ng knh 20cm
(H.9.7). Tnh momen xoan ln nhat co the tac
dung len truc theo ieu kien ben cua truc va
bu long. Cho: []tr = 4 kN/cm2; []bl = 2 kN/cm2.
9.8 Hai truc tron bang thep va ura c
noi vi nhau bang mat bch va bu long chu
mot momen xoan M nh (H.9.8). Tnh
momen xoan noi lc tac dung len hai truc.
Cho: Gth = 2Gura; Dura = 1,5Dth .

H. 9.7

thep

ura

M
Ddu

Dth
2a

H. 9.8

9.9 He chu lc nh H.9.9.a,b


Tnh ng suat trong lo xo 1 va 2.Tnh chuyen v ng tai C, xem thanh ABC
la tuyet oi cng. Biet : D1 =6cm; d1=1cm;n1=10; D2=5cm; d2=0,8cm; n2= 8
P=1kN ; G1=G2 =8.103kN/cm2
P

P
B

2a

a
a)

B
a

2a

b)

H. 9.9

9.10
Mot thanh tuyet oi cng AB c at
tren ba lo xo co cung so vong va chu mot lc
P at au B nh tren H.9.10. Tnh lc tac
dung len cac lo xo. Tnh chuyen v ng tai B.
Cho:
C3 = 2C2 = 2C1 = 2kN/cm; P = kN; a = 1m.

Chng 9: XOAN THUAN TUY

B
C1

C2

C3

H. 9.10

http://www.ebook.edu.vn

18

GV: Le c Thanh
Chng 10

THANH CHU LC PHC TAP


10.1 KHAI NIEM
nh ngha
Thanh chu lc phc tap khi tren cac mat
cat ngang co tac dung ong thi cua nhieu
thanh phan noi lc nh lc doc Nz, momen uon
Mx, My, momen xoan Mz (H.10.1).
Khi mot thanh chu lc phc tap, anh
hng cua lc cat en s chu lc cua thanh
rat nho so vi cac thanh phan noi lc khac nen

Mz

Mx
O

Nz

My

y
H.10.1

trong tnh toan khong xet en lc cat.


2- Cach tnh toan thanh chu lc phc tap
Ap dung Nguyen ly cong tac dung
Nguyen ly cong tac dung: Mot ai lng do nhieu nguyen nhan ong
thi gay ra se bang tong ai lng o do tac ong cua cac nguyen nhan
rieng le ( Chng 1)
10.2 THANH CHU UON XIEN
1- nh ngha Noi lc
Thanh chu uon xien khi tren moi mat cat
ngang ch co hai thanh phan noi lc la momen
uon Mx va momen uon My tac dung trong cac
mat phang yoz va xoz (H.10.2).
Dau cua Mx , My :
Mx > 0 khi cang th y > 0
My > 0 khi cang th x > 0
Theo C hoc ly thuyet, ta co the bieu
dien momen Mx va My bang cac vec t
momen Mx va My (H.10.3); Hp hai momen
nay la momen tong Mu . Mu nam trong mat
phang voz, mat phang nay thang goc vi
truc u (cha vec t momen Mu) va cha
truc thanh (H.10.3).
Chng 10: Thanh chu lc phc tap

Mx
O

My

y
H.10.2
v

u
My
Mu
Mx
x

Mu

y mat phang tai trong

H.10.3 Momen tong


va mat phang tai trong

http://www.ebook.edu.vn

GV: Le c Thanh
Mat phang tai trong la mat phang cha Mu.
Giao tuyen cua mat phang tai trong vi mat cat ngang la ng tai trong
(truc v )
Ky hieu : Goc hp bi truc x va ng tai trong; Ta co
(10.1)
Mu = M x2 + M y2
tan =

Mx
My

(10.2)

nh ngha khac cua uon xien: Thanh chu uon xien khi tren cac mat cat
ngang ch co mot momen uon Mu tac dung trong mat phang cha truc ma
khong trung vi mat phang quan tnh chnh trung tam yOz hay xOz.
ac biet, oi vi thanh tiet dien tron, moi ng knh eu la truc chnh
trung tam ( truc oi xng ), nen bat ky mat phang cha truc thanh nao cung
la mat phang quan tnh chnh trung tam. Do o, mat cat ngang thanh tron
luon luon ch chu uon phang.
2- ng suat phap tren mat cat ngang
Theo nguyen ly cong tac dung, tai mot iem A (x,y) bat ky tren tiet dien,
ng suat do hai momen Mx , My gay ra tnh theo cong thc sau :
My
M
z = x y +
x
(10.3)
Jx
Jy
Trong (10.3), so hang th nhat chnh la ng suat phap do Mx gay ra, so
hang th hai la ng suat phap do My gay ra
Cong thc (10.3) la cong thc ai so, v cac momen uon Mx, My va toa
o iem A(x,y) co dau cua chung
Trong tnh toan thc hanh, thng dung cong

z =

Mx
Jx

Mx

thc ky thuat nh sau:

+ o

My
Jy

(10.4)

x
My

Trong (10.4), lay dau cong (+) hay () tuy theo H.10.4 Bieu dien cac

mien keo, nen tren mat

iem tnh ng suat nam mien chu keo hay nen cat do M , M gay ra
x
y
do tng noi lc gay ra

H.10.4 bieu dien cac mien keo, nen tren mat cat do cac momen uon
Mx , My gay ra : + , - do Mx
+ , _
do My
Chng 10: Thanh chu lc phc tap

http://www.ebook.edu.vn

GV: Le c Thanh
Th du 1. Tiet dien ch nhat bxh= 2040 cm2 chu

uon xien (H.10.5), cho Mx = 8 kNm va My = 5 kNm.


Chieu he truc chon nh h.10.5a

ng suat phap tai B (xB =+10 cm; yB =- 20 cm)

+ Tnh theo (10.3) nh sau:


B =

Mx

My

800
500
(20) +
(10) kN/cm 2
20(40) 3
40(20) 3
12
12

y
H.10.5a)

+ Tnh theo (10.4) nh sau:


Mx gay keo nhng iem nam di Ox va gay nen nhng iem tren Ox;
My gay keo pha trai Oy va gay nen pha phai Oy.
Bieu dien vung keo bang dau (+) va vung nen bang dau () tren tiet
dien (H.10.4a) ta co the thay, tai iem B; Mx gay nen; My gay keo.

B =

800
20 ( 40 )
12

( 20 ) +

500
40 ( 20 ) 3
12

(10 ) kN/cm

3- ng trung hoa va bieu o ng suat


Cong thc (10.3) la mot ham hai bien, no co o th la mot mat phang
trong he truc Oxyz. Neu bieu dien gia tr ng suat phap z cho (10.3) bang
cac oan thang ai so theo truc z nh hng dng ra ngoai mat cat
(H.10.6a), ta c mot mat phang cha au mut cac vect ng suat phap
tai moi iem tren tiet dien, goi la mat ng suat (H.10.6.a).
y

min

_
_
x

min

+
+
z

max

z
y

y
a)

b)

max

Hnh 10.6
a) Mat ng suat; b) Bieu o ng suat phang
Goi giao tuyen cua mat ng suat va mat cat ngang la ng trung
hoa, ta thay, ng trung hoa la mot ng thang va la quy tch cua
nhng iem tren mat cat ngang co tr so ng suat phap bang khong.
Chng 10: Thanh chu lc phc tap

http://www.ebook.edu.vn

GV: Le c Thanh
Cho bieu thc z = 0, ta c phng trnh ng trung hoa:
My
M y Jx
Mx
. .x
y +
x = 0 y =
Jx
Jy
Mx Jy

(10.5)

Phng trnh (10.5) co dang y = ax, ng trung hoa la mot ng


thang qua goc toa o, va co he so goc tnh theo cong thc:
tg =

M y Jx
.
Mx Jy

(10.5)

Ta thay:
- ng trung hoa chia tiet dien lam hai mien: mien chu keo va mien
chu nen.
- Nhng iem nam tren nhng ng thang song song vi ng trung
hoa co cung gia tr ng suat.
- Cang xa ng trung hoa, tr so ng suat cua cac iem tren mot
ng thang vuong goc ng trung hoa tang theo luat bac nhat.
Da tren cac tnh chat nay, co the bieu dien s phan bo bang bieu o
ng suat phang nh sau.
Keo dai ng trung hoa, ve ng chuan vuong goc vi ng trung
hoa tai K, ng suat tai moi iem tren ng trung hoa (z = 0) bieu dien
bang iem K tren ng chuan. S dung phep chieu thang goc, iem nao
co chan hnh chieu xa K nhat la nhng iem chu ng suat phap ln nhat.
- iem xa nhat thuoc mien keo chu ng suat keo ln nhat, goi la max.
- iem xa nhat thuoc mien nen chu ng suat nen ln nhat, goi la min.
Tnh max, min roi bieu dien bang hai oan thang ve hai pha cua ng
chuan roi noi lai bang ng thang, o la bieu o ng suat phang, tr so ng
suat tai moi iem cua tiet dien tren ng thang song song vi ng trung
hoa chnh la mot tung o tren bieu o ng suat xac nh nh (H.10.6.b).
4- ng suat phap cc tr va ieu kien ben
ng suat phap cc tr: Goi A(xA, yA) va B(xB, yB) la hai iem xa
ng trung hoa nhat ve pha chu keo va chu nen, cong thc (10.4) cho:
A = max =
B = min

My
Mx
yA +
xA
Jx
Jy

My
Mx
yB
xB
=
Jx
Jy

Chng 10: Thanh chu lc phc tap

(10.6)

http://www.ebook.edu.vn

GV: Le c Thanh
oi vi thanh co tiet dien ch nhat (b x h), iem xa ng trung hoa
nhat luon luon la cac iem goc cua tiet dien, khi o:
h
;
2

xA = xB =
max =

vi:

Wx =

Mx
Wx

My
Wy

yA = yB =

min =

Mx

Wx

h
2

My

(10.7)

Wy

Jy
Jx
bh2
hb2
=
; Wy =
=
6
6
h/ 2
b/ 2

oi vi thanh co tiet dien tron, khi tiet dien chu tac dung cua hai
momen uon Mx, My trong hai mat phang vuong goc yOz, xOz, momen tong
la Mu tac dung trong mat phang vOz cung la mat phang quan tnh chnh
trung tam , ngha la ch chu uon phang, do o:
max, min =

Mu
Wu

; Mu =

M x2 + M y2 ; Wu =

.D3
0,1 D3
32

(10.8)

ieu kien ben: tren mat cat ngang cua thanh chu uon xien ch co
ng suat phap, khong co ng suat tiep, o la trang thai ng suat n, hai
iem nguy hiem la hai iem chu max, min, tiet dien ben khi hai iem nguy
hiem thoa ieu kien ben:
max []k ;

(10.9)

min []n

oi vi vat lieu deo: [ ]k = [ ]n = [ ], ieu kien ben c thoa khi:


(10.8)

max max , min []

Th du 2. Mot dam tiet dien ch T chu lc nh tren H.10.7.a. Ve bieu o


noi lc, xac nh ng trung hoa tai tiet dien ngam, tnh ng suat max, min.
Cho: q = 4 kN/m; P = qL; L = 2 m; a = 5 cm. Cac ac trng cua tiet dien ch
T c cho nh sau: yo = 7a/4, Jx = 109a4/6 ; Jy = 34a4/6.
Giai. Phan tch lc P thanh 2 thanh phan tren hai truc x va y, ta c:
Px

P.cos300

3 /2

Chng 10: Thanh chu lc phc tap

qL

3 /2;

Py

P.sin300

http://www.ebook.edu.vn

P/2

GV: Le c Thanh
y

30

yo

2a

2a

Py = P/2

4a

y
qL2
2

qL

Mx
3

Mx

b)

My

d)
Px = P 3 / 2

x
3

My

c)

Hnh 10.7 a) S o tai trong


dung len thanh
b) Xet thanh trong mat phang
ve bieu o Mx
c) Xet thanh trong mat phang
ve bieu o My
d) Bieu o noi lc khong

Xet thanh chu lc trong tng mat phang rieng le.


Trong mat phang (yOz), he chu lc phan bo va lc tap trung Py, bieu o
momen ve tren H.10.7.b, theo quy c, bieu o nay la Mx. Tng t, trong
mat phang (xOz), he chu lc phan bo va lc tap trung Py, bieu o momen ve
tren H.10.7.c, o la My.
Phng trnh ng trung hoa:

M y Jx
. .x
Mx Jy

y =

Tai tiet dien ngam: Mx = qL2; My =

3 qL

(a)

/2

Chieu Mx va My bieu dien H.10.5.d, neu chon chieu dng cua truc x
va y nh tren H.10.8.a th trong (a), cac momen uon deu co dau +.
Ta co:

y =

3qL2 / 2 109a 4 / 6
.
x = 2,77.x
qL2
34 a 4 / 6

(b)

Bieu dien tiet dien bang hnh phang theo ty le, t (b) co the ve chnh
xac ng trung hoa, ap dung cach ve bieu o ng suat, ta cung ve c
bieu o ng suat phang (H.10.8.b).

Chng 10: Thanh chu lc phc tap

http://www.ebook.edu.vn

GV: Le c Thanh
y

Mx
My
o

z
B

max

min
b)

a)

Hnh 10.8
a) Chon chieu dng cua truc x, y .
b) ng trung hoa va bieu o ng suat phang
Da tren bieu o ng suat ta co the tm thay iem chu keo nhieu nhat
la iem A(xA = 2a,yA = 7a/4), iem chu nen nhieu nhat la iem
C(xB = 2a,yB = 3a/4); iem B(xB = a/2,yB = 13a/4) co chan hnh
chieu kha gan C, can tnh ng suat tai ay.
Ap dung cong thc (10.4), ta co:
A = max = +

qL2 7a
3qL2 / 2
kN
(. ) +
(2a) = 5,145
Ix
4
Iy
cm 2

C = min = +

qL2 3a
3qL2 / 2
kN
(. )
(2a) = 3,384
Ix
4
Iy
cm 2

Th du 3.

Mot thanh tiet dien tron rong chu tac dung cua ngoai lc

(H.10.9). Tnh ng suat phap max, min, xac nh ng trung hoa tai tiet
dien ngam.
o

60

30o 2P

30o 2P

60o

2a

x
a

Hnh 10.9 Thanh tiet dien tron rong chu tai


trong hai mat phang khac

Giai. Phan tch lc 2P va lc P len hai truc vuong goc x, y. Lan lt xet s
lam viec cua thanh trong tng mat phang yOz, xOz, ta ve c bieu o
momen Mx, My tng ng (H.10.10b).
Chng 10: Thanh chu lc phc tap

http://www.ebook.edu.vn

GV: Le c Thanh
3P

P/2

P
3

a)

2a

2a

Mx

(3 3

b)

z
a

My

(3 3

Pa

Hnh 10.10 Bieu o momen bieu dien trong hai mat phang vuong goc
Vi thanh tiet dien tron, khi co hai momen uon Mx, My tac dung trong hai
mat phang vuong goc yOz, xOz, ta co the a ve mot momen uon phang Mu
trong tac dung mat phang quan tnh chnh trung tam vOz, vi: Mu la momen
tong cua Mx va My.
Tai tiet dien ngam, Mx, My co gia tr ln nhat, ta co:
Mu =

= 9,475 Pa

M x2 + M y2

Theo cong thc cua uon phang, ta c:


max, min =

Mu
Wu

9,745Pa
9,745Pa
kN
=
= 8,41
D 3
.103
d4
84
cm 2
(1 4 )
(1 4 )
32
32
D
10

Phng trnh ng trung hoa:


M y Jx
x
Mx Jy

y =

Tai tiet dien ngam:

(a)

M x = (3 3 + 1) Pa = 6,196 Pa

chieu Mx va My bieu dien H.10.11.a, neu chon chieu dng cua truc x va y
ve pha gay keo cua My va Mx (H.10.11.a) th trong (a), gia tr cua cac
momen uon lay tr tuyet oi.
(b)
Ta co: y = 1.268Pa .(1).x = 0,204 x
6,196 Pa

Mx

x
My

ng trung hoa
B

a)

b)

Hnh 10.11
a) nh hng he truc x,y;

b) Ve ng trung hoa tren hnh phang

ng trung hoa c ve tren hnh phang (H.10.11b), neu ve mot


ng thang qua tam O, thang goc vi ng trung hoa, giao iem cua
ng nay vi chu vi la hai iem chu ng suat keo va nen ln nhat.
Chng 10: Thanh chu lc phc tap

http://www.ebook.edu.vn

GV: Le c Thanh
10.3 THANH CHU UON CONG KEO ( HAY NEN )
1- nh ngha

Thanh chu uon cong keo

(hay

thi khi tren cac mat cat ngang

Nz

noi lc la momen uon Mu va lc


Mu la momen uon tac dung

Mx

My

cha truc z, luon luon co the


momen uon Mx va My trong mat

Hnh 10.11 Cac thanh phan noi


lc tren mat cat ngang

yOz va xOz (H.10.11).

nen)

ong

co cac thanh phan


doc Nz.
trong mat phang
phan

thanh

phang

oi

hai
xng

2- Cong thc ng suat phap


Ap dung nguyen ly cong tac dung, ta thay bai toan ang xet la to hp
cua thanh chu uon xien va keo (hay nen) ung tam. Do o, tai mot iem bat
ky tren mat cat ngang co toa o (x,y) chu tac dung cua ng suat phap tnh
theo cong thc sau:

z =

My
Nz Mx
+
y+
x
A
Ix
Iy

(10.9)

ng suat phap gay keo c quy c dng.


Cac so hang trong cong thc (10.9) la so ai so, ng suat do Nz lay (+)
khi lc doc la keo va ngc lai lc nen lay dau tr; ng suat do Mx, My lay
dau nh trong cong thc (10.1) cua uon xien, neu nh hng truc y,x dng
ve pha gay keo cua Mx, My th lay theo dau cua y va x.
y
x

A
Nz

O
My

+ O

z
My

Mx

Nz

Mx

h
b
b
a)

Hnh 10.12

b)

a) nh hng he truc x,y khi dung cong thc (9.9)


b) nh dau cong tr khi dung cong thc (9.10)

Khi tnh toan thc hanh, ta cung co cong thc ky thuat:


Z =

Nz
A

Mx
Ix

My
Iy

(10.10)

Trong cong thc (10.10), ng vi moi so hang, ta lay dau (+) neu ai
Chng 10: Thanh chu lc phc tap

http://www.ebook.edu.vn

GV: Le c Thanh
lng o gay keo va ngc lai.
V du, oi vi tiet dien tren H.10.12.a, cho Mx = 10 kNm; My = 5 kNm;
Nz = 10 kN; h = 2b = 40 cm, tnh ng suat tai A.
S dung cong thc (10.9), chon chieu dng truc x,y nh H.10.12.a,
xA = 10, yA = 20, ta c:
A =

10
1000
500
+
(20) +
(10)
20.40 20.403 : 12
40.203 : 12

A = 0,0125 0,1875 + 0,1875 = 0,0125 kN/cm2

e ap dung cong thc (10.10), co the bieu dien tac dung gay keo, nen
cua cac thanh phan noi lc nh (H.10.12.b), vi xA =10, yA = 20, ta
c:
10
1000
500
(20) +
(10)

20.40
20.403 : 12
40.203 : 12
= 0,0125 0,1875 + 0,1875 = 0,0125 kN/cm2

A =
A

3- ng trung hoa va bieu o ng suat phap


Tng t nh trong uon xien, co the thay rang phng trnh (10.9) la
mot ham hai bien z = f(x,y), neu bieu dien trong he truc Oxyz, vi O la tam
mat cat ngang va z nh hng dng ra ngoai mat cat, th ham (10.9) bieu
dien mot mat phang, goi la mat ng suat, giao tuyen cua no vi mat cat
ngang la ng trung hoa. De thay rang, ng trung hoa la mot ng
thang cha tat ca nhng iem tren mat cat ngang co ng suat phap bang
khong. T o, cho z = 0, ta co phng trnh ng trung hoa:
y =

M y Ix
N I
x z x
Mx Iy
A Mx

(10.11)

Phng trnh (10.11) co dang y = ax + b, o la mot ng thang khong


qua goc toa o, cat truc y tai tung o

b=

N z .I x
A.M x

e s dung (10.11) thuan li, ta nen nh hng truc x,y nh khi s


dung cong thc (10.9), con Nz van lay dau theo quy c lc doc.
Mat khac, do tnh chat mat phang ng suat, nhng iem nam tren
nhng ng song song ng trung hoa co cung gia tr ng suat, nhng
iem xa ng trung hoa nhat co gia tr ng suat ln nhat, ng suat tren mot
ng vuong goc vi ng trung hoa thay oi theo quy luat bac nhat.
Ro rang ng trung hoa chia tiet dien thanh hai mien, mien chu ng
suat keo va mien chu ng suat nen. Nh cac tnh chat nay, co the bieu dien
Chng 10: Thanh chu lc phc tap
10

http://www.ebook.edu.vn

GV: Le c Thanh
s phan bo cua ng suat phap tren mat cat ngang bang bieu o ng suat
phang nh sau.
Keo dai ng trung hoa ra ngoai tiet dien, ve ng chuan vuong goc
vi ng keo dai tai iem O, o cung la iem bieu dien gia tr ng suat
phap tai moi iem tren ng trung hoa. S dung phep chieu thang goc,
chieu moi iem tren nhng
ng song song

ng trung hoa len ng


chuan, iem co
chan hnh chieu xa O nhat
chu ng suat phap
+
ln nhat.

min

max

iem xa nhat ve mien

Nz z

keo chu ng suat

Mx

M
keo ln nhat, goi la max,
iem xa nhat ve
x
y
mien nen chu ng suat nen
ln nhat, goi la min.
Hnh 10.13 nh hng he truc x,y
Bieu dien gia tr max, min
bang cac tung o
khi dung cong thc 9.11
ve hai pha ng chuan roi
noi chung lai bang
ng thang, ta c bieu o ng suat phang (H.10.13).
y

4. ng suat phap cc tr va ieu kien ben


Goi A(xA,yA) va B(xB,yB) la hai iem xa ng trung hoa nhat ve mien
keo va ve mien nen, ap dung (10.10), ta co cong thc tnh ng suat phap
cc tr.
Nz

A = max =
B = min =

Nz
A

Mx

Ix

Mx
Ix

yA +
yB

My
Iy
My
Iy

xA

(10.12)

xB

Theo (10.12), ta thay, khi ng suat do lc doc trai dau vi ng suat do


Mx, My va co tr so ln hn tong tr so tuyet oi cac ng suat do Mx, My,
ng trung hoa nam ngoai mat cat, tren mat cat ngang ch co ng suat mot
dau (ch chu keo hoac ch chu nen).
- Vi thanh co tiet dien ch nhat, cac iem nguy hiem A, B luon luon la
cac iem goc cua tiet dien:
xA=xB= b/2; yA=yB= h/2
A = max =

Nz

B = min =

Nz

Mx

Mx

Wx
Wx

My
Wy

(10.13)

My
Wy

Chng 10: Thanh chu lc phc tap


11

http://www.ebook.edu.vn

GV: Le c Thanh
- Thanh co tiet dien tron, momen tong cua Mx, My la Mu gay uon thuan
tuy phang, khi o ta co cong thc tnh ng suat phap cc tr:
= max =

Nz

B = min =

Nz

Mu =

A
A

Mu

Mu

(10.13)

Wu
Wu

M x2 + M y2

Thanh chu uon cong keo hay nen ong thi ch gay ra ng suat phap
tren mat cat ngang, tai iem nguy hiem, phan to trang thai ng suat n,
do o ieu kien ben cua thanh la:
(10.14)
max []k ; min []n
5- Thanh chu keo hay nen lech tam
Thanh chu keo hay nen lech tam khi ngoai lc hay noi lc tac dung
tren mat cat ngang tng ng mot lc P song song truc thanh ma khong
trung vi truc thanh. Neu lc P nay hng vao mat cat, thanh chu nen lech
tam, ngc lai, neu lc P hng ra, thanh chu keo lech tam (H.10.14.a).
z

My

P
xK
O

K
yK

xK

Mx

My

K
yK

xK

P
Mx

yK

a)

b)

c)

Hnh 10.14
a) Tiet dien b keo lech tam; b) Di lc ve tam tiet dien

Trong thc te, bai toan nen lech tam rat thng gap trong tnh toan cot,
mong nha cong nghiep hay dan dung, trong tnh toan tru, mong
cau
thap...
Ap dung nguyen ly di lc, a lc keo hay nen lech tam ve tam tiet
dien, ta co the chng minh hai trng hp nay thc chat la bai toan uon
cong keo hay nen ong thi. Tren H.10.14.a, goi K(xK, yK) la iem at lc
lech tam P, di ve tam O, ta co:
Nz = P

, lay (+) khi P la lc keo, ngc lai, lay ().

Mx = P.yK

(10.15)

My = P.xK
Chng 10: Thanh chu lc phc tap
12

http://www.ebook.edu.vn

GV: Le c Thanh
Chieu cua momen lay theo nguyen ly di lc.
Do o, tat ca cong thc a c thiet lap cho bai toan uon cong keo hay
nen ong thi eu ap dung c cho bai toan keo hay nen lech tam.
6- Loi tiet dien
oi vi thanh chu keo hay nen lech tam, phng trnh ng trung hoa
co the viet dang khac. Cho bieu thc z trong (10.9) bang khong, ta c
phng trnh ng trung hoa:
z =
Thay :

My
Nz Mx
y+
+
x = 0
A
Ix
Iy

M x = N z . yK ;

M y = N z . xK

Nz
N z . yK
N x
+
y+ z K x = 0
A
Ix
Iy

Nz
y .F
x .F
[1 + K
x] = 0
y+ K
Iy
Ix
A
at :

at:

Iy
Ix
; iy =
A
A
y . y x .x
1 + K2 + K2 = 0
ix
iy
ix =

a =

i y2
xK

b =

ix2
yK

(10.16)

Ta thu c dang khac cua phng trnh ng trung hoa :


x y
+ =1
a b

(10.17)

T (10.16), (10.17), ta thay ng trung hoa co cac tnh chat sau:


- ng trung hoa cat truc x tai a va truc tung tai b.
- ng trung hoa khong bao gi qua phan t cha iem at lc K v a
va b luon trai dau vi xK, yK.
- iem at lc tien gan tam O cua tiet dien th ng trung hoa ri xa
tam v xK, yK giam th a, b tang.
- Khi ng trung hoa nam ngoai tiet dien, tren tiet dien ch chu ng
suat mot dau: keo hoac nen.
Goi loi tiet dien la khu vc bao quanh tam sao cho khi lc lech tam at
trong pham vi o th ng trung hoa hoan toan nam ngoai tiet dien.
Vi mot thanh chu keo hay nen lech tam, viec xac nh loi tiet dien co y
ngha thc tien. Trong thc te co nhieu loai vat lieu ch chu nen tot nh
gach, a, gang, betong khong thep..., neu chung chu nen lech tam ma lc
nen at ngoai loi tiet dien, ng suat keo phat sinh co the ln hn kha nang
Chng 10: Thanh chu lc phc tap
13

http://www.ebook.edu.vn

GV: Le c Thanh
chu keo cua chung, khi o vat lieu se b pha hoai, e tan dung tot kha nang
chu lc cua vat lieu can thiet ke at lc nen trong loi tiet dien.
Co the xac nh loi tiet dien theo cach sau:
Gia s ng trung hoa tiep xuc mot canh tiet dien, t (10.17) ta viet
c phng trnh ng trung hoa, roi t (10.16) ta suy ra toa o iem at
lc K tng ng vi v tr ng trung hoa. Ap dung cach tng t oi vi tat
ca cac canh con lai, noi v tr cac iem at lc, ta c loi tiet dien. e y
rang, du tiet dien la a giac lom th loi tiet dien luon la mot a giac loi.
V du: tiet dien ch nhat (H.10.15).
Khi ng trung hoa trung canh AB:
x
y
+
=1
h/2
i2y

= xK = 0
xK

ix2

yK

ng trung

B
x

C
ng trung hoa

h
h
h
yK =
=
h
2
6
12.
2

Hnh

Khi ng trung hoa trung canh BC:

Loi tiet dien ch nhat

x
y
+
=1
b/ 2
iy2
b
b2
b

=
xK =
=
xK
2
12.b / 2
6

ix2
= yK = 0
yK

Do tnh oi xng cua tiet dien, khi ng trung hoa trung canh CD, AD,
ta xac nh hai iem K tng ng co toa o lan lt la:
xK = 0; yK =

h
va
6

xK = b ; yK = 0
6

Noi cac iem K, ta c loi tiet dien cua tiet dien ch nhat la mot hnh
thoi co nh tren truc x,y (H.10.15).
- Tiet dien tron (H.10.16)
Khi ng trung hoa la mot tiep tuyen vi ng tron tai A:
x
y
+
=1
D/2

iy2
xK

= xK = 0 ;

yK =

ix2
= D/2
yK

.D4
D
=
8
.D2 D
64.
4 2

ng trung hoa

Do tnh oi xng cua tiet

dien, ta thay loi


D/8

Chng 10: Thanh chu lc phc tap

14
Hnh

Lohttp://www.ebook.edu.vn
i tiet dien

GV: Le c Thanh
tiet dien la mot ng tron ong tam ng knh D/8.
V du 10.3 Mot thanh tiet dien ch nhat (b.h), chu tac dung cua ngoai lc
nh H.10.17.a. Ve bieu o noi lc, tnh max, min. xac nh ng trung hoa
tai ngam.
Cho: q = 5 kN/m, P1 = 100 kN, P2 = 6 kN, H = 6 m, h = 2b = 40 cm.
z

P1

P1

P2

P1

P2

b
h

qH /2

Nz

a)

P2H

My

Mx

b)

Hnh 10.17

a) Thanh chu nen cong uon; b) Bieu o noi lc

Giai:
Bieu o noi lc do tng nguyen nhan gay ra c ve tren H.10.17.b.
Tai ngam, noi lc co gia tr ln nhat:
Nz = P1 (nen); Mx = qH2/2; My = P2.H
Ap dung cong thc (10.12):
max, min =

P1 q.H 2 / 2 P2 .H

A
Wx
Wy

Thay so, ta c:
100
5.62.100 6.6 .100

20.40
20.402
40.202
2.
6
6
2,912kN/cm 2
= 0.125 1,687 1,350 =
3,162 kN/cm 2

max, min =

Phng trnh ng trung hoa:


y =

M y Ix
N
I
.x z . x
Mx Iy
A Mx

(a)

Chon he truc y,x dng ve pha gay keo cua Mx va My, thay so vao (a) ta
c:

Chng 10: Thanh chu lc phc tap


15

http://www.ebook.edu.vn

GV: Le c Thanh
y =

100 20.403 / 12
6.6 20.403 / 12
= 1,6 x + 1,48
.x
.
2
3
20.40
6 40.20 / 12
62
5.
5.
.100
2
2

ng trung hoa va bieu o ng suat c ve tren H.10.18.


max

Mx

P1
y

My

ng trung hoa

min

Hnh 10.18 ng trung hoa cua thanh chu nen uon

V du 10.4 Mot cot chu nen lech tam va lc ay cua gio nh H.10.19.a.
xem chan cot b ngam. Tnh max, min. Neu khoi mong co kch thc
1m3m0,5m c at nh H.10.19.a, hay tnh ap lc ln nhat tren nen at.
P1 = 50 kN; q = 4 kN/m; H = 6 m; h = 2b = 40 cm; = 25

Cho:
kN/m3.

e = 20 cm

P1

Mx

Nz
0,5 m
1m

q
h

b)

3m
141,3 cm

8,7 cm

0,5 m
min

1m
3m
c)

a)

26 cm

Hnh 10.19 a) Cot chu nen lech tam


b) Noi lc tai tiet dien chan cot; c) Bieu o ap lc len nen at

Noi lc ln nhat tai tiet dien ngam:


Nz = P1 = 50 kN (nen)
Mx = P1.e + qH2/2 = 50.20 + 4.62.100/2 = 8200 kN.cm
Ap dung cong thc (10.12), ng suat phap ln nhat:
max, min =

P1 q.H 2 / 2 + P1 .e

=
A
Wx

max, min =

1,47 kN
50
8200

= 0,0625 1,537 =
1,60 cm 2
20.40 20.402 / 6

Chng 10: Thanh chu lc phc tap


16

http://www.ebook.edu.vn

GV: Le c Thanh
Di lc ve ay mong, ke them trong lng ban than mong va momen do
lc cat qH, ta c:
Nz = 50 25.0,5.2.1 = 75 kN (nen)
Mx = 10600 kNcm.
Tai ay mong, neu vat lieu van lien tuc, ta co phng trnh ng trung
hoa:
y =

N z Ix
75
100.3003
=
.
= 53,07 cm
100.300 12.10600
A Mx

Theo (10.12), ta co ng suat phap ln nhat:


max, min =

0,0045 kN
75
10600

=
0,0095 cm 2
100.300 100.3002 / 6

Thc te, tai ay mong, vat lieu la at ch chu nen, khong the chu ng
suat keo, do o, e am bao ieu kien can bang, hp lc cua phan lc nen
phai can bang vi ngoai lc tac dung.
Ngoai lc tai mat ay mong gom mot lc nen 75 kN va mot momen
Mx = 10600 kNcm tng ng mot lc nen 75 kN lech tam at tren truc y
vi o lech tam la e = 10600/75 =141,3 cm, at cach mep chu nen ln nhat
la 150 141,3 = 8,7 cm.
e can bang vi lc nay, hp lc cua phan lc nen phai oi ang vi
lc nen 75 kN, gia s phan lc nen phan bo theo quy luat bac nhat, phan
lc nen phai phan bo tren mot dien tch mat mong 100 (3 8,7) = 100 26
cm2 tnh t mep chu nen ln nhat (H10.19.c).
ieu kien can bang cho:

min.100.26/2 = 75 => min = 0,0577 kN/cm2 = 5,77 kG/cm2


Ket qua nay cho thay, do mat e mong khong c thiet ke s dung
toan bo dien tch mat mong nen ng suat nen truyen len nen tang len, mong
thiet ke khong hp ly.
10.4 UON CONG XOAN
1- nh ngha
Thanh chu uon cong xoan khi tren cac mat cat ngang co tac dung ong
thi cua momen uon Mu trong mat phang cha truc thanh va momen xoan
Mz.
Chng 10: Thanh chu lc phc tap
17

http://www.ebook.edu.vn

GV: Le c Thanh
2- Thanh tiet dien ch nhat
Uon xoan thanh tiet dien ch nhat thng gap trong cong trnh dan
dung nh lanh to o vang, dam chu lc ngoai mat phang oi xng, thanh
chu uon trong he khong gian...
Xet mot tiet dien ch nhat chu uon xoan (H.10.20) trong o momen
uon Mu a c phan tch thanh hai momen uon Mx, My trong cac mat phang
quan tnh chnh trung tam yOz, xOz.
min(Mx,M y)

y
D

min(Mx)

Mz

max(M y)

My

C
A

max(M x,M y)

min(My)

min(My)

max(M x)

max(M x)
max(M x,M y)

max

max(M y)

Mx

min(Mx,M y)

min(Mx)

max

b)

a)

Hnh 10.20 a) Cac thanh phan noi lc cua thanh chu uon cong xoan
b) Trang thai ng suat cua cac phan to

Ap dung nguyen ly cong tac dung va ly thuyet ve uon, ve xoan, ta c


cac ket qua nh sau (H.10.20.b):
Tai cac goc tiet dien (A,B), ch co ng suat phap ln nhat do Mx,My,
phan to trang thai ng suat n:
max, min =

ieu kien ben:

Mx
Wx

My

(10.19)

Wy

max []k ;

min []n

Tai iem gia canh ngan (C,D), chu ng suat phap ln nhat do Mx va
ng suat tiep 1 do Mz, phan to trang thai ng suat phang:
max, min =

Mx
Wx

1 = max

(10.20)

ieu kien ben:


Theo thuyet ben th 3:

2 + 42 []

Theo thuyet ben th 4:

2 + 3 2 []

Tai iem gia canh dai (E,F), chu ng suat phap ln nhat do My va ng
suat tiep 1max do Mz, phan to trang thai ng suat phang:
Chng 10: Thanh chu lc phc tap
18

http://www.ebook.edu.vn

GV: Le c Thanh
My

max, min =

Wy

max =

Mz
.h.b2

(10.21)

ieu kien ben:


Theo thuyet ben th 3:

2 + 4 2 []

Theo thuyet ben th 4:

2 + 3 2 []

3- Tiet dien tron


Thanh tiet dien tron chu uon xoan ong thi rat thng gap khi tnh truc
truyen ong v qua trnh truyen tac dung xoan qua cac puli luon kem theo
tac dung uon do lc cang day ai, do trong lng ban than truc, puli...
Xet mot thanh tiet dien tron chu tac dung cua momen uon Mu va
momen xoan Mz (H.10.21.a). Neu co nhieu ngoai lc gay uon tac dung
trong nhng mat phang khac nhau, ta luon luon co the phan tch chung
thanh cac thanh phan tac dung trong hai mat phang vuong goc yOz, xOz, t
o xac nh Mx, My, sau o xac nh momen tong Mu =
v

min(Mu)

Mz

B
z

max(Mz)
max(Mu)

max(Mu)

Mu
A

a)

max(Mz)

min(Mu)

M x2 + M y2

b)

Hnh 10.21 a) Thanh tiet dien tron chu uon xoan


b) Trang thai ng suat phan to

Ap dung nguyen ly cong tac dung va ly thuyet ve uon, ve xoan, ta c


cac ket qua nh sau (H.10.21.b):
Di tac dung cua momen uon Mu, hai iem A,B chu ng suat phap ln
nhat max, min, ngoai ra, do tac dung cua momen xoan Mz, tai hai iem A, B
con chu ng suat tiep max, o la hai iem nguy hiem nhat tren tiet dien.

Ta co:

max, min =

max =

Mu
Wu

; Mu =

M x2 + M 2y

(10.22)

Mz
Wp

Chng 10: Thanh chu lc phc tap


19

http://www.ebook.edu.vn

GV: Le c Thanh
Phan to ang xet va chu ng suat phap va chu ng suat tiep, o la
phan to trang thai ng suat phang.
ieu kien ben:
Theo thuyet ben th 3:
2 + 4 2 [ ]

Theo thuyet ben th 4:


2 + 32 []
P
q

P
q

a/2

a)

C
a/2

a
A

1728 kN.cm

648 kN.cm

3qa2

9qa2/8

Muon

30cm
9qa2/8

20cm

c)

Mxoan

b)

Hnh 10.22 a) Khung chu uon vi tai trong thang goc mat phang
khung
b) S o tnh khung va bieu o noi lc khong gian ve theo
nguyen ly cong tac dung
c) Cac iem nguy hiem tren tiet dien
V du 10.5 Mot thanh gay khuc ABC tiet dien ch nhat (20cm 30cm) chu
tac dung cua tai trong nh H.10.22.a. Ve bieu o noi lc, kiem tra ieu kien
ben tai tiet dien ngam. Cho: q = 4 kN/m; P = 2qa; a = 1,2 m; [] = 1 kN/cm2.
Giai. Bieu o noi lc c ve tren H.10.22.b, tai tiet dien ngam chu noi lc
ln nhat (H.10.22.c):
Mx = 3qa2
2

= 3.4.(1,2)2.100

= 1728 kN.cm

Mz = 9qa /8 = 9.4.(1,2) .100/8 = 648 kN.cm


Tai trung iem canh ngan, phan to trang thai ng suat phang:
2

Mx
1728
=
= 0,576 kN/cm 2
Wx
20.302 / 6
Mz
648
1 = . max = .
= 0,859.
= 0,2 kN/cm 2
2
.h.b
0,231.30.202

max =

Chng 10: Thanh chu lc phc tap


20

http://www.ebook.edu.vn

GV: Le c Thanh
ieu kien ben:
0,5762 + 4.0,22 = 0,7 kN/cm 2 < []

2 + 42 =

= 1 kN/cm 2

Tai trung iem canh dai, phan to trang thai trt thuan tuy:
max =

Mz
648
=
= 0,233 kN/cm 2
.hb2
0,231.30.202

ieu kien ben:

max = 0,233 kN / cm 2 < [] / 2 = 0,5 kN / cm 2

V du 10.6 Mot truc tron ng knh d, mang pu li chu ong ng knh D1


va pu li b ong ng knh D2. Mo t truyen lc keo T1 len mot nhanh day
ai cua pu li D1 lam quay truc, keo theo pu li D2. Coi hieu suat truyen la 1,
lc keo tren mot nhanh day ai D2 la T2 = T1.D1/D2. Ngoai ra, gia s lc cang
ban au tren day ai bang na lc keo tac dung len day ai. Tnh ng
knh truc d (H.10.23.a).
Cho: trong lng pu li G1 = G2 = 1 kN; D1 = 50 cm; D2 = 30 cm;
T1 = 5 kN; [] = 12 kN/cm2. Bo qua trong lng ban than cua truc.
Giai. Lc cang ban au tren day ai cua pu li D1 la: T1/2 = 5/2 = 2,5 kN
Lc keo truyen len day ai D2 la: T2 = T1.D1/D2 = 5.50/30 = 8,33 kN
Lc cang ban au tren day ai D2 la: T2/2 = 8,33/2 = 4,17 kN
Di lc tren day ai ve tam cua truc, ta co the a ra s o tnh cua truc
nh tren H.10.23.b. Bieu o momen uon Mx, My va momen xoan Mz ve
H.10.23.c.
Tai tiet dien at pu li D2 chu noi lc ln nhat:
Mx = 20 kN.cm, My = 150 kN.cm; Mz = 125 kN.cm.
Momen uon tong Mu =

M x2 + M 2y

= 151,32 kN.cm gay ra ng suat phap

ln nhat la:
Mu
151,32
1542,1
=
=
3
Wu
D3
.D / 32

z =

Momen xoan Mz = 125 kNcm gay ra ng suat tiep ln nhat la:


max =

Mz
125
636,9
=
=
3
Wp
D3
.D / 16

ieu kien ben theo thuyet ben th ba:


Ta co:

2 + 4. 2 []

1542,12
636,9 2
2000
+
4
.
[]
[] D 5,5 cm
( D3 )2
( D3 )2
D3

Co the chon ng knh truc la 55 mm.

Chng 10: Thanh chu lc phc tap


21

http://www.ebook.edu.vn

GV: Le c Thanh
20cm
T2.D2 /2

40cm

20cm
40cm
T1/2

20cm

D1

T2/2

D2

20cm

T2

2T2

G1

T2/2

T1/2

G2

T1.D1/2

b)

T1

MZ

2T1

Mx

125kN.cm

a)

20

c)

Hnh 10.23
a) Truc tiet dien tron chu uon cong xoan
b) S o tnh truc
c) Bieu o noi lc

.c
kN

My
150 kN.cm

116,6 kN.cm

10.5 THANH CHU LC TONG QUAT


1. nh ngha
Thanh chu lc tong quat khi tren cac mat cat ngang co tac dung cua
lc doc Nz, momen uon Mu va momen xoan Mz.
Thanh chu lc tong quat thng gap khi tnh cac thanh chu lc theo s
o khong gian.
1- Thanh co tiet dien ch nhat
Ap dung nguyen ly cong tac dung va ly thuyet ve keo (nen), ve uon, va
ve xoan, ta c cac ket qua nh sau (H.10.24.a,b):
B

y
D

min (Mx,My,Nz)

Mz

F
Nz

min (Mx,Nz)

max

Mx

My

max (My,Nz)

max (My,Nz)
1

max
min (My,Nz)

min (My,Nz)

max (Mx,Nz)
C

max (Mx,My,Nz)

a)

min (Mx,My,Nz)

min (Mx,Nz)

max (Mx,Nz)

C
A

max (Mx,My,Nz)
b)

Hnh 10.24 a) Cac thanh phan noi lc tren mat cat ngang
b) Trang thai ng suat cua cac phan to

Tai cac goc tiet dien, ch co ng suat phap do Nz, Mx, My, phan to
trang thai ng suat n:
max, min =

Nz
A

Mx
Wx

My
Wy

(10.23)

Chng 10: Thanh chu lc phc tap


22

http://www.ebook.edu.vn

GV: Le c Thanh
ieu kien ben:

max []k ;

min []n

Tai iem gia canh dai, phan to va chu ng suat phap ln nhat do My
va lc doc Nz, va chu ng suat tiep ln nhat do Mz, o la phan to trang
thai ng suat phang:
max, min =

Nz
A

My
Wy

max =

Mz
hb2

(10.24)

ieu kien ben:


Theo thuyet ben th 3:

2 + 42 []

Theo thuyet ben th 4:

2 + 3 2 []

(10.25)

Tai iem gia canh ngan, phan to va chu ng suat phap ln nhat do
Mx va lc doc Nz, va chu ng suat tiep do Mz, phan to trang thai ng suat
phang:
max, min =

Nz
A

Mx
Wx

1 = max

(10.26)

ieu kien ben:


Theo thuyet ben th 3:

2 + 4 2 []

Theo thuyet ben th 4:

2 + 3 2 []

2- Thanh co tiet dien tron (H.10.25.a,b)


iem nguy hiem nam tren chu vi, o la hai iem A,B. hai iem nay va
chu ng suat phap ln nhat do momen Mu va lc doc Nz, va chu ng suat
tiep ln nhat do Mz, phan to trang thai ng suat phang.
max, min =
max =

Nz
A

Mu
Wu

Mu =

Mz
Wp

M x2 + M 2y

(10.27)
(10.28)

ieu kien ben:


Theo thuyet ben th 3:

2 + 4 2 []

Theo thuyet ben th 4:

2 + 3 2 []

Chng 10: Thanh chu lc phc tap


23

http://www.ebook.edu.vn

GV: Le c Thanh
v

max(Mz)

min(Mu, Nz)

min(Mu, Nz)

Mz
z

max(Mz)
max(Mu, Nz)

Mu

max(Mu, Nz)

b)

a)

Hnh 10.25
a) Cac thanh phan noi lc
b) Trang thai ng suat cua cac phan to

V du 10.7 Co mot thanh tiet dien tron ng knh D chu mot he lc khong
gian nh tren H.10.26.a. Ve bieu o noi lc. xac nh ng knh D.
Cho: q = 4 kN/m; P = qa; a = 4 m; [] = 16 kN/cm2.
Giai. Bieu o noi lc c ve H.10.26.b.
Tai ngam tiet dien chu noi lc ln nhat:
Nz = qa = 4.4= 16 kN (nen); Mx = qa2 = 4.42.100 = 6400 kN.cm
My = qa2/2 = 4.42.100/2 = 3200 kN.cm; Mz = qa2/8 = 4.42.100/8 = 800
kN.cm
P = qa
q = 4 kN/m
a/2

qa2/8

qa2/2

qa2/8

qa

a
qa2/2
qa2
a)

Muon do P

Muon do q

b)

Nz

Mz

Hnh 10.26 a) S o tnh thanh chu lc phc tap


b) Bieu o noi lc ve theo nguyen ly cong tac dung

ng suat phap ln nhat:


max =

Nz
A

M x2

Mu =
max =

Mu
Wu

+ M y2 =

64002 + 32002 = 7155,41 kN.cm

16
7155,4
+
.D 2 / 4 .D3 / 32

ng suat tiep ln nhat:


max =

Mz
800
=
Wp
.D 3 / 16

Chng 10: Thanh chu lc phc tap


24

http://www.ebook.edu.vn

GV: Le c Thanh
ieu kien ben:
Theo thuyet ben th 3:

2 + 42 []

800
16
7155,4 2
+
) 2 []
) + 4.(
.D 2 / 4 .D3 / 32
.D 3 / 16

Trong tnh toan thc hanh, e thuan li cho viec giai bat phng trnh
tren, ban au chon D theo uon xoan, bo qua ng suat do lc doc, sau o
kiem tra lai, ta co:

7155,4 2
800
) + 4.(
) 2 [] D 16,6 cm
.D3 / 32
.D3 / 16

Ban au, chon: D = 168 mm.


Kiem tra ieu kien ben:
Theo thuyet ben th 3:
(

16

2 + 42 []

7155,4

.16,8 2 / 4 .16,83 / 32

) 2 + 4.(

800

.16,83 / 16

)2

(0,072 + 15,38) 2 + 4.(0,86) 2 = 15,54 kN/cm 2 < [ ] = 16 kN/cm 2

Vay chon: D = 168 mm.


BAI TAP CHNG 10
10.1 Mot thanh cong xon tiet dien ch nhat chu tac dung cua tai trong nh
H.10.27. Ve bieu o noi lc, tnh ng suat phap ln nhat, xac nh v tr
ng trung hoa tai mat cat ngam.
P = qL

P = qL

E = 103 kN/cm2

30
20 cm

q = 4 kN/m

12 cm

L=2m

Hnh 10.27

10.2 Xac nh gia tr tuyet oi ln nhat cua ng suat phap, v tr ng trung


hoa tai mat cat nguy hiem cua dam (H.10.28), a = 1 m.
P = 4 kN

cm

20 cm

12

2a

Hnh 10.28

Chng 10: Thanh chu lc phc tap


25

http://www.ebook.edu.vn

GV: Le c Thanh
10.3 Xac nh max , min va v tr ng trung hoa tai mat cat nguy hiem cua
cot H.10.29.
P = 80 kN

40cm

x
q = 2 kN/m

m
4

20 cm

Hnh 10.29

10.4 Mot cot chu tai trong nh H.10.30. Xac nh ng suat nen ln nhat va
nho nhat tai mat cat chan cot. Cho trong lng rieng cua vat lieu cot la:
= 20 kN/m3.
1m

P = 1000 kN

0,8 m

3m

m
6

Hnh 10.30

10.5 a. Mot tru co tiet dien gom hai thep hnh so hieu [ 24 chu tai trong
nh H.10.31.
Xac nh ng suat keo va nen ln nhat tai mat cat chan cot co xet ca
trong lng cua cot.
b. Mot cot chu tai trong nh H.10.32. Tnh ng suat ng suat keo va nen
ln nhat.

Chng 10: Thanh chu lc phc tap


26

http://www.ebook.edu.vn

GV: Le c Thanh
4m

P1 = 20 kN

2m
P2 = 5 kN

q = 3 kN/m
P2 = 0,5 kN
m
6

P1 = 2 kN

e = 60 cm
m
6

40 cm

m
4

Hnh 10.31

m
c
0
2

Hnh 10.32

10.6 Mot cot tron rong chu tac dung cua tai trong nh H.10.33.a.
Tnh ng suat phap max, min tai tiet dien chan cot, xac nh v tr va bieu
dien ng trung hoa tai tiet dien nay.
Gia s mong cot co kch thc 2 m 1,2 m h, trong lng rieng
= 25 kN/m3 (H.10.33.b) va truc cot c bo tr i qua tam mong. Hay
ch cach bo tr mat bang mong va tnh kch thc h sao cho ay mong
khong phat sinh ng suat keo.
z
P1 = 100 kN
x
P2 =10 kN
y

H=4m

P2 = 5 kN

1,2 m
2d = 40 cm

2m
h
b)

a)

Hnh 10.33

10.7 Mot khung tiet dien ch nhat eu, co thanh cang AB, chu tac dung
cua tai trong nh H.10.34. Ve bieu o noi lc cua khung va noi lc keo
trong thanh AB. xac nh ng suat max, min va v tr ng trung hoa tai
mat cat ngang K.

Chng 10: Thanh chu lc phc tap


27

http://www.ebook.edu.vn

GV: Le c Thanh
q = 6 kN/m
q = 4kN/m

2m
K

P = 4 kN

h = 2b
m
3
=
H

4m

b=
20 cm
A

I24

B
L=2m
L/2 = 4 m

L/2 = 4 m

Hnh 10.35

Hnh 10.34

10.8 Mot khung tiet dien ch I24, chu tac dung cua tai trong nh H.10.35.
xac nh noi lc tai tiet dien chan cot. Kiem tra ben.
Cho []=16 kN/cm2.
10.9 Mot thanh gay khuc tiet dien tron ng knh d chu lc nh H.10.36.
Ve bieu o noi lc, xac nh ng knh d theo thuyet ben ng suat tiep
ln nhat. Cho [] = 2,8 kN/cm2.
30 cm

40 cm

P1 = 0,8 kN

10 cm

P2=0,5 kN

Hnh 10.36

10.10 Mot truc truyen ong tiet dien tron ng knh d co s o tnh nh
H.10.37. Ve bieu o noi lc, xac nh ng knh d theo thuyet ben ng
suat tiep ln nhat. Cho [] = 10 kN/cm2.

Chng 10: Thanh chu lc phc tap


28

http://www.ebook.edu.vn

GV: Le c Thanh
M2 = 2 kNm

M3 = 4 kNm M1 =10 kNm M4 = 4 kNm


P

P=1 kN

Hnh 10.37

Chng 10: Thanh chu lc phc tap


29

http://www.ebook.edu.vn

GV: Le c Thanh
Chng 11

ON NH CUA THANH THANG CHU NEN UNG TAM


11.1 KHAI NIEM VE S ON NH CUA TRANG THAI CAN BANG
e ap ng yeu cau chu lc bnh thng, mot thanh phai thoa man
ieu kien ben va cng, nh a c trnh bay trong cac chng trc ay.
Tuy nhien, trong nhieu trng hp, thanh con phai thoa man them ieu
kien on nh. o la kha nang duy tr hnh thc bien dang ban au neu b
nhieu. Trong thc te, nhieu co the la cac yeu to sai lech so vi s o tnh
nh o cong ban au, s nghieng hoac lech tam cua lc tac dung...
Khai niem on nh co the minh hoa bang cach xet s can bang cua
qua cau tren cac mat lom, loi va phang tren H.11.1.

H.11.1 S can bang ve v tr cua qua cau


Neu cho qua cau mot chuyen dch nho (goi la nhieu) t v tr ban au
sang v tr lan can roi bo nhieu i th:
- Tren mat lom, qua cau quay ve v tr ban au: s can bang v tr
ban au la on nh.
- Tren mat loi, qua cau chuyen ong ra xa hn v tr ban au: s can
bang v tr ban au la khong on nh.
- Tren mat phang, qua cau gi nguyen v tr mi: s can bang v tr
ban au la phiem nh.
Hien tng tng t cung co the xay ra oi vi s can bang ve trang
thai bien dang cua he an hoi. Chang han vi thanh chu nen tren H.11.2.
Trong ieu kien ly tng (thanh thang tuyet oi, lc P hoan toan ung
tam...) th thanh se gi hnh dang thang, ch co ngan do chu nen ung
tam. Neu cho iem at cua lc P mot chuyen v be do mot lc ngang nao
o gay ra, sau o bo lc nay i th se xay ra cac trng hp bien dang nh
sau:

Chng 11: On nh thanh thang chu nen ung tahttp://www.ebook.edu.vn


m
1

GV: Le c Thanh
+ Neu lc P nho hn mot gia tr Pth nao o, goi la lc ti han, tc la
P < Pth, th thanh se phuc hoi lai trang thai bien dang thang. Ta noi thanh
lam viec trang thai on nh.
+ Neu P > Pth th chuyen
v se tang va thanh b cong
them. S can bang cua trang

P< Pth

P= Pth

P> Pth

thai thang ( = 0) la khong on


nh. Ta noi thanh trang
c)
b)
thai mat on nh .Trong thc a)
TT ma t O n nh
TT ti ha n
te thanh se co chuyen v va TT On nh
chuyen sang hnh thc bien
H. 11.2 S can bang cua TT bie n dang
dang mi b uon cong, khac
trc ve tnh chat, bat li ve ieu kien chu lc.

+ ng vi P = Pth th thanh van gi nguyen chuyen v va trang thai


bien dang cong. S can bang cua trang thai thang la phiem nh. Ta noi
thanh trang thai ti han
H.11.3 gii thieu them vai ket cau co the b mat on nh nh dam
chu uon, vanh tron chu nen eu
Khi xay ra mat on nh du ch
cua mot thanh cung dan ti s sup o
cua toan bo ket cau. Tnh chat pha
hoai do mat on nh la ot ngot va
P > Pth

nguy hiem. Trong lch s nganh xay


dng a tng xay ra nhng tham hoa
sap cau ch v s mat on nh cua mot

q > qth

H. 11.3 Cac dang mat on nh

thanh dan chu nen nh cau Mekhelstein Thuy S (1891), cau Lavrentia
My (1907)... V vay khi thiet ke can phai am bao ca ieu kien on nh,
ngoai ieu kien ben va ieu kien cng a neu trc ay.
ieu kien on nh: P [P ]o =

Pth
ko

Hay : N z [P ]o =

Pth
ko

(11.1)
(11.2)

ko : He so an toan ve mat on nh, do quy nh, va thng ln hn he


so an toan ve o ben n.
P ( hay Nz ) : Lc nen ( noi lc nen ) thanh.
Chng 11: On nh thanh thang chu nen ung tahttp://www.ebook.edu.vn
m
2

GV: Le c Thanh
11.2 KHAO SAT ON NH TRONG MIEN AN HOI
1- Tnh lc ti han Pth thanh co ket khp hai au ( Bai toan Euler)
Xet thanh thang lien ket khp hai au,

Pth

chu nen bi lc ti han Pth. Khi b nhieu,


thanh se b uon cong

Pth

va can bang hnh

dang mi nh tren H.11.4a.


at he truc toa o (x,y,z) nh H.11.4a

y(z)

Xet mat cat co hoanh o z ;

Pth

o vong mat cat nay la y.


Ta co phng trnh vi phan ng an hoi:
y '' =

M
EJ

(b) vao (a) y '' =


at: 2 =

Pth

EJ

(a)

Vi: momen uon M = Pth y


Pth y
EJ

b)

H. 11.4

(b)

(t ieu kien can bang tren H.11.4b)

hay

y '' +

Pth
y=0
EJ

(c)

y '' + 2 y = 0

Nghiem tong quat cua (c) la:


y = A sin( z ) + B cos( z )

(d)

Cac hang so c xac nh t ieu kien bien y(0) = 0 va y(L) = 0.


Vi: y(0) = 0
B=0
A sin( L) = 0
y(L) = 0
e bai toan co ngha

y( z) 0

A 0 , sin( L) = 0

phng trnh nay co nghiem L = n , vi n = 1, 2, 3,...

Pth =

n 2 2 EJ
L2

(e)

Thc te, khi lc nen at en gia tr ti han nho nhat theo (e) ng vi n = 1
th thanh a b cong. V vay, cac gia tr ng vi n > 1 khong co y ngha.
Ngoai ra, thanh se cong trong mat phang co o cng uon nho nhat.
Do o, cong thc tnh lc ti han cua thanh thang hai au lien ket khp la:
Pth =

2 EJ min
L2

(11.3)

ng an hoi tng ng co dang mot na song hnh sine:


y = A sin(

z
L

(11.4)

vi: A la mot hang so be, the hien o vong gia nhp.


Chng 11: On nh thanh thang chu nen ung tahttp://www.ebook.edu.vn
m
3

GV: Le c Thanh
2- Tnh Pth thanh co cac lien ket khac au thanh
Ap dung phng phap tren cho thanh co cac lien ket khac nhau hai
au, ta c cong thc tnh lc ti han co dang chung:
Pth =

m 2 2 EJ min
L2

(11.5)

vi: m - la so na song hnh sine cua ng an hoi khi mat on nh.


at =

1
, goi la he so quy oi, (11.5) thanh
m
2 EJ min
Pth =
2
( L)

(11.6)

(11.6) c goi chung la cong thc Euler


Dang mat on nh va he so cua thanh co lien ket hai au khac nhau
the hien tren H.11.5.

m=1/2
= 2

m= 1
= 1

m= 1,43
= 0,7

m= 2
= 1/2

m= 1
= 1

m=1/2
= 2

H. 11.5 Dang mat on nh va he so


3- ng suat ti han
ng suat trong thanh thang chu nen ung tam bi lc Pth goi la ng
suat ti han va c xac nh theo cong thc:
th =

2
Pth
2 EJ min
2 Eimin
2E
=
=
=
2
2
2
F
( L) F
( L) L

imin

(11.7)

J min
la ban knh quan tnh nho nhat cua tiet dien .
F
L
: o manh cua thanh
(11.8)
at =
imin

vi: imin =

(11.7) thanh:

th =

2 E
2

(11.9)

o manh khong co th nguyen, phu thuoc vao chieu dai thanh, ieu
kien lien ket va ac trng hnh hoc cua tiet dien; thanh co o manh cang
ln th cang de mat on nh.
Chng 11: On nh thanh thang chu nen ung tahttp://www.ebook.edu.vn
m
4

GV: Le c Thanh
4- Gii han ap dung cong thc Euler
Cong thc Euler c xay dng tren c s phng trnh vi phan ng
an hoi, v vay ch ap dung c khi vat lieu con lam viec trong giai oan
an hoi, tc la ng suat trong thanh nho hn gii han ty le:
th =

hay:

Neu at:

2 E
tl
2
2 E
tl

o =

(f)
2 E
tl

(11.10)

th ieu kien ap dung cua cong thc Euler la:


o

(11.11)

trong o: o - c goi la o manh gii han va la mot hang so oi vi moi


loai vat lieu.
Th du: Thep xay dng thong thng o = 100, go o = 75; gang o = 80.
Neu o th goi la o manh ln.
Nh vay, cong thc Euler ch ap dung c cho thanh co o manh ln.

Chng 11: On nh thanh thang chu nen ung tahttp://www.ebook.edu.vn


m
5

GV: Le c Thanh
11.3 ON NH NGOAI MIEN AN HOI
1- Y ngha
Cong thc Euler ch ap dung c khi vat
lieu an hoi. o th cua phng trnh (11.6) la
mot hyperbola nh tren H.11.6, ch ung khi
th tl .
Khi th f tl vat lieu lam viec ngoai mien

h
0
l

Iasinski
Hyperbola Euler

H. 11.6 ng suat ti han

an hoi, can thiet phai co cong thc khac e tnh Pth.


2- Cong thc thc nghiem Iasinski
Cong thc Iasinski c e xuat da tren nhieu so lieu thc nghiem,
phu thuoc vao o manh cua thanh.
- Thanh co o manh va 1 p o :
(11.12)

th = a b

vi: a va b la cac hang so phu thuoc vat lieu, c xac nh bang thc
nghiem: Thep xay dng: a = 33,6 kN/cm2; b = 0,147 kN/cm2
Go:
a = 2,93 kN/cm2; b = 0,0194 kN/cm2
o manh 1 c xac nh t cong thc:
1 =

a tl
b

thc nghiem cho thay pham vi gia tr

(11.13)
1 = 30 40

- Thanh co o manh be p 1 : Khi nay thanh khong mat on nh ma


at en trang thai pha hoai cua vat lieu. V vay, ta coi:
th = 0 = b oi vi vat lieu don
th = 0 = ch oi vi vat lieu deo

(11.14)

va Lc ti han cua thanh : Pth = th . F

(11.15)

Chng 11: On nh thanh thang chu nen ung tahttp://www.ebook.edu.vn


m
6

GV: Le c Thanh
Th du 11.1 Tnh Pth va th cua mot cot lam bang thep so 3 co mat cat
ngang hnh ch so 22. Cot co lien ket khp hai au. Xet hai trng hp:
a. Chieu cao cua cot 3,0 m
b. Chieu cao cua cot 2,25 m
Biet: E = 2,1.104 kN/cm2;tl = 21 kN/cm2 ; o = 100
Cac hang so trong cong thc Iasinski : a= 33,6 kN/cm2, b=0,147 kN/cm2
Giai.
Tra bang thep nh hnh (phu luc ) ta co cac so lieu cua thep No22:
imin = i y = 2,27 cm; F = 30,6 cm 2 ; theo lien ket cua thanh th ta co = 1 .
+ Trng hp a)
o manh :

l
imin

1.300
= 132 > o = 100
2,27

Thanh co o manh ln, ap dung cong thc Euler


th =

2 E 2 2,1.104
=
= 11,88 kN / cm 2
2
2

132

Pth = th F = 11,88.30,6 = 363,62 kN .

+ Trng hp b)
o manh :

l
imin

1 =

1.225
= 99,11 < 0
2,27
a tl
33,6 21
=
= 85,7
b
0,147

1 < < 0

Thanh co o manh va, dung cong thc Iasinski:


th = a b = 33,6 0,147.90 = 20,37 kN / cm 2

Pth = th F = 20,37.30,6 = 623,32 kN .

Chu y: - Neu lien ket cua thanh trong hai mat phang quan tnh giong nhau
trong cac cong thc a co se dung Jmin va imin.
- Neu lien ket cua thanh trong hai mat phang quan tnh khac nhau
th khi mat on nh thanh se cong trong mat phang co o manh ln va cac
ai lng J , i se lay trong mat phang nay.

Chng 11: On nh thanh thang chu nen ung tahttp://www.ebook.edu.vn


m
7

GV: Le c Thanh
11.4 PHNG PHAP THC HANH TNH ON NH THANH CHU NEN
1- Phng phap tnh: Thanh chu nen can phai thoa :
ieu kien ben: =

[ ]n ; vi: []n = o
n
Fth

(11.16)

trong o: n - he so an toan ve o ben


Fth - dien tch tiet dien giam yeu (b khoet lo); neu khong khoet lo
th Fth = F la tiet dien nguyen
ieu kien on nh: =

[ ]o ; vi: [ ]o = th
ko
F

(11.17)

trong o: ko ( hay k)- he so an toan ve on nh.


V s giam yeu cuc bo tai mot so tiet dien co anh hng khong ang
ke en s on nh chung cua thanh.
Do tnh chat nguy hiem
cua hien tng mat on nh va
xet en nhng yeu to khong
tranh c nh o cong ban
au, o lech tam cua lc nen
nen chon ko > n, va k thay oi
phu thuoc vao o manh. Thep
xay dng co ko = 1,8 3,5 nh
minh hoa tren H.11.7; gang
ko = 5 5,5; go ko = 2,8 3,2.

,kG/cm2

e thuan tien cho tnh toan

2400
2000
Euler Hyperbola

2400

1400
1000

k = 3,5

k =1,7

ng gii han ng suat


0

50

100

150

200

250

Hnh.11.7 He so an toan ko cho thep

thc hanh, ngi ta a vao


khai niem he so uon doc hoac he so giam ng suat cho phep c
nh ngha nh sau:
=

[]o
n
= th
[]n
o k

< 1, v ca hai t so:

th
<1
o

va

n
<1
k

t o: [ ]o = [ ] , va ieu kien on nh tr thanh: =


hay:
hay:

P
[ ]n
F

(11.18)

P
[ ]n ;
F
P [P ]o = [ ]n F

(11.19)

ieu kien on nh (11.18) thoa, ieu kien ben (11.16) khong can kiem tra
Chng 11: On nh thanh thang chu nen ung tahttp://www.ebook.edu.vn
m
8

GV: Le c Thanh
He so =

[E, , k ]

c cho bang 11.1

Bang 11.1 He so

o
manh

Tr so oi vi
Thep
so
2,3,4

Thep

Thep

so 5

CK

Gang

Go

1,00

1,00

1,00

1,00

1,00

10

0,99

0,98

0,97

0,97

0,99

20

0,96

0,95

0,95

0,91

0,97

30

0,94

0,92

0,91

0,81

0,93

40

0,92

0,89

0,87

0,69

0,87

50

0,89

0,86

0,83

0,54

0,80

60

0,86

0,82

0,79

0,44

0,71

70

0,81

0,76

0,72

0,34

0,60

80

0,75

0,70

0,65

0,26

0,48

90

0,69

0,62

0,55

0,20

0,38

100

0,60

0,51

0,43

0,16

0,31

110

0,52

0,43

0,35

0,25

120

0,45

0,36

0,30

0,22

130

0,40

0,33

0,26

0,18

140

0,36

0,29

0,23

0,16

150

0,32

0,26

0,21

0,14

160

0,29

0,24

0,19

0,12

170

0,26

0,21

0,171

0,11

180

0,23

0,19

0,15

0,10

190

0,21

0,17

0,14

0,09

200

0,19

0,16

0,13

0,08

Chng 11: On nh thanh thang chu nen ung tahttp://www.ebook.edu.vn


m
9

GV: Le c Thanh
V < 1 nen thng ch can kiem tra ieu kien on nh la u. Tuy
nhien, neu thanh co giam yeu cuc bo do lien ket bu long, inh tan th can
kiem tra ca hai ieu kien ben va on nh.
- ieu kien ben:

- ieu kien on nh

P
[ ]n
Fth

(11.20)

P
[ ]n
F

(11.21)

trong thc te, neu thoa (11.21) th thng cung thoa (11.20).
oi vi bai toan on nh cung co ba bai toan:
1. Kiem tra on nh:
=

P
[ ]n
F

(11.22)

2. Xac nh tai trong cho phep:


(11.23)

[ P ] F [ ]n

Trong hai bai toan tren, v tiet dien thanh a biet nen co the suy ra he
so theo trnh t: F, I =

J /F

(tra bang 11.1)

3. Chon tiet dien:


F

P
[ ]n

(11.24)

viec tm F phai lam ung dan, v trong (11.22) cha hai bien: F va (F).
Trnh t nh sau:
- Gia thiet: o = 0,5; tnh c: Fo =
- T

F1 =

tra bang ta c

o' .

Neu

P
o
o [ ]n

o' o

th lay:

1 =

o + 'o
2

P
1 1'
1[ ]n

thng lap lai qua trnh tnh khoang 2 - 3 lan th sai so tng oi gia hai
lan tnh u nho ( 5%).

Chng 11: On nh thanh thang chu nen ung tahttp://www.ebook.edu.vn


m
10

GV: Le c Thanh
Th du 11.3 Chon so lieu thep cho thanh dai 2,0m, lien ket khp hai
au va chu lc nen P = 230 kN. Biet vat lieu la thep so 2 co [ ]n = 14 kN / cm 2 .
Giai:
a. Lan chon th nhat
Gia thiet

= 0,5 ,

230
P
=
= 32,8cm 2
[ ]n 14,0.0,5

Tra bang thep nh hnh ta chon thep ch so 24 co F = 34,8 cm2,


iy = imin = 2,37 cm, ta co o manh:
=

imin

1.200
= 84,4
2,37

Tra bang quan he gia va ta c = 0,724 . He so nay khac vi


gia thiet ban au nen ta phai chon lai.
b. Lan chon th hai
Gia thiet:

0,5 + 0,724
230
= 26,84cm 2
= 0,612 F
0,612.14
2

Tra bang thep nh hnh ta tm c thep ch so 20 vi F= 26,8 cm2,


imin = 2,07 cm. o manh luc o bang:
=

1.200
= 96,6
2,07

tra bang ta tm c = 0,631 gan ung gia tr 0,625 theo gia thiet. Do o, ta
kiem tra lai ieu kien on nh:
P
[ ]n ;
F

230
= 13,6 kN / cm 2 < [ ] = 14 kN / cm 2
0,631.26,8

Vay ta chon thep ch so 20.

Chng 11: On nh thanh thang chu nen ung tahttp://www.ebook.edu.vn


m
11

GV: Le c Thanh
2- Chon mat cat ngang va vat lieu hp ly
Khi thiet ke thanh chu nen, ngi ta co gang lam cho kha nang chu
lc cua thanh cang ln cang tot. Theo cong thc (11.6) va (11.15) ta co lc
ti han:
- Trong mien an hoi:

Pth =

2 EI

(11.6)

(l) 2

- Ngoai mien an hoi: Pth = th .F


Thng th chieu dai va lien ket
hai au thanh c cho trc. V vay,

(11.15)
th, MN/m2

Thep hp kim
e tang Pth co hai cach:
300
Thep t cacbon
1) Chon vat lieu co moun an 240
hoi ln, V du dung thep thay cho be 200
tong. Tuy nhien, ch dung thep cng
o cao thay cho thep cng o thap 100
khi thanh lam viec ngoai mien an
hoi; con trong mien an hoi thep co
0
40
80 100 120
160

moun an hoi giong nhau nen viec


thay the khong co li ve mat chu lc
nh o th tren H.11.8 the hien.
2) Neu he so lien ket giong nhau theo hai phng th cau tao tiet

dien co

Ix = Iy ,

va thng lam tiet dien rong e tang momen quan tnh cua

mat cat nhng phai co cau tao e khong mat on nh cuc bo. Tiet dien hp
ly cua cot chu nen trong thc te thng co dang nh tren H.11.9

Hnh 11.9 Dang tiet dien hp ly

Neu lien ket hai phng khac nhau th nen cau tao tiet dien sao cho co
max = min

hay:

Jx

2
x

Jy

y2

(11.25)

Chng 11: On nh thanh thang chu nen ung tahttp://www.ebook.edu.vn


m
12

GV: Le c Thanh
11.5 XAC NH LC TI HAN BANG PHNG PHAP NANG LNG
1- Khai niem
Viec tm lc ti han cua thanh co o manh ln theo phng phap tnh
do Euler thc hien la chnh xac. Tuy nhien, trong thc te co nhng bai toan
phc tap hn nh thanh co o cng EJ thay oi, lc phan bo doc theo truc
thanh... th viec thiet lap va giai phng trnh vi phan e tm lc ti han tr
nen phc tap.
Trong trng hp o, ngi ta co the da tren nguyen ly bao toan
nang lng e tm nghiem gan ung.
2- Phng phap nang lng xac nh lc ti han
Gia s thanh chu nen ung tam bi lc Pth, nh c minh hoa tren
H.11.10.
z

dz

dz

Pth

de

Hnh 11.10 Xac nh lc ti han


Di tac ong cua nhieu, thanh b uon cong vi phng trnh y(z),
iem at cua lc Pth dch chuyen mot oan e. Theo nguyen ly bao toan
nang lng, cong A cua lc Pth bang the nang bien dang uon U cua thanh:
A=U
(11.26)
A = Pth e
(11.27)
trong o:
l

U =

1
M2
'' 2
o 2 EJ dz = 2 o EJy dz

(11.28)

e xac nh o co ngan e cua thanh do s uon cong gay ra, ta xet


phan to thanh dz tren H.11.11. Ta co:
de = dz dz cos = dz(1 cos ) = dz(2 sin 2

hay:

de =

y'2
dz
2

(11.29)

Chu y rang, v goc xoay


sin

= ;
2
2

) = dz2 =
dz
2
2
2

la be nen tren ta a coi:

= tg = y'

Tch phan (11.30) ta c:


e=

l
1
y' 2
=
y' 2 dz
dz
2
2
o
o
l

(11.30)

Chng 11: On nh thanh thang chu nen ung tahttp://www.ebook.edu.vn


m
13

GV: Le c Thanh
l

Do o:

A=

Pth
y '2 dz
2 o

(11.31)

The (11.31) va (11.28) vao (11.26) ta co:


Pth
2

y'

dz =

1
2
EIy" dz
2 o
l

hay:

Pth =

EIy

"2

dz

(11.32)

y'

dz

Khi tm lc Pth theo phng phap nang lng, ta chon y(z) thoa ieu
kien bien va the vao (11.33). V thng y(z) la gan ung nen lc Pth cung
gan ung. S sai lech cua ng an hoi y(z) co y ngha nh la thanh c
at them mot he lien ket an hoi nao o phan bo doc theo truc thanh va
lam cho thanh tr nen cng hn. V vay, lc Pth tm theo phng phap nang
lng luon ln hn gia tr that (ch bang gia tr that khi ng an hoi c
chon chnh xac).
Th du 11.4 Tm lc Pth cho thanh tren H.11.11

Pth

vi EJ = hang so
Giai.
Gia s ng an hoi c chon gan ung theo
dang do lc phan bo eu gay ra nh sau:
y = z( z3 2lz2 + l 3 )

vi - la mot hang so be.


ta co:
y' = (4 z3 6lz2 + l 3 )
y' ' = 12 ( z2 lz)

the vao (11.33) ta tm c:


So vi nghiem chnh xac

Pth =

Pth =

Hnh 11.11
Tm Pth bang
phap naphng
ng ln
g

9,882 EI
l2

2 EI
9,8696EI
=
l2
l2

th ket qua tnh ln hn 0,25%.

Neu ng an hoi chon la mot na song hnh sine, tc la trung vi


ng an hoi chnh xac cua bai toan Euler, th Pth tm theo phng phap
nang lng cung cho ket qua chnh xac.

Chng 11: On nh thanh thang chu nen ung tahttp://www.ebook.edu.vn


m
14

GV: Le c Thanh
BAI TAP CHNG 11
11.1 Cho bon thanh co mat cat ngang nh nhau lam bang cung mot loai
vat lieu va co lien ket nh tren H.11.1.
Neu muon chu c cung mot lc nen ung tam th chieu dai cua moi
thanh phai bang bao nhieu La. Gia thiet vat lieu mat on nh trong mien an

lc

lb

la

a)

ld

hoi va EJ = hang so.

c)

b)

d)

Hnh 11.1
11.2 Thanh co chieu dai L = 3 m, mot au ngam, mot au khp. Hay xac
nh lc ti han cua thanh trong ba trng hp sau ay:
a. Mat cat hnh tron ban knh R = 4 cm, vat lieu la gang xam co:

tl = 17,8 kN/cm2; E = 1,15.104 kN/cm2.


b. Mat cat hnh tron rong ban knh ngoai R = 3 cm va ban knh trong
r = 2 cm, vat lieu la ura co tl = 18 kN/cm2; E = 0,71.104 kN/cm2.
c. Mat cat hnh vuong canh 15 cm 15 cm, vat lieu bang go co:

tl = 1,7 kN/cm2; E = 0,1.104 kN/cm2. Biet hai he so trong cong thc


Iasinski la a = 2,93 kN/cm2 va b = 0,0194 kN/cm2
11.3 Cho thanh bang gang co l = 1,6 m;

a = 6 cm; t = 1 cm nh H.11.14. Xac nh


lc ti han va ng suat ti han. Cho o = 80;
a = 77,6 kN/cm ; b = 1,2 kN/cm . Muon
2

thanh mat on nh khi vat lieu con lam viec


trong gii han an hoi th chieu dai cua
thanh phai bao nhieu?

l
t
a

Hnh 11.3

Chng 11: On nh thanh thang chu nen ung tahttp://www.ebook.edu.vn


m
15

GV: Le c Thanh
11.4

Kiem tra on nh cua cac

P = 200kN

thanh cho tren H.11.4, neu []


= 14 kN/cm2. Lc nen cho phep
2L 160 x100 x9
4L 80 x 6

3m

cua thanh thep la thep so 3.

5m

ln nhat la bao nhieu? Vat lieu

P = 200kN

20

a)

b)
Hnh 11.4

11.5 Cho hai he thanh chu lc nh tren H.11.5. Xac nh so hieu mat cat
ch I cua thanh chong AB, biet [ ] = 16 kN/cm2. Vat lieu la thep so 3.
Xac nh he so an toan ve on nh cua cac thanh o.
q = 40 kN/m

P = 200 kN

P = 950 kN
A

A
2m

3m

2m

C
2m

B
a)
o

60
2m

b)

4m

Hnh 11.5

11.6 Mot gia chu tai trong phan bo eu nh tren H.11.6. Xac nh tr so
cho phep cua cng o tai trong phan bo tac dung len gia. Thanh AB co
mat cat hnh vuong canh 5 cm x 5 cm lam bang go co [] = 1 kN/cm2.
a

10 m

A
q

B
y
1

P = 100 kN
8m
1

x
5 cm

Hnh 11.7

Hnh 11.6

Chng 11: On nh thanh thang chu nen ung tahttp://www.ebook.edu.vn


m
16

GV: Le c Thanh
11.7 Mot dam cau truc AD chu lc nh H.11.7. Cot BC lam bang hai thep
ch I so 14 ghep lai sao cho mo men quan tnh oi vi hai truc bang nhau.
Xac nh chieu dai toi a cua mut tha a, biet rang cot lam viec bat li nhat
khi xe cau truc mang mot trong lng 100 kN at au mut tha. Tai trong
phan bo q = 4 kN/m.
L 80 x 80 x 6

2m

L 100 x 100 x 10

1 cm

l
B

1 cm
6m

Hnh 11.8

Hnh 11.9

11.8 He thanh chu lc nh H.11.8. Xac nh chieu dai l cua thanh chong
AB lam bang thep co [] = 14 kN/cm2. Cho biet tai trong P = 300 kN.
11.9 Mot thanh chu nen ung tam c lam bang bon thep goc eu canh
loai 80 80 6 (H.11.9). Xac nh kch thc a cua mat cat. Biet thanh
dai l = 6 m hai au lien ket khp va chu lc nen au cot P =200 kN.
Vat lieu co [] = 20 kN/cm2.
11.10 Mot cot go dai L= 3 m, mat cat hnh ch nhat b h. au di cua cot
c chon vao nen be tong, au tren co the trt theo mot khe nho
song song vi phng chieu dai h cua mat cat (H.11.10). Xac nh kch
thc cua mat cat b h sao cho mat cat la hp ly nhat. Cho biet lc
nen P = 100 N, [] = 1 kN/cm2.

3m

Chng 11: On nh thanh thang chu nen ung tahttp://www.ebook.edu.vn


m
17

GV: Le c Thanh
Chng 12

UON NGANG VA UON DOC ONG THI


12.1 AC IEM BAI TOAN
Xet mot thanh chu uon bi tac ong ong thi cua lc ngang R va
lc nen doc P nh tren H.12.1. Neu chuyen v la ang ke th can phai xet
can bang cua thanh tren s o bien dang va momen noi lc se bao gom
anh hng cua lc R va P:
(12.1)
M(z) = MR + MP = MR + Py(z)
MR - momen uon do rieng tai trong ngang gay ra

trong o:

Py(z) - momen uon do lc doc gay ra.


R

z
y(z)

Hnh 12.1 Uon ngang va uon doc ong thi


Bai toan nh vay c goi la uon ngang va uon doc ong thi.
ac iem cua bai toan:
- Momen M(z) phu thuoc vao o vong y(z)
- Momen M(z) phu thuoc phi tuyen vao lc P v o vong y(z) cung phu
thuoc vao P. V vay, nguyen ly cong tac dung khong ap dung c cho loai
bai toan nay.
12.2 PHNG PHAP CHNH XAC
e tm c momen uon, trc het can thiet lap phng trnh vi phan
ng an hoi cua dam chu lc nen P va tai trong ngang.
Q

q(z)

q(z)
P
P

y(z)

M + dM
M

dz

O
Q + dQ

Hnh 12.2 Thanh chu uon nen


Chng 12: Uon ngang va uon doc ong thi

http://www.ebook.edu.vn

GV: Le c Thanh
Xet can bang tren s o bien dang cua phan to thanh dz nh tren
H.12.2

Mo = 0 :
chu y rang :

tg =

M + dM M Qdz Pdz tg = 0

dy
dz

dM
dy
P
= Q
dz
dz

ta co:

(12.2)

lay ao ham hai ve cua (12.2), chu y rang

dQ
= q(z) ,
dz

d2 M
d2 y

= q ( z)
P
dz2
dz2

the

M = EIy" (*)

ta co phng trnh:

(12.3)

vao (12.3) ta thu c:


(12.4)

EIy IV + Py" = q ( z)

ay la phng trnh vi phan ng an hoi cua dam chu nen uon.


Neu biet tai trong tac dung va cac ieu kien bien th co the giai (12.4) e
tm ng an hoi, t o suy ra momen uon theo phng trnh (*). Trong
thc te, thng co nhieu quy luat tai trong khac nhau tren chieu dai thanh
nen viec giai phng trnh (12.4) rat phc tap. V vay, ngi ta thng ap
dung phng phap gan ung di ay.
12.3 PHNG PHAP GAN UNG
Xet dam n gian chu tai trong oi xng nh H.12.3.
q

q
P

f0
l

l
a)

Hnh 12.3 ng an hoi oi xng

b)

S o (a) ch chu tai trong ngang, vi o vong gia nhp fo.


S o (b) chu ong thi tai trong ngang va tai trong doc, co o vong
gia nhp f.
Gia thiet ng an hoi co dang hnh sine (giong dang mat on nh), ta
co phng trnh ng an hoi trong hai trng hp nh sau:
z
yo = fo sin
; y = f sin z
l

Dang phng trnh nay thoa ieu kien bien y = y " = 0 tai hai khp.
Momen uon noi lc tng ng nh sau:
M o = EIyo" = EI

2
z
2
f sin
= EI 2 yo
2 o
l
l
l

Chng 12: Uon ngang va uon doc ong thi

http://www.ebook.edu.vn

GV: Le c Thanh
M = EIy" = EI

2
z
2
f sin
= EI 2 y
2
l
l
l

The cac ket qua nay vao phng trnh (12.1) ta co:
2
2
y
=
EI
yo + Py
l2
l2
yo ( z)
ra:
y( z) =
2 EI
1 P/ 2
l
yo ( z)
y( z) =
P
1
Pth

(12.5)

EI

t o suy
hay:
vi:

Pth =

2 EI
l2

(12.6)

la lc ti han cua thanh khi mat on nh trong mat

phang

uon.
ao ham hai ve cua (12.6) va nhan vi EI ta co:
EIy" ( z) =

hay:

EIy0" ( z)
P
1
Pth

M ( z) =

Mo
P
1
Pth

(12.7)

Chu y: - Neu tai khong oi xng nhng cung hng ve mot pha th cac
cong thc tren kem chnh xac hn nhng van dung c.
- Neu thanh co lien ket hai au khac th van dung c cac cong thc
(12.6), (12.7) nhng can xet ti he so lien ket
Pth =

2 EI
(l) 2

trong cong thc Pth:


(12.8)

12.4 NG SUAT VA KIEM TRA BEN


ng suat ln nhat c tnh theo cong thc:
max =

Mo
P M
P
+
=
+
A W
A W (1 P )
Pth

(12.9)

V ng suat phu thuoc phi tuyen vao tai trong nen kiem tra ben theo
ng suat cho phep khong am bao an toan theo he so n d kien. Trong
trng hp nay, ngi ta dung ieu kien an toan theo tai trong nh sau:
nM o
nP
+
o
A W (1 nP )
Pth

(12.10)

V du 12.1 Tm momen uon va o vong ln nhat cua dam thep ch INo36


chu lc nh tren H.12.4.

Chng 12: Uon ngang va uon doc ong thi

http://www.ebook.edu.vn

GV: Le c Thanh
q = 2 kN/m
S = 120 kN

x
y
4m

Hnh 12.4

Giai. S dung bang tra thep nh hnh, tng ng vi so hieu INo36 va cac
ky hieu tren hnh tren, ta co:
A = 61,9 cm2; Ix = 516 cm4; Iy = 13380 cm4; E = 2,1.104 kN/cm2
Tr so ln nhat cua momen uon, o vong do tai trong ngang gay ra tai
gia nhp:

Mo =

yo =

ql 2
2.4 2
=
= 4 kNm
8
8

5 ql 4
5 2.102.4004
.
=
.
= 0,615 cm
384 EI x
384 2,1.104.516

Tr so lc ti han:
Pth =

2 EI x

(l)

2 .2,1.104.516

(1.400)2

= 668 kN

o vong cua dam, theo cong thc gan ung:


yo
0,615
y =
=
= 0,75cm , tang 22% so vi
S
120
1

Pth

yo

668

Momen uon ln nhat, theo cong thc gan ung th nhat:


M = M o + Sy = 4 + 120.0,075 = 4,9 kNm

Momen uon ln nhat, theo cong thc gan ung th hai:


Mo
4
M =
=
= 4,87 kNm sai so 0,5% so vi cong thc gan ung th
S
120
1

Pth

668

nhat.
Gia tr momen trong trng hp uon ngang va doc tang 22,5% so vi
momen ch do lc ngang gay ra, tc la thien ve an toan hn.
12.5 THANH CO O CONG BAN AU
1- Anh hng cua o cong ban au
Xet thanh co o cong ban au, chu lc nen P nh tren H.12.5. Gia s
ng cong ban au co dang:
z
yo = a sin
(12.11)
l

Chng 12: Uon ngang va uon doc ong thi

http://www.ebook.edu.vn

GV: Le c Thanh
z

P
yo

y1

l/2

l/2

Hnh 12.5 Thanh co o cong ban au


Do tac dung cua lc P, thanh b vong them co phng trnh y1(z). o
vong toan phan: y = yo + y1
(12.12)
Momen uon do lc P gay ra:
(12.13)

M = Py = P ( yo + y1 )

Phng trnh vi phan o vong them:


(12.14)

EIy1'' = M = P ( yo + y1 )

the (12.11) vao (12.14) va at:


y1'' + 2 y1 = 2 a sin

2 =

z
l

P
EI

ta co:
(12.15)

Nghiem cua phng trnh nay co dang:


y1 = A sin z + B cos z +

Cac ieu kien bien:


Do o:

hay:
vi:

y1 =

1
2l2
y1 (0) = 0

y1 (l) = 0

z
1
=
a sin
l
2
1
2l2

a sin

z
l

(12.16)

B=0
A=0

z
1
a sin
l
2
1
P 2
l
EI

z
k
a sin
1 k
l
P
P
k =
= 2
Pth
EI
l2

(12.17)

y1 =

o vong toan phan: y


hay:

y =

(12.18)

= yo + y1 = (a +

yo
P
1
Pth

a
k
z
z
=
sin
a) sin
1 k
l
1k
l

(12.19)

Momen ln nhat gia nhp:


M max = Pymax =

Pa
P
1
Pth

(12.20)

Neu ng cong ban au co dang bat ky th co the phan tch thanh


chuoi Fourier nh sau:

yo = a1 sin

2z
z
+ a2 sin
+ ...
l
l

(12.21)

the (12.13) vao (12.21) va giai ra y1 ta co:


Chng 12: Uon ngang va uon doc ong thi

http://www.ebook.edu.vn

GV: Le c Thanh
z
2z
a
a

+ 2 2 sin
+ ...
y1 = k 1 sin
l
l
2 k
1 k

v:

k=

P
<1
Pth

(12.22)

nen khi P u ln th so hang au troi han va ch can xet so

hang nay.
2- Xac nh lc ti han bang thc nghiem thanh lien ket khp hai
au
Xet thanh chu nen nh tren H.12.6, trong thc te thanh luon co o
cong ban au.
P

a1
tan = Pth

a1

Hnh 12.6

Hnh 12.7

Thanh co o cong ban au chu nen

Cach xac nh lc ti han

Khi lc P u ln th du thanh b cong ban au the nao, ta van co quan


he gia va a1 theo (12.17):
a1
k
a1 =
P
1 k
th
1
P

= Pth ( ) a1
P
=

hay:

ay la phng trnh bac nhat cua hai bien va / P nen co o th la


mot ng thang nh tren H.12.7.
Khi th nghiem, ng vi moi gia tr lc nen Pi , ta o c chuyen v i
va tnh c

i / Pi ,

t o lap bang ket qua th nghiem co dang:

P1

P2

Pn

/P

1 / P1

2 / P2

n / Pn

T o xac nh cac iem tren he truc

va ve c o th nh tren

H.12.7. Ta thng dung phng phap bnh phng cc tieu e xac nh


va o vong ban au ln nhat

a1 .

Chng 12: Uon ngang va uon doc ong thi

http://www.ebook.edu.vn

Pth

GV: Le c Thanh
12.6 COT CHU NEN LECH TAM
Xet cot manh chu nen lech tam bi lc P nh tren H.12.8.
z
yo = a sin
(12.11)
l

Do tac dung cua lc P, cot b cong va co phng trnh y(z).


Momen uon tai mot tiet dien do lc P gay ra:
(12.23)

M = P{e + y( z)} = Pe + Py( z)

trong o:
e - la o lech tam ban au; y - la o vong cua truc cot.
Phng trnh vi phan ng an hoi nh sau:
z

y'' ( z) =

M
EI

(12.24)

The (12.23) vao (12.24) va at

2 =

P
EI

ta

c:
(12.25)

y" + 2 y = 2e

Nghiem tong quat cua phng trnh nay la

tong cua nghiem thuan nhat va nghiem rieng:

y(z)

y = A sin z + B cos z e
P

trong o:

Hnh 12.8 Cot co o cong ban au

(12.26)

A va B - la cac hang so cua nghiem

thuan nhat; e - la nghiem rieng.

Cac ieu kien bien:


y (0) = 0

B = e

y (l) = 0

A =

e(1 cos l)
l
= e tan
sin l
2

Phng trnh ng an hoi tr thanh:


y = e(tan

l
sin z + cos z 1)
2

o vong ln nhat tai gia nhp, tc


= ymax = e (

1
cos

l
2

(12.27)
z=

1)

l
2

la:
(12.29)

(12.28)
Neu e = 0 hoac P = 0 th = 0 .

Chng 12: Uon ngang va uon doc ong thi

http://www.ebook.edu.vn

GV: Le c Thanh
o th quan he gia P - c cho trong H.12.9. o th nay ch co y
ngha khi vat lieu con an hoi, tc la con nho va P < Pth.
P
e=0

P th

e = e1
e = e2
e 2 > e1

Hnh 12.9 o th quan he gia P -

Momen uon ln nhat tai gia nhp c tnh:


M max = P (e + ymax ) = Pe

Quan he
ln th

M max

M max -

1
cos

(12.30)

P l
EI 2

P cho bi H.12.10. Khi P nho th

M max Pe ,

nhng khi P

tang rat nhanh.

T cac o th nay ta thay quan he P -

va

M max -

P phi tuyen.

Trong thc te, tnh cot manh chu nen lech tam can thiet phai xet ac
iem phi tuyen nay e am bao an toan.
Mmax

Pe
P th

Hnh 12.10 Quan he gia

Mmax - P

ng suat cc ai trong thanh:


max

P M max c
P
ec
1

=
+
=
1+ 2
A
I
A
r
P l
cos

EI 2

vi: A - dien tch tiet dien thanh;

(12.31)

r - ban knh quan tnh

c - khoang cach t truc trung tam en mep xa nhat cua tiet dien.
V ng suat phu thuoc phi tuyen vao tai trong nen kiem tra ben theo
ng suat cho phep khong am bao an toan theo he so n d kien. Trong

Chng 12: Uon ngang va uon doc ong thi

http://www.ebook.edu.vn

GV: Le c Thanh
trng hp nay, ngi ta dung ieu kien an toan theo tai trong nh phng
trnh (12.10).
BAI TAP CHNG 12
12.1 Tnh ng suat nen ln nhat theo phng phap gan ung cua dam chu
uon ngang va uon doc ong thi cho tren H.12.11.
q = 200 N/m

P = 257 kN

P = 4 kN

Po = 5 kN

q = 3 kN/m

1
4m

2m

4m

2m

1 - 1

2C N 20

100 E = 103 kN/cm2


100

2m

2m

b)

a)

11

Hnh 12.11
12.2

Cho dam chu lc nh tren H.12.9. Hay tnh ng suat phap ln nhat va
he so an toan n neu [] = 24 kN/cm2 . Tnh o vong ln nhat.
P1 = 1 kN

20 cm

q = 0,5 kN/m
P = 8 kN

40 cm

10 cm
10 cm

P = 4 kN
1m

2m

1m

E = 103 kN/cm2
b)

E = 2 x 104 kN/cm2
a)

Hnh 12.12

60

12.3 Tnh cng o tai trong cho


dam AB nh tren H.12.10, biet
o ben n = 1,6. Dam AB bang

phep tac dung len


he so an toan ve

q
B

cat hnh ong vi ng knh

5m

Hnh 12.13

ng knh ngoai D = 10 cm, vat


kN/cm2, khi tnh bo qua trong lng cua dam.

thep so 3 co mat
trong d = 6 cm va
lieu co [] = 24

Kiem tra on nh cua dam neu lay ko = 2. Cho E = 2.104 kN/cm2.

Chng 12: Uon ngang va uon doc ong thi

http://www.ebook.edu.vn

GV: Le c Thanh
Chng 13

TAI TRONG ONG


13.1 KHAI NIEM
1- Tai trong ong
Trong cac chng trc, khi khao sat mot vat the chu tac dung cua
ngoai lc, ta coi ngoai lc tac dung la tnh, tc la nhng tai trong gay ra gia
toc chuyen ong be, v vay khi xet can bang co the bo qua c anh hng
cua lc quan tnh.
Tuy nhien, cung co nhng trng hp ma tai trong tac dung khong the
coi la tnh v gay ra gia toc ln, v du nh s va cham gia cac vat, vat quay
quanh truc, dao ong... Khi nay, phai xem tac dung cua tai trong la ong, va
phai xet en lc quan tnh khi giai quyet bai toan.
2- Phng phap nghien cu
Khi giai bai toan tai trong ong, ngi ta tha nhan cac gia thiet sau:
- Vat lieu an hoi tuyen tnh
- Chuyen v va bien dang cua he la be.
Nh vay, nguyen ly cong tac dung van ap dung c trong bai toan tai
trong ong.
Khi khao sat can bang cua vat the chu tac dung cua tai trong ong,
ngi ta thng ap dung nguyen ly dAlembert. Tuy nhien, trong trng hp
vat chuyen ong vi van toc thay oi ot ngot nh bai toan va cham th
nguyen ly bao toan nang lng c s dung.
e thuan tien cho viec tnh he chu tai trong ong, cac cong thc thiet
lap cho vat chu tac dung cua tai trong ong thng a ve dang tng t
nh bai toan tnh nhan vi mot he so ieu chnh nham ke en anh hng
cua tac dung ong, goi la he so ong.
Trong chng nay ch xet cac bai toan tng oi n gian, thng gap,
co tnh chat c ban nham m au cho viec nghien cu tnh toan ong lc
hoc chuyen sau sau nay.

Chng 13: Tai trong ong

http://www.ebook.edu.vn
1

GV: Le c Thanh
13.2 THANH CHUYEN ONG VI GIA TOC LA HANG SO
Mot thanh tiet dien A co chieu dai L va trong lng rieng , mang mot
vat nang P, c keo len vi gia toc a nh H.13.1.a.
Tng tng cat thanh cach au mut mot

oan x. Xet phan di nh tren H.13.1.b, lc


tac dung gom co: trong lng vat nang P

Lc quan tnh cua oan thanh la

.A.1

Trong lng oan thanh Ax


Lc quan tnh tac dung tren vat P la

,A

.A.1a/g

P.a
g

Axa
g

Noi lc ong N tai mat cat ang xet.

P
a)

P
b)

P.a/g

Hnh 13.1
a) Vat chuyen ong len vi gia toc a
b) Noi lc va ngoai lc tac dung len
phan thanh ang xet

Theo nguyen ly dAlembert, tong hnh


chieu cua tat ca cac lc tac dung len thanh theo phng ng ke ca lc
quan tnh phai bang khong, ta c:
N Ax P

Pa
g

N = Ax + P +

Axa
g

=0

Pa
+ Axa
g
g

N = (Ax + P)(1 + a )
g

ai lng (Ax + P) chnh la noi lc trong thanh trang thai treo khong
chuyen ong, goi la noi lc tnh Nt.
N = Nt.(1 + a )

Ta c:

(13.1)

ng suat trong thanh:


d =

co the at:

Nd
N
= t
A
A

a
a
1 + = t 1 +
g
g

K = 1 + a

: He so ong

(13.2)
(13.3)

= tK

(13.4)

ng suat ln nhat tai mat cat tren cung cua thanh:

max = t,max.K
vi:

t = (AL + P)/A
ieu kien ben trong trng hp nay la:

max [ ]k

(13.5)

Ta thay co hai trng hp:


Chng 13: Tai trong ong

http://www.ebook.edu.vn
2

GV: Le c Thanh
- Khi chuyen ong len nhanh dan eu (gia toc a cung chieu chuyen
ong) va chuyen ong xuong cham dan eu (gia toc a ngc chieu chuyen
ong) he so ong K > 1, noi lc ong ln hn noi lc tnh.
- Ngc lai, khi chuyen ong len cham dan eu va chuyen ong xuong
nhanh dan eu th K < 1, noi lc ong nho hn noi lc tnh.
Du vay, khi mot vat the chuyen ong nh bai toan tren ay, phai tnh
toan thiet ke vi K > 1.
Th du 13.1 Mot thanh dai 10m co tiet dien vuong 30 cm x 30 cm va trong
lng rieng = 2500 kG/m3, c keo len vi gia toc a = 5 m/s2 (H.13.2).
Xac nh oan mut tha b e momen am tai goi ta bang momen dng tai
gia nhp. Ve bieu o momen, tnh ng suat phap ln nhat.
qqt = .A(1)a/g

Nd

qbt = .A(1)

a
b

L - 2b
qa
2

L - 2b

qa
2

b
2

q(L - 2b) qa
2
8

b)

a)

Hnh 13.2
a) Thanh c keo len vi gia toc a;

b) S o tnh va bieu o momen

Khi thanh c keo len vi gia toc a, thanh chu tac dung cua lc quan
tnh, khi o tai trong tac dung len he la tai trong phan bo eu, gom co:
q

= qbt + qqt = A(1) + A(1).a/g


= 2500(0,3.0,3) + 2500(0,3.0,3).5/10 = 337,5 KG/m

S o tnh cua thanh va bieu o momen cho H.13.2.b.


e momen tai goi bang momen gia nhp, ta co:
qb2
q( L 2b) 2 qb2
=

b = 0,206 L
2
8
2

vi b = 0,206L th momen ln nhat la:


337,5(0,206.10) 2
qb2
q(0,206 L) 2
=
=
= 716,11 KG.m
2
2
2
Mx
716,11.100.6
=
=
= 15,9 KG/cm 2
Wx
30.302

M x, max =
max

Chng 13: Tai trong ong

http://www.ebook.edu.vn
3

GV: Le c Thanh
13.3 VO LANG QUAY EU
Mot vo lang co be day , ng knh trung bnh D, tiet dien A, trong
lng rieng , quay quanh truc vi van toc goc khong oi (H.13.3.a).
q

,A,

a)

b)

Hnh 13.3 a) Tai trong tac dung len vo lang


b) Tach vo lang theo mat cat xuyen tam

Vi chuyen ong quay eu, gia toc goc


at = &

D
=0
2

&

= 0, gia toc tiep tuyen:

ch co gia toc phap tuyen hng tam la:

a n = 2

D
2

(a)

Mot oan dai n v cua vo lang co khoi lng A/g chu tac dung cua
lc quan tnh ly tam la:

q =

AD 2
A
.an =
2g
g

(b)

e tnh noi lc trong vo lang, dung mat cat tach vo lang theo mat cat
xuyen tam, xet can bang cua mot phan (H.13.3.b), do oi xng, tren mat cat
vo lang khong the co bien dang uon (do momen), bien dang trt (do lc
cat) ma ch co bien dang dai do lc doc, ngha la ch co ng suat phap .
V be day be, co the xem la phan eu, lc ly tam tac dung tren
chieu dai ds cua vo lang la q ds, phan to ds nh v bi goc , lay tong hnh
chieu theo phng ng, ta co:
2A = o qd ds sin

q = AD2/2g va ds = D d/2 vao, ta c:

thay:
d =

D 2 w2
4g

(13.6)

V ng suat trong vo lang la ng suat keo nen ieu kien ben vo lang:

[ ]k

(13.7)

Chu y. Khi tnh vo lang, ta a bo qua anh hng cua cac nan hoa noi truc
va vo lang, neu ke en th ng suat keo trong vo lang se giam, o phc tap
trong tnh toan tang len nhieu, khong can thiet lam trong tnh toan thc
hanh.

Chng 13: Tai trong ong

http://www.ebook.edu.vn
4

GV: Le c Thanh
V du 13.2 Mot truc ng ng knh D = 10 cm, trong lng rieng = 7850
kG/m3, mang mot khoi lng lech tam Q = 20 kG (H.13.4.a), truc quay vi
van toc n = 500 vong/phut. Kiem tra ben truc, tnh chuyen v tai iem at
khoi lng. Cho: [ ] = 1600 kG/cm2; E = 2.106 kG/cm2, a = 0,5m.
a

136,94 KGm

547,75 KG

Q
2 KG.m

1 KGm

20 KG

50,8 KG

61,6 KG

Nz

Mx,Qqt
a)

1 KGm

30,8 KG

Mx,Q

b)

Hnh 13.4

Giai. Van toc goc:


=

2n
= 2(3,14)500 / 60 = 52,33 rad/s
60

Lc quan tnh ly tam Qlt do trong lng Q la:


Q 2
e = 20.52,332.0,1 = 5476,85 N
g
Qqt = 547,68 KG
Qqt =

Bo qua anh hng do tac dung tnh cua trong lng Q va trong lng
ban than cua truc v chung nho so vi lc ly tam Qlt.
Momen do lc ly tam gay ra la (H.13.4.b):
Mxmax = QltL/4 = 547,68(1)/4 = 136,92 kGm
ng suat ln nhat cua truc:
max =

M x, max
Wx

136,92.100
= 1395,36 kG/cm2
3,14(10)2 / 32

Neu ke en trong lng ban than truc va tac dung tnh cua Q, tai tiet
dien gia truc chu tac dung cua cac noi lc nh sau (H.13.4.b)
Nz = 50,8 kG (nen); Mx = 135,92 kGm.
max =

M
Nz
30,8
136,92.100
+ x ,max =
+
2
3,14(10) / 4 3,14(10) 2 / 32
A
Wx

= 0,392 + 1395,75 kG/cm 2

Trong trng hp nay, trong lng ban than cua truc va tac dung tnh
cua Q co the bo qua.
Chuyen v do tac dung cua lc Qlt co the tnh theo cong thc sau:
y=

QL3
547,75.(100) 3
=
= 0,0116 cm
48EI x 48.2.10 6.3,14(10) 4 / 64

13.4 DAO ONG CUA HE MOT BAC T DO


Chng 13: Tai trong ong

http://www.ebook.edu.vn
5

GV: Le c Thanh
1- Khai niem
Mot he chuyen ong qua lai mot v tr can bang xac nh nao o, V du
qua lac ong ho, goi la he dao ong. Khi he chuyen t v tr can bang nay
sang v tr can bang ke tiep sau khi a qua moi v tr xac nh bi quy luat
dao ong, ta goi he a thc hien mot dao ong.
Chu ky la thi gian he thc hien mot dao ong, ky hieu la T tnh bang
giay (s).
Tan so la so dao ong trong mot giay, ky hieu la f, chnh la nghch ao
cua chu ky, f = 1 / T (1/s).
So dao ong trong 2 giay goi la tan so goc, hay con goi la tan so vong,
ky hieu la , ta thay = 2 / T (1/s).
Bac t do la so thong so oc lap xac nh v tr cua he oi vi mot he
quy chieu nao o. oi vi mot he dao ong nh tren H.13.5.a, v tr cua he
xac nh bi o dch chuyen (y) theo thi gian (t), he quy chieu se la (t,y).
Khi tnh mot he dao ong, ta can a ve s o tnh. Xac nh s o tnh
cua mot he da tren ieu kien phai phu hp vi he thc trong mc o gan
ung cho phep.
Xet dam cho tren H.13.5.a, neu khoi lng dam khong ang ke, co the
xem dam nh mot lien ket an hoi khong khoi lng, v tr cua he quyet nh
do v tr cua khoi lng vat nang, he co mot bac t do, v ch can biet tung
o y(t) cua vat nang la xac nh c v tr cua he tai moi thi iem (t). Vi
he H.13.5.b, bac t do la hai, v can phai biet y1(t), y2(t). oi vi truc chu
xoan (H.13.5.c), bac t do cung la hai, v can phai biet goc xoan 1(t), 2(t).
a)

y(t)

1(t)

2(t)

b)

y1(t)

y2(t)

c)

Hnh 13.5 a) He mot bac t do; b), c) He hai bac t do

Khi ke en khoi lng cua dam tren H.13.5.a, he tr thanh vo han bac
t do, v phai biet vo so tung o y(t) tai vo so iem khoi lng suot chieu dai
dam. Trong trng hp nay, can chon s o tnh thch hp, v du neu khoi
lng dam la nho so vi khoi lng vat nang, co the coi vat nang at tren
mot lien ket an hoi khong khoi lng, he co mot bac t do.
Chng 13: Tai trong ong

http://www.ebook.edu.vn
6

GV: Le c Thanh
Neu khong the bo qua
co the a ve he hu han bac
xem khoi lng dam gom N

mi

Hnh 13.6 He hu han bac t do

khoi lng dam,


t do, bang cach
khoi lng mi at

tren N iem nut cua thanh an hoi khong khoi lng (H.13.6), N cang ln,
o chnh xac tnh toan cang cao.
Mot he an hoi co the dao ong t do hay dao ong cng bc.
Dao ong cng bc la dao ong cua he khi chu mot tac ong bien oi
theo thi gian, goi la lc kch thch, ton tai trong suot qua trnh he dao ong
nh dao ong cua dam mang mot mot ien khi no hoat ong, khoi lng
lech tam cua roto gay ra lc kch thch.
Dao ong t do la dao ong do ban chat t nhien cua he khi chu mot
tac ong tc thi, khong ton tai trong qua trnh he dao ong nh dao ong
cua day an.
2- Phng trnh vi phan dao ong cng bc cua he mot bac t do
P(t)
M
y(t)
y

Hnh 13.7 He mot bac t do chu dao ong cng bc

Xet he mot bac t do chu tac dung mot lc kch thch thay oi theo thi
gian P(t) at tai khoi lng M (H.13.7), tai thi iem (t), o vong cua khoi
lng M la y(t). Gia thiet lc can moi trng ty le bac nhat vi van toc
chuyen ong, he so ty le .
Goi la chuyen v tai iem at khoi lng M do lc n v at tai o gay
ra. Chuyen v y(t) la ket qua cua cac tac ong:
- Lc kch thch P(t) gay ra chuyen v P(t)
- Lc quan tnh M &y&( t ) gay ra chuyen v M &y&( t )
- Lc can moi trng y& ( t ) gay ra chuyen v y& ( t )
ta c

y(t) = P(t) + [My(t) ] + [ y(t) ]

(a)

M &y&( t ) + y& ( t ) + y(t) = P(t).

(b)

Chia hai ve cho M va at:

= 2;
M

1
= 2
M

(c)

phng trnh (b) tr thanh:


&y&( t )

+ 2

y& ( t )

+ 2 y(t) = P(t).. 2

Chng 13: Tai trong ong

(13.8)

http://www.ebook.edu.vn
7

(b)

GV: Le c Thanh
(13.8) la phng trnh vi phan dao ong cng bc he mot bac t do.
3- Dao ong t do
Khi khong co lc kch thch va lc can bang khong, he dao ong t do,
phng trnh (13.8) tr thanh phng trnh vi phan cua dao ong t do:
&y&( t )

+ 2 y(t) = 0

(13.9)

Tch phan phng trnh (13.9), ta c nghiem tong quat co dang:


(d)
y(t) = C1 cost + C2 sint
S dung gian o cong cac vect quay (H.13.8), co the bieu dien ham
(a) di dang:
y(t) = A sin(t + )
Ham (e) la ham sin, chng
t do la mot dao ong tuan
goc , o lech pha . con
c tnh theo cong thc:
=

to
dao ong
hoan, ieu hoa.

C1

A=

Bien o dao ong la

(e)

C2

Hnh 13.8 Gian o cac vect quay

C12 + C22

tan

so

goi la tan so rieng

(13.10)

1
M

Goi P la trong lng cua khoi lng M, ta co M = P/g, thay vao (13.10),
ta c:

g
P

Tch so (P.) chnh la gia tr chuyen v tai iem at khoi lng M do


trong lng P cua khoi lng dao ong M tac dung tnh gay ra, goi la t.
Cong thc tnh tan so cua dao ong t do tr thanh:

g
t

(13.11)

Chu ky cua dao ong t do:

T =

2
=

g/t

(13.12)

4- Dao ong t do co can


Trong (13.8), cho P(t) = 0, ta c phng trnh vi phan cua dao ong
t do co can, he mot bac t do:
&y&( t )

+ 2

y& (t )

+ 2 y(t) = 0

(13.13)

Nghiem cua (13.13) tuy thuoc vao nghiem cua phng trnh ac trng:
K2 + 2K + 2 = 0
Khi: = 2 2 0, phng trnh ac trng co nghiem thc:

Chng 13: Tai trong ong

http://www.ebook.edu.vn
8

GV: Le c Thanh
K1,2 =

2 2

Nghiem tong quat cua (13.13) co dang:


y(t) = C1e K 1 t + C2e K 2 t

Ta thay ham y(t) khong co tnh tuan hoan, do o he khong co dao ong,
ta khong xet trng hp nay.
Khi: = 2 2 < 0, at: 12 = 2 2, phng trnh ac trng co
K1,2 =

nghiem ao:

i1

Nghiem tong quat cua (13.13) co dang:


y(t ) = A1e t sin( 1t + 1 )

Ham y(t) la mot ham sin co tnh tuan hoan, the hien mot dao ong vi
tan so goc 1, o lech pha 1, bien o dao ong la mot ham mu am A1et,
tat rat nhanh theo thi gian.
Tan so dao ong 1 =

2 2

, nho hn tan so dao ong t do (H.13.9).

y
Hnh 13.9 o th ham so dao ong t do co can

4- Dao ong cng bc co can


T phng trnh vi phan dao ong cng bc co can he mot bac t do
(13.8):

q &y&( t ) + 2

y& ( t )

+ 2 y(t) = P(t)2

(f)

Vi cac bai toan ky thuat thong thng, lc kch thch P(t) la mot ham
dang sin, do o co the lay P(t) = Po.sinrt, khi o phng trnh vi phan (f) co
dang:
&y&( t )

+ 2

y& ( t )

+ 2 y(t) = 2Po sinrt (13.14)

Nghiem tong quat cua (13.14) co dang:


y(t) = y1(t) + y2(t)
trong o: y1(t) - la mot nghiem tong quat cua (13.14) khong ve phai, chnh la
nghiem cua dao ong t do co can (e):
y1(t) = A1et sin(1 t + 1)

Chng 13: Tai trong ong

(g)

http://www.ebook.edu.vn
9

GV: Le c Thanh
y2(t) - la mot nghiem rieng cua (13.14) co ve phai, v ve phai la mot
ham sin, do o co the lay y2 (t) dang sin:
y2(t) = C1 cosrt + C2 sinrt
(h)
vi: C1 va C2 - la cac hang so tch phan, xac nh bang cach thay y2(t) va
cac ao ham cua no vao (13.14), roi ong nhat hai ve. S dung
gian o vect quay bieu dien (h) di dang:
y2 (t) = V sin(rt + )

(i)

Nh vay, phng trnh dao ong cua he la:


y (t) = A1et sin(1 t + 1) + V sin(rt + ) (j)
Phng trnh (j) chnh la o vong y(t) cua dam.
So hang th nhat cua ve phai trong (j) la mot ham co bien o tat rat
nhanh theo quy luat ham mu am, sau mot thi gian ngan, he dao ong theo
y (t) = V sin(rt + )

quy luat:

(13.15)

o la mot ham sin bieu dien mot dao ong tuan hoan, ieu hoa, tan so
goc cua dao ong bang tan so lc kch thch r, o lech pha , bien o dao
ong V (H.13.10).
t
V= ymax

Hnh 13.10 o th bieu dien dao ong cng bc co can

Bien o dao ong chnh la o vong cc ai cua dam ymax, ta co:


V = ymax =

C12 + C 22

(k)

Tnh cac gia tr cua C1 va C2, thay vao (k), ta c o vong cc ai cua
dam:

ymax =

Po

4 2 r 2
r
(1 2 )2 +

(h)

Tch so Po chnh la gia tr cua chuyen v tai iem at khoi lng M do


lc co gia tr Po (bien o lc kch thch) tac dung tnh tai o gay ra, at la yt,
ta co:
ymax = yt

co the viet la:

1
2

4 2 r 2
r
(1 2 )2 +

(13.16)

ymax = yt.K

Chng 13: Tai trong ong

http://www.ebook.edu.vn
10

GV: Le c Thanh
vi:

(13.17)

K =
(1

) +
2

4 2 r 2

K c goi la he so ong, the hien anh hng cua tac dung ong so
vi tac dung tnh ng vi tr so cua bien o lc.
5- Hien tng cong hng
Khao sat s bien

ong K cong thc

5,0

coi K la mot ham hai

4,0

(r/,2/). ng vi mot
(K, r/) co dang hnh
tai hoanh o

nhanh, vi = 0, gia tr
cc (H.13.11), ngha la
vo cung.

dien

quan

he

chuong ma nh
lt cho

1,0

bien

1, lan

dan, ta thay nh cua o

(13.17) bang cach

2,0

gia tr khac nhau ng vi

gia t xac nh 2 ,

3,0

ta ve c o th bieu
r
=
w

thien cua he so

2
w

nhieu

he so can giam
0,5

1,0

1,5

2,0

Hnh 13.11 o th ham so K = f(r/w; 2a/w)


vi 2 a/w la cac hang so cho trc

th

(K)

tang

cua K tien en vo
o vong dam ln

Hien tng bien o dao ong tang ot ngot khi tan so lc kch thch
bang tan so rieng cua he an hoi goi la hien tng cong hng. Tren o th
con cho thay khi hai tan so nay xap x nhau (r/ [0,75 1,5]), bien o tang
ro ret, ngi ta goi la mien cong hng. Hien tng cong hng ro rang rat
nguy hiem cho chi tiet may hay cong trnh lam viec trong mien cong hng,
do o trong thiet ke, ta phai tnh toan sao cho he dao ong nam ngoai mien
cong hng.
o th cho thay nen chon ty so r/ ln hn 2, khi o K nho hn 1, bai
toan ong t nguy hiem hn bai toan tnh. e co r/ ln, thng phai giam ,
ngha la chuyen v t phai ln. Muon vay, phai giam o cng cua thanh an
hoi, ieu nay nhieu luc mau thuan vi yeu cau o ben cua cong trnh. e
tranh lam giam o cng cong trnh co the at lo xo hay loai vat lieu co kha
nang phat tan nang lng em gia khoi lng dao ong va thanh an hoi.
Co trng hp khi khi ong mo t, toc o mo t tang dan en toc o
on nh, mot thi gian ngan ban au cong trnh co the trong mien cong

Chng 13: Tai trong ong

http://www.ebook.edu.vn
11

GV: Le c Thanh
hng, can phai dung loai ong c tang toc nhanh e hien tng cong
hng neu co xay ra cung ch trong thi gian rat ngan.
Neu khi hoat ong, cong trnh dao ong vi K ln, can tnh toan ky e
s dung cac bo giam chan lam tieu hao nang lng dao ong hay tang he
so can.
Tren H.13.11, ta thay, khi ty so r/ [0,5 2], cac ng cong K gan
trung nhau, he so can xem nh khong anh hng, hoac khi he so can
khong ang ke, co the tnh K theo cong thc:
(13.18)

Kd =
1

r2

V cac ai lng nh chuyen v, noi lc hay ng suat ty le bac nhat vi


ngoai lc, ta co the viet:
d = t K d + t ,ds

(13.19)

d = t K d + t ,ds
M d = M t K d + M t ,ds

trong o:

t, t - la cac ng suat do tai trong co gia tr bang bien o lc


kch thch (Po) tac dung tnh

t,s, t,s - la cac ng suat do tai trong tnh at san, ma khi khong co
dao ong no van ton tai nh trong lng ban than mot.
ieu kien ben:
max [ ] hay max [ ]
(13.20)
6- Phng phap thu gon khoi lng
Khi phai ke en khoi lng dam (cac lien ket an hoi) anh hng qua
trnh dao ong va khong oi hoi o chnh xac cao, co the tnh gan ung nh
he mot bac t do theo phng phap thu gon khoi lng nh sau.
Xet mot dam ta n (H.13.12) khoi lng M tai gia nhp, gia s khoi
lng dam u nho e khong lam thay oi dang dao ong nh khi ch co mot
khoi lng M, neu goi y(t) la o vong cua M tai gia nhp, ta co:
y(t) = PL3 /48EIx

L/2

L/2

Hnh 13.12 Dam n dao ong co ke en khoi lng dam

o vong tai mat cat tai hoanh o z se la:


y( z) =

PL2 z
Pz 3
Lz 4 z3

= y(t)
16 EI x 12 EI x
L3

Chng 13: Tai trong ong

http://www.ebook.edu.vn
12

GV: Le c Thanh
Goi q la trong lng 1 m dai cua dam, ong nang cua mot phan to khoi
lng dai dz cua dam la:
dT =

2
3
1 qdz 3L z 4 z
2
2 g
L3

( )

dy2
dt2

ong nang cua toan dam la:

2
3
dy2
1 qdz 3L z 4 z
T = 2.
2
2
g L3 dt 2

( )

T =

1 17 qL dy2
.
2 35 g dt 2

(13.21)

ong nang cua toan dam tng ng ong nang cua mot khoi lng
m = (17/35)(qL/g) at tai gia dam. Nh vay, tren c s tng ng ong
nang, co the xem he la mot bac t do vi khoi lng dao ong tai gia dam
17 qL
la:
M1 = m +
.
(13.22)
35

trong o: qL/g - chnh la khoi lng cua toan bo dam.


Goi la he so thu gon khoi lng. Ta co:
- oi vi dam n (H.13.12), khoi lng thu gon tai gia nhp,

= 17/35
- oi vi dam cong xon (H.13.12a), khoi lng thu gon tai au t do,
= 33/140.
- oi vi lo xo dao ong doc, thanh thang dao ong doc (H.13.14), khoi
lng thu gon tai au t do, = 1/3.
= 33 /140
a)

Hnh 13.12a

b)

c)
= 1/3

Hnh 13.13

Po

N = 600vg/ph
L =2m
PL

PoL

I-16:

Po

= 1/3

Hnh 13.14

Hnh 13.15 a) Dam cong xon I-16 mang mot mo t


b) va c) S o tnh va bieu o mo men do trong
lng mo t P va l
o c ly tam P

V du 13.3 Mot dam cong xon tiet dien I-16 mang mot mo t trong lng
P = 2,5 kN, van toc 600 vong/phut, khi hoat ong mo t sinh ra lc ly tam
0,5 kN (H.13.15). Bo qua trong lng dam, tnh ng suat ln nhat, o vong
ta au t do. Neu ke en trong lng dam q, tnh lai ng suat va o vong.
Cho: E = 2.104 kN/cm2; he so can = 2(1/s).
Giai. Theo so lieu e bai, ta thay khi mo t hoat ong th dam chu tac dung
mot lc kch thch dang sin P(t) = Posinrt, vi Po = 0,5 kN va tan so goc r.
a) Khong ke en trong lng dam
Chng 13: Tai trong ong

http://www.ebook.edu.vn
13

GV: Le c Thanh
ng suat ong:

d = t ,Q K d + t , ds

He so ong:

Kd =

1
(1

)2 +

4 2 r 2

r = 2n/60 = 2600/60 = 62,8 rad/s;

trong o:

g
t

g = 10 m/s2 = 1000 cm/s2

vi:

t =
ta c:

2,5(300) 3
PL3
=
= 1,19cm
3EI x 3.2.10 4.945
g
t

Kd =

1000
1,19

= 29

1
62,8 2 4.2 2 62,8 2
(1
) +
29 2
29 4
2

= 0,27

T bieu o momen do trong lng P (H.13.15), ta thay tai ngam momen


ln nhat, do o ng suat ln nhat do tai trong at san tren dam la:
ds ,max =

M x ,max,P
Wx

PL 2,5.3.100
=
= 6,35 kN/cm 2
118
Wx

ng suat do Po tac dung tnh c tnh tng t:


t ,max =

Po L 0,5.3.100
=
= 1,27 kN/cm 2
118
Wx

ng suat ong ln nhat:


d = 1,27(0,27) + 6,35 = 6,69 kN/cm 2

Chuyen v do trong lng at san tai au t do la:


yt,P = t = 1,19 cm
suy ra chuyen v do Po tac dung tnh tai au t do la:
yt, Po =

0,5
1,19 = 0,238 cm
2,5

Chuyen v ong ln nhat tai au t do, ta co:


yd = 0,238(0,27) + 1,19 = 1,25 cm

b) Ke en trong lng dam


e a he ve mot bac t do, ta dung phng phap thu gon khoi lng.
Coi dam khong trong lng va au t do co at mot khoi lng:
m=

33 AL
140 g

ngha la tai o co them mot trong lng bang:

33
aAL = 0,119 kN
140

Chuyen v tnh do khoi lng dao ong la:

t =

( P + 0,119) L3 (2,5 + 0,119)(300) 3


=
= 1,247 cm
3EI
3.2.10 4.945

Chng 13: Tai trong ong

http://www.ebook.edu.vn
14

GV: Le c Thanh
ta c:

g
=
t

1000
= 28,31
1,247

Kd =

= 0,25

62,8 2 4.2 2 62,8 2


) +
(1
28,314
28,312

T bieu o momen do trong lng P (H.13.15), ta thay tai ngam


momen ln nhat, ng suat ln nhat do tai trong at san tren dam co ke
them trong lng ban than la:
ds ,max =
ds ,max =

M x ,max,P
Wx

( PL + qL2 / 2)
Wx

(2,5.3 + 0,169.32 / 2).100


= 7 kN/cm 2
118

ng suat do Po tac dung tnh khong khac phan tren la 1,27 kN/cm2.
ng suat ong ln nhat:
d = 1,27(0,25) + 7 = 7,31 kN/cm 2

Chuyen v do trong lng at san tai au t do gom trong lng mot


va phai ke them do trong lng ban than la:
yt,P = PL3/3EIx + ql4/8EIx = 1,19 + 0,307 = 1,497 cm
con chuyen v do Po tac dung tnh tai au t do van la 0,238 cm.
Chuyen v ong ln nhat tai au t do, ta co:
d = 0,238(0,25) + 1,497 = 1,556 cm

V du 13.4 Mot dam


40, mang mot mot
2,5 kN, van toc 600
hoat ong mo t sinh
kN (H.13.16). Ke en
tnh ng suat ln nhat,

Po

n = 600vg/ph

I 40

P
P

PL/ 4

Cho: E = 2.104

= 2(1/s); thep I40 co


Wx = 947 cm3, trong
q = 0,56 kN/m.

thep tiet dien I trong lng P =


vong /phut, khi
ra lc ly tam 0,5
trong lng dam,
o vong cua dam.
kN/cm2; he so can

qL2/8

Hnh 13.16 a) Dam n I40 mang mot mo t


b) va c) S o tnh va bieu o momen do
trong lng mo t P va trong lng ban than

Ix = 19840 cm4,
lng

met

dai

Giai. Theo so lieu e bai, ta thay khi mo t hoat ong th dam chu tac dung
mot lc kch thch dang sin P(t) = Posinrt, vi Po = 0,5 kN va tan so goc r.
ng suat ong: d = t ,Q K d + t ,ds
He so ong:

Kd =
(1

Chng 13: Tai trong ong

)2 +

4 2 r 2

http://www.ebook.edu.vn
15

GV: Le c Thanh
trong o: r = 2n/60 = 2..600/60 = 62,8 rad/s;

g
t

vi: g = 10 m/s2 = 1000 cm/s2.


o vong tai gia dam do lc tap trung P la: t =

PL3
48EI x

Ke en trong lng dam, phai a dam ve mot bac t do, ta dung


phng phap thu gon khoi lng. Coi dam khong trong lng va gia dam
co at mot khoi lng: m = 17 AL
35 g

ngha la tai o co them mot trong lng bang:


17
aAL = 0,56(12) = 6,72 kN
35

khi o chuyen v tnh do khoi lng dao ong la:


t =
ta c:

(2,5 + 6,72) L3
(9,22)(1200) 3
=
= 0,876 cm
48EI x
48.2.10 4.18930
g
=
t

Kd =

1000
= 33,77
0,876

1
2

62,8 2 4.2 2 62,8 2


(1
) +
33,77 2
33,77 4

= 0,405

T bieu o momen do trong lng P va do trong lng ban than q


(H.13.16), ta thay tai gia nhp momen ln nhat, ng suat ln nhat do tai
trong at san tren dam co ke them trong lng ban than la:
ds ,max =
ds ,max =

M x ,max,P
Wx

( PL / 4 + qL2 / 8)
Wx

(2,5.12 / 4 + 0,56.12 2 / 8).100


= 1,856 kN/cm 2
947

ng suat do Po tac dung tnh la:


t , Po =

Po L 0,5.(12)100
=
= 0,158 kN/cm 2
4Wx
4(947)

ng suat ong ln nhat:


d = 0,158(0,405) + 1,856 = 1,92 kN/cm 2

Chuyen v do trong lng at san tai gia nhp gom trong lng mo t
va phai ke them do trong lng ban than la:
yt , p =

PL3
5qL4
+
= 0,237 + 0,4 = 0,637 cm
48EI x 384 EI x

con chuyen v do Po tac dung tnh tai gia nhp la:


0,237 x (0,5/2,5) = 0,0474 cm
Chuyen v ong ln nhat tai gia nhp, ta co:
yd = 0,0474(0,405) + 0,637 = 0,656 cm

Chng 13: Tai trong ong

http://www.ebook.edu.vn
16

GV: Le Hoang Tuan


13.5 TOC O TI HAN CUA TRUC
Mot truc quay mang mot pu li khoi lng M, quay
eu vi van toc goc , goi o vong cua truc tai pu li la y,
gia s trong tam cua pu li lech tam so vi tam truc la e
(H.13.17).

y
e

Hnh 13.17 Truc quay mang khoi lng lech tam


Lc ly tam tac dung len truc:
F = M 2 (e + y)
Goi la chuyen v tai v tr pu li do lc n v gay ra,
ta co, chuyen v gay ra bi lc ly tam F la:
y = M2(e + y)
suy ra

y=

e
1
2
M
2

(a
(13.23)

Theo cong thc (13.23), o vong truc cc ai khi


2 =

1
M

, ngha la khi toc o cua truc bang tan so rieng

1
M

, goi la toc o ti han cua truc quay. Khi truc

lam viec toc o gan toc o ti han, o vong ln, chi tiet
may co tieng on, nen trong thiet ke phai tnh toan sao
cho toc o khac xa toc o ti han.
Nhan xet rang, neu toc o truc 2 ln hn nhieu so
vi (1/ M.), cong thc (13.23) chng to o vong y e,
trong tam cua pu li gan trung vi tam truc, truc trang
thai lam viec tot nhat.
Chng 13: Tai trong ong

http://www.ebook.edu.vn

GV: Le c Thanh
13.6 DAO ONG CUA HE HAI BAC T DO
Xet mot he co 2 bac t do nh tren H.13.18. Nhieu
bai toan thc tien co the a ve s o tnh nay.
Goi y1(t), y2(t) la chuyen v cua M1, M2; ij la chuyen
v tai iem i do lc n v at tai iem j gay ra. Co the
chng minh ij = ji.
Ta co: y1(t) = 11 (M1 y1) + 12 (M2 y2)
y2(t) = 21 (M1 y1) + 22 (M2 y2)
Nghiem tong quat cua (a) co dang: Hnh 13.18

(a)

He hai bac t do

y1(t) = A1sin(t + )

y2(t) = A2sin(t + )
thay (b) vao (a), ta c he phng trnh thuan nhat:
A1 (11 M1 2 1) + A2 12 M2 2 = 0
A121 M1 2 + A2(22 M2 2 1) = 0
(c)
e A1, A2 khac khong th nh thc cac he so cua (c)
phai bang khong:
(11 M12 1)
(12 M 2 2 )
2
( 21 M1 )
( 22 M 2 2 1)

=0

(d)

t (d), va 12 = 21, ta c:
4M1M2(1122 212) 2 (11M1 + 22M2) + 1 =
0 (e)
Phng trnh (e) goi la phng trnh tan so, giai (e),
ta xac nh c hai tan so rieng xep th t t nho en
ln 1, 2. Nh vay, he co hai bac t do se co hai tan so
rieng.
ng vi tan so 1, theo (b), phng trnh dao ong
co dang:
Chng 13: Tai trong ong

http://www.ebook.edu.vn

(b

GV: Le c Thanh
y1(t) = A11sin(1t + 1)
y2(t) = A21sin(1t + 1)
ng vi tan so 2, theo (b), phng trnh dao ong
co dang:
y1(t) = A12sin(2t + 2)
y2(t) = A22sin(2t + 2)
Khi he dao ong vi tan so 1, ta co the chng
minh he dao ong ieu hoa cung pha (H.13.19.a), goi la
dang dao ong chnh th nhat.
y1

y2

y1

y2

b)

a)

Hnh 13.19

a) Dang dao ong chnh th nhat


b)Dang dao ong chnh th hai

Khi he dao ong vi tan so 2, ta co the chng


minh he dao ong ieu hoa lech pha 180o (H.13.19.b),
goi la dang dao ong chnh th hai.
Dao ong cua ca he mot dao ong phc hp co
phng trnh:
y1(t) = A11sin(1t + 1) + A12sin(2t + 2)
y2(t) = 1 A11sin(1t + 1) - 2 A12sin(2t + 2) (f)
(f) khong phai la mot dao ong ieu hoa, nhng co the
bieu dien theo cac dang chnh.
13.7 PHNG PHAP RAYLEIGH
oi vi he nhieu bac t do, viec xac nh tan so
rieng bang phng phap chnh xac rat phc tap, do o
Chng 13: Tai trong ong

http://www.ebook.edu.vn

GV: Le c Thanh
trong mot so trng hp ngi ta dung phng phap
gan ung. Trong phan nay, ta xet phng phap
Rayleigh.
mi

Coi dam nh mot thanh an


hoi mang n khoi lng Mi, moi
khoi lng bang khoi lng cua
tng oan thanh dam (H.13.20).

Hnh 13.20 He n bac t do

Gia s he dao ong t do vi cac dang chnh, khi o


phng trnh chuyen ong cua mot khoi lng Mi la mot
ham ieu hoa, co the viet:
yi(t) = Aisin(t + )
van toc cua Mi la:

dyi (t)
= Ai cos(t + )
dt

Khi he v tr can bang y(t) = 0, van toc cc ai, the


nang bien dang an hoi luc o bang khong, ong nang
he ln nhat co gia tr bang:
T =

2
2

Mi yi2

Khi he xa v tr can bang nhat, van toc bang


khong, the nang cc ai. Goi phng trnh ng an
hoi cua dam la y(z).
V:

y =

M
EJ

M = EI y

ap dung cong thc tnh the nang bien dang an hoi cua
dam, ta c:
1
U =
2

d 2 y( z)
dz
EI
2
dz

theo nguyen ly bao toan nang lng, T = U, ta c:


Chng 13: Tai trong ong

http://www.ebook.edu.vn

GV: Le c Thanh
2
2

M i yi2

1
=
2

tan so rieng la:

d 2 y( z)
dz
EI
2
dz

2 =

1
2

d 2 y( z)
dz
EI
2
dz
Mi yi2

(13.24)

Vi dam n, tiet dien eu, trong lng phan bo q =

A, ng an hoi do tai trong ban than la:


y( z) =

q
( z4 4 Lz 3 + 6 L2 z2 )
24 EI

khi dam dao ong, co the chon dang a thc nh tren:


y(z) = z4 4Lz3 + 6L2 z2
Ap dung phng phap Rayleigh ta tnh c tan so
cua dao ong chnh th nhat la:

1 =

3,49 EIg
L2
A

So vi gia tr giai theo phng phap chnh xac la:


1 =

3,52 EIg
L2
A

th sai so la 1% u nho, chap nhan c trong ky thuat.


13.8 VA CHAM CUA HE MOT BAC T DO
1- Va cham ng
Xet mot dam mang vat nang P va chu va cham bi
vat nang Q, ri theo phng thang ng t o cao H vao
vat nang P nh tren H.13.21. Trong lng ban than cua
dam c bo qua. Gia thiet khi vat Q va cham P ca hai
vat cung chuyen ong them xuong di va at chuyen
v ln nhat y.
Q
H
y0

y
He mot bac t do chu va cham ng

Chng 13: Tai trong ong


Hnh 13.21

http://www.ebook.edu.vn

GV: Le c Thanh

Chuyen v cua vat nang P do trong lng ban than


cua no c ky hieu la

y0 .

Goi Vo la van toc cua Q ngay trc luc cham vao P,


V la van toc cua ca hai vat P va Q ngay sau khi va
cham. Ap dung nh luat bao toan ong lng trc va
ngay sau khi va cham, ta c:
QVo
(P + Q) V
=
g

hay

V =

Q
Vo
P+Q

(a)

Trong bai toan nay, ta da vao phng phap nang


lng e tm chuyen v trong dam.
Ta goi trang thai 1 tng ng vi khi vat Q va
cham vao vat P va ca hai cung chuyen ong xuong di
vi van toc V (luc nay chuyen v la

y0 ).

Trang thai 2

tng ng vi khi Q va P at ti chuyen v tong cong


y 0 + y .
ong nang cua vat P va Q trang thai 1 ngay sau
khi va cham:
2

1
1 (P + Q ) Q
1 Q2
2

T1 = mV =
V
=
Vo2
o

2
2 g P+Q
2 g (P + Q )
ong nang cua vat P va Q trang thai 2:
Chng 13: Tai trong ong

http://www.ebook.edu.vn

GV: Le c Thanh
T2 =

1 2 1 (P + Q ) 2
mv =
0 =0
2
2 g

o giam ong nang khi he chuyen t trang thai 1 sang


trang thai 2 la:
T = T1 T2 =

(b)

1 Q2
Vo2
2 g (P + Q )

o thay oi the nang khi he chuyen t trang thai 1 sang


trang thai 2 la:
= mgh =

P+Q
g ( y + y0 y0 ) = ( P + Q) y
g

(c)

Theo nguyen ly bao toan nang lng, khi he chuyen


t trang thai 1 sang trang thai 2, o thay oi c nang
cua vat P va Q se chuyen thanh the nang bien dang
an hoi U tch luy trong dam.
U=T+

(1

Tnh U da vao quan he gia lc va chuyen v trong


dam nh tren H.13.22. trang thai 1, trong dam tch luy
mot the nang bien dang an hoi U1 c tnh nh sau:
U1 =

at =

y0
P

1
Py0
2

Lc

la chuyen v tai

iem va cham do lc n v
gay ra. The vao bieu thc
tren ta co:
U1 =

1 2
y0
2

trang thai 2, the nang bien

y0

y0+y

Chuyen v

Hnh 13.22. o th tnh TNBDH

dang an hoi U2 trong dam la:


Chng 13: Tai trong ong

http://www.ebook.edu.vn

GV: Le c Thanh
1 ( y + y0 )
2

U2 =

Nh vay khi he chuyen t trang thai 1 sang trang thai 2,


the nang bien dang an hoi trong dam c tch luy
them mot lng:
U = U 2 U1 =
U =

1
2

{( y

+ y0 ) y02 =
2

1 2
y + 2 y y 0
2

y2
+ Py
2

(d)

Thay cac bieu thc (b), (c), (d) vao (13.25) ta co:
y 2
1 Q 2Vo2
+ y P =
+ (P + Q ) y
2
2 g (P + Q )

Goi yt la chuyen v cua


dam tai iem va cham do
trong lng Q tac dung tnh
tai o gay ra nh tren
H.13.23. Thay yt = Q vao

yt

Hnh 13.23. S o tnh chuyen v yt

phng trnh tren, ta c:


y2 2 yt y

yt Vo2
= 0
g (1 + P / Q)

(e)

Nghiem cua phng trnh bac hai (e) la:


yd = yt yt2 +

ytVo2
P
g (1 + )
Q

V y > 0, nen ch chon nghiem dng cua (e), tc la:

2
Vo
ytV
=K y
= yt 1 + 1 +
y d = yt + yt2 +
t
P
P
gyt (1 + )
g (1 + )

Q
Q

2
o

(13.26)

Do o he so ong c tnh bi:

Chng 13: Tai trong ong

http://www.ebook.edu.vn

GV: Le c Thanh
Kd = 1+ 1+

V02
gyt (1 +

(13.27)

P
)
Q

Khi vat Q ri t do t o cao H xuong dam, tc la


Vo = 2 gH , thay vao (13.27):
Kd = 1+ 1+

(13.28)

2H
P
yt (1 + )
Q

Khi tai iem va cham khong co trong lng at san


P = 0, he so ong tang len:

Kd = 1 + 1 +

2H
yt

(1

Khi P = 0, H = 0, ngha la trong lng Q at ot ngot


len dam:
K = 2

(1

Theo (13.29), khi yt cang ln, ngha la o cng cua


thanh cang nho, th K cang nho, do o s va cham
cang t nguy hiem.
e am bao ieu kien ben, ngi ta co the lam tang
yt bang cach at tai iem chu va cham nhng vat the
mem nh lo xo hay tam em cao su...
Khi a tnh c K, co the tnh ai lng S khac
trong he tng t nh chuyen v, ngha la:
(13.31)

S tp = K S tQ + S P
S tQ

la ai lng can tnh (noi lc, ng suat) do Q coi

nh at tnh len he tai mat cat va cham gay ra.


S tP

la ai lng can tnh (noi lc, ng suat) do cac

tai trong hoan toan tnh at len he gay ra.

Chng 13: Tai trong ong

http://www.ebook.edu.vn

GV: Le c Thanh
ieu kien ben: ,max []
Chu y:
Neu chon moc the nang bang khong v tr dam
khong bien dang, th c nang ban au cua he chnh la
the nang:
= QH

Ngay sau khi va cham, P va Q cung chuyen ong


xuong di vi van toc V th c nang cua he chnh la
ong nang:
T =

Q
1 P + Q 2 1 Q2
V =
Vo2 =
QH <
(P + Q )
2 g
2 g (P + Q )

Nh vay a co s mat mat nang lng tng ng vi gia


thiet va cham mem tuyet oi cua 2 vat the; nang lng
nay lam cho 2 vat the bien dang hoan toan deo, ap sat
vao nhau va chuyen ong cung van toc ve pha di.
2- Va cham ngang
Xet mot dam mang vat nang P.
Vat nang Q chuyen ong ngang vi
van toc V0 va cham vao vat nang P
nh tren H.13.24. Trong lng ban
than cua dam c bo qua. Gia

Vo P
Q
y

Hnh 13.24. He mot bac t do chu va cham ngang

thiet khi vat Q va cham P ca hai vat cung chuyen ong


ngang va at chuyen v ln nhat y.
Lap luan nh trng hp va cham ng, ta cung co:

Chng 13: Tai trong ong

http://www.ebook.edu.vn

GV: Le c Thanh
Van toc cua hai vat P, Q cung chuyen ong ngay
sau khi va cham la:
V =

Q
Vo
P+Q

o giam ong nang trong he:

T=

1 Q2
Vo2
2 g (P + Q )

V hai vat chuyen ong theo phng ngang, nen


khong co s thay oi the nang, tc la:
=0
The nang bien dang an hoi tch luy trong he la:
U =

y2
2

Nguyen ly bao toan nang lng, T+ = U, ta c


phng trnh sau:
y2
1 Q2
Vo2 =
2 g (P + Q )
2

Lay gia tr nghiem dng cua y, ta c:


y =

Ta lai co

yt
Q

QVo2

(13.32)

P
g 1 +
Q

, vi yt la chuyen v ngang cua dam

tai iem va cham do trong lng Q tac dung tnh nam


ngang tai o. Thay vao phng trnh (13.32) nh sau:
y = yt

Vo
P
gyt (1 + )
Q

= yt K

(13.33)

He so ong:
K =

Vo
P
gyt (1 + )
Q

(13.34)

Chng 13: Tai trong ong

http://www.ebook.edu.vn

GV: Le c Thanh
Khi khong at san trong lng chu va cham, tc P
= 0, he so ong la:
K =

(13.35)

Vo
gyt

Khi o, noi lc, ng suat cung c tnh nh sau:


M = Mt.K

= t.K

Q = 1 kN

...............
ieu kien ben: ,max [ ]

H = 0,5 m
L=2m

V du 13.5 Mot
dam
cong xon tiet dien ch
nhat (20 40) cm chu va
cham ng bi mot trong

a)

b)

Q.L
Mx,Q
Q.L2
2

M x,q

Hnh 13.25 Dam cong xon chu va cham

lng Q = 1 kN ri t do t

o cao H = 0,5 m
(H.13.25.a). Bo qua trong lng ban than dam, tnh ng
suat va o vong ln nhat cua dam. Neu ke en trong
lng ban than dam q, tnh lai ng suat va o vong. Neu
at tiet dien dam nh (H.13.25.b), tnh lai ng suat va
o vong. Cho:

E = 0,7.103 kN/cm2; q = 0,64 kN/m.

Giai. ng suat ong:


d = t ,Q K d

vi:

Kd = 1+ 1+

2H
yt

Khong ke trong lng ban than dam, ta co:


yt =

QL3
=
3EI x

1(200) 3
= 0,0357 cm
20.40 3
3(0,7.10 3 )
12

Chng 13: Tai trong ong

http://www.ebook.edu.vn

GV: Le c Thanh
He so ong :

Kd = 1+ 1+

2(50)
= 53,93
0,0357

ng suat ln nhat tai ngam (H.13.25):


d ,max = t ,max,Q K d =
=

M x ,max
Wx

Kd =

Q.L
Kd
Wx

1(200)
(53,93) = 2,02 kN/cm 2
20.40 2 / 6

o vong ln nhat tai au t do:


y max = yt ,max,Q K d = 0,0357(53,93) = 1,92 cm

Khi ke en trong lng ban than, co the dung


phng phap thu gon khoi lng, khi o coi nh dam
khong trong lng va tai au t do co mot trong lng la
(33/140)qL = 0,3 kN (qL la trong lng dam).
He so ong se la:
Kd = 1+ 1+

2H
P
yt (1 + )
Q

= 1+ 1+

2(50)
0,0357(1 +

0,3
)
1

= 47,43

ng suat do va cham la:


d ,max = t ,Q K d =

1(200)
.47,43 = 1,78 kN/cm 2
20.40 2 / 6

Ke them ng suat do trong lng dam:


d ,max =

M t ,max,q
Wx

qL2 / 2 0,64.2 2.100


=
= 0,024 kN/cm 2
2
Wx
20.40 / 6

ng suat ln nhat trong dam la:


max = 1,78 + 0,024 = 1,804 kN/cm2
Khi ke en trong lng dam, ng suat ln nhat
giam.
o vong tai au t do
o vong do trong lng ban than:
yt =

0,64.10 2 (200) 4
qL4
=
= 0,017 cm
3
8EI x
3 20.40
8(0,7.10 ).
12

Chng 13: Tai trong ong

http://www.ebook.edu.vn

GV: Le c Thanh
o vong khi co va cham:
y d ,max = yt ,max,Q K d + yt ,q = 0,0357.47,43 + 0,017 = 1,71 cm

Neu at tiet dien dam nh (H.13.25.b), ta c:


Khong ke trong lng dam:
yt =

He so ong :

QL3
=
3EI x

1.( 200) 3
= 0,143 cm
3
3 40.20
3(0,7.10 ).
12

Kd = 1+ 1+

2(50)
= 27,46
0,143

ng suat ln nhat tai ngam :


d ,max = t ,max,Q K d =
=

M x ,max
Wx

Kd =

QL
Kd
Wx

1.(200)
(27,46) = 2,06 kN/cm 2
40.20 2 / 6

o vong tai au t do:

yt = 0,143.(27,46) = 3,93 cm

Ke en trong lng ban than, ta dung phng


phap thu gon khoi lng, khi o coi nh dam khong
trong lng va tai au t do co mot trong lng la
(33/140)qL = 0,3 kN (qL la trong lng dam).
He so ong se la:
Kd = 1+ 1+

2H

P
yt (1 + )
Q

= 1+ 1+

2(50)
= 24,21
0,3
0,143(1 +
)
1

ng suat do va cham la:


d ,max = t ,Q K d =

1(200)
.24,21 = 1,816 kN/cm 2
40.20 2 / 6

Ke them ng suat do trong lng dam:


d ,max =

M t ,max,q
Wx

qL2 / 2 0,64.2 2.100


=
= 0,096 kN/cm 2
Wx
40.20 2 / 6

ng suat ln nhat trong dam la:

Chng 13: Tai trong ong

http://www.ebook.edu.vn

GV: Le c Thanh

max = 1,816 + 0,096 = 1,912 kN/cm2


Khi ke en trong lng dam, ng suat ln nhat
giam.
o vong tai au t do:
yt = 0,143.(24,21) + 0,017 = 3,48 cm

V du 13.6 Dam ABC tiet dien I-24 chu va cham ng


bi mot trong
lng Q = 2 kN ri t do t o cao H =
50 cm (H.13.26.a), bo qua trong lng ban than dam,
tnh max; kiem tra ben. Cho: I-24 co: Ix = 3460 cm4, Wx
= 289 cm3, q = 0,273 kN/m; [] = 16 kN/cm2.

Chng 13: Tai trong ong

http://www.ebook.edu.vn

GV: Le c Thanh
Q = 2 kN

a)

H = 50 cm

I-24

L=6m

L/2
QL/2
C

A
B

Q = 2 kN
H = 50 cm

b)

c)

Clx = 5 kN/m

B
A
B

d)

A
B
qL 2/8

b) va c) He chu va cham co lo xo; d) Dam chu trong lng ban than

Bay gi, at mot lo xo co Clx = 5 kN/m tai C e


vat va cham Q (H.13.24.b), tnh lai he so ong va max;
xet lai ieu kien ben. Neu khong at C ma thay lo xo
vao goi ta tai B (H.13.26.c), he so ong la bao nhieu?
Cho: E = 2.104 kN/cm2; [] = 16 kN/cm2.
Giai.
Khong ke trong lng ban than dam.
Chuyen v do Q tac dung tnh tai C la:
yt =

1.(600) 3
QL3
=
= 0,39 cm
8EI x 8(2.10 4 ).3460

Chng 13: Tai trong ong

http://www.ebook.edu.vn

GV: Le c Thanh
He so ong:
Kd = 1+ 1+

2(50)
= 17,04
0,39

ng suat ln nhat tai B (H.13.21):


d ,max = t ,max,Q K d =
d ,max =

M x ,max
Wx

Kd =

Q.L
Kd
2.Wx

1.(600)
(17,04) = 17,69kN/cm 2 > [ ] = 16 kN/cm 2
2.289

Dam khong ben.


Chuyen v tai C: yC = 0,39(17,04) = 6,64 cm
Xet trng hp co lo xo at ngay tai iem va cham.
Chuyen v do Q tac dung tnh tai C la:
yt =

1.(600) 3
1
QL3
Q
+
=
+ = 0,39 + 0,2 = 0,59 cm
4
8EI x Clx 8(2.10 ).3460 5

He so ong :
Kd = 1+ 1+

2(50)
= 14,06
0,59

ng suat ln nhat tai B (H.13.24):


d ,max = t ,max,Q K d =

1.(300)
14,06 = 14.6 kN/cm 2
289

max < [] = 16 kN/cm2

dam thoa ieu kien ben.


Chuyen v cua dam tai C: yC = 0,39(14,06) = 5,48
cm
giam so vi trng hp tren.
Xet trng hp co lo xo at tai goi B.
Bay gi, chuyen v do Q tac dung tnh tai C la:
yt =

1.(600) 3
31
QL3 3 (3Q / 2)
+
=
+
= 0,39 + 0,3 = 0,69 cm
4
8EI x 2 Clx
8(2.10 ).3460 2 5

He so ong:

Kd = 1+ 1+

2(50)
= 13,08
0,69

ng suat ln nhat tai B (H.10.21):

Chng 13: Tai trong ong

http://www.ebook.edu.vn

GV: Le c Thanh
d ,max = t ,max,Q K d =

1.(300)
13,08 = 13,57 kN/cm 2
289

Chuyen v tai C: yC = 0,69(13,08) = 9,02 cm


Trong trng hp nay, ng suat giam nhng chuyen
v tang so vi khi at lo xo au t do.
BAI TAP CHNG 13
13.1

Mot vat nang P c nang len cao vi bang he


thong rong roc n gian nh tren H.13.24.a. Neu
keo day cap vi gia toc eu a, tnh lc cang tren day
cap. Neu dung he thong ba cap rong roc va cung
keo day vi gia toc a th lc cang la bao nhieu?
B

C
A = 5 m/s2
450

P
a)

Hnh 13.25

P = 2kN

b)

Hnh 13.26

13.2 Mot ket cau nang vat nang P chuyen ong len vi
gia toc a (H.13.26). Tnh noi lc phat sinh trong cac
thanh AB, BC va CD.
13.3 Mot tru AB co chieu cao H, dien tch mat cat
ngang la F, moun chong uon W, trong lng rieng
la mang mot vat nang P. Tru c gan chat vao
mot be van chuyen theo phng ngang vi gia toc a
(H.13.27).
Chng 13: Tai trong ong

http://www.ebook.edu.vn

GV: Le c Thanh
Xem tru b ngam tai tiet dien A vao be, xac nh ng
suat phap max, min tai mat cat nguy hiem cua tru.
a = 2 m/s2

P
B

F = 1 cm2

F, W,
F = 1 cm

4m

2m

Hnh 13.27

2m

Hnh 13.28

13.4 Xac nh ng suat phap ln nhat trong day cap va


trong dam I-24 do tac dung ong thi cua trong lc
va lc quan tnh khi he c keo len vi gia toc a
(H.13.28).
13.5 Mot truc tiet dien tron AB ng knh D mang mot
thanh CD tiet dien ch nhat b.h, au thanh CD co
mot vat nang trong lng P, he quay quanh truc AB
vi van toc n = 210 vg/ph (H.13.29). Tnh ng suat
ln nhat trong thanh CD va truc AB.
Cho: a = 1 m; D = 4 cm; h = 2b = 6 cm; P = 0,1 kN.
M2

a/2

a/2
B
a

Hnh 13.29

Chng 13: Tai trong ong

http://www.ebook.edu.vn

GV: Le c Thanh
Bo qua trong lng ban than cua he.
13.6 Tnh tan so goc va chu ky dao ong cua cac he ve
tren H.13.30, C1 va C2 la o cng cua lo xo.
C1

C1

C1

C1

C2

C1

C2

C2

C2

C1

C2

a)

b)

c)

d)

e)

Hnh 13.30

13.7 Mot dam n gian mat cat hnh ch I so 40 dai 8


m mang mot trong lng 20 kN gia nhp. Tnh tan
so rieng cua he khi co ke va khi khong ke en
trong lng dam.
13.8 Mot dam thep I24 mang mot mot nang 2 kN toc
o 200 vg/ph, lc quan tnh do khoi lng lech tam
la 0,2 kN (H.13.31). Bo qua trong lng ban than
cua dam va lo xo, xac nh ng suat ong ln nhat
trong dam trong cac trng hp sau:
a) Dam I24 at theo
phng ng (I)
b) Dam I24 at theo
phng ngang ( ).
13.9 Gia s hai goi ta
lo xo tren dam

n = 200vg/ph

Qo = 0,2 kN

Q = 2 kN

c =1,5 kN/cm

2m

c = 1,5 kN/cm

2m

Hnh 13.31

Chng 13: Tai trong ong

http://www.ebook.edu.vn

GV: Le c Thanh
H.13.31 c thay bang goi ta cng va at hai lo
xo di e mot nh H.13.32. Tnh lai ng suat va
o vong ln nhat trong dam theo ca hai trng hp
nh tren. Cho:
E = 2.104 kN/cm2.
n = 200 vg/ph
Q = 2 kN

Qo = 0,2 kN
c = 1,5 kN/cm

2m

2m

Hnh 13.32

13.10 Mot dam go tiet dien ch nhat b.h, co au mut


tha gan mot rong roc e a mot thung trong lng
Q cha vat nang P len cao. (H.13.33). Hay xet hai
trng hp:
a) Vat nang P c treo trong thung va thung c
keo len vi gia toc a = 2 m/s2. Bo qua trong lng
dam, day va rong roc, tnh ng suat ln nhat cua
dam. Cho: P = 0,5 kN; Q = 1 kN; L = 4 m.
b) Trong qua trnh dch chuyen vi gia toc a = 2 m/s2
vat nang P b ri xuong ay thung. Tnh lai ng suat
cua dam. Cho: H = 0,4 m.

Chng 13: Tai trong ong

http://www.ebook.edu.vn

GV: Le c Thanh
b.h

300
L/2

P
H = 0,4 m

Hnh 13.33

13.11 Mot trong lng P = 0,5 kN ri t mot o cao H =


10 cm xuong au C cua mot dam tiet dien ch nhat
b h = 20 40 cm2, dai L = 4 m (H.13.34.a). Tnh
ng suat va o vong ln nhat cua dam
Neu thay goi ta B bang mot lo xo co ng knh D
= 100 mm, ng knh si thep d = 10 mm, so vong
lam viec n = 10 (H.13.34.b). Tnh ng suat va o
vong ln nhat cua dam.
Cho: Edam = 2.104 kN/cm2, Gloxo = 8.103 kN/cm2.
P

b.h
A

L/2
a)

b.h
B

L/2

L
Hnh 13.34

b)

13.12 Xac nh ng suat cua dam khi vat b va cham


ngang (H.13.35). Cho: a = 2 m; b.h = 20 40 cm2.
Thanh DB tuyet oi cng.
Chng 13: Tai trong ong

http://www.ebook.edu.vn

GV: Le c Thanh
D
b.h

Q = 0,1 kN

V = 5 m/s
B

2a

Hnh 13.35

Chng 13: Tai trong ong

http://www.ebook.edu.vn

Vous aimerez peut-être aussi